Вы находитесь на странице: 1из 167

COMPILATION OF CASE

DIGESTS ON LAW ON
PUBLIC CORPORATIONS
ATTY. IRENE VALONES

CLASS SCHEDULE: MONDAY 7:30-9:30 P.M.

OCTOBER 11, 2018


CASE NO. 1
GREATER BALANGA DEVCORP VS. MUNICIPALITY OF BALANGA, BATAAN
G.R. No. 83987
December 27, 1994

Facts:
On January 11, 1988, petitioner applied with the Office of the Mayor of Balanga for a
business permit to engage in business in the said area. On the same day, Mayor Melanio
S. Banzon, Jr. issued Mayor’s Permit No. 2729, granting petitioner the privilege of a “real
estate dealer/privately-owned public market operator” under the trade name of Balanga
Central Market. The permit was to expire on December 31, 1988. Petitioner likewise
registered “Balanga Central Market” as a trade name with the Bureau of Trade Regulations
and Consumer Protection.

On February 19, 1988, however, the Sangguniang Bayan of Balanga passed Resolution
No. 12, s-88 annulling the Mayor’s permit issued to petitioner and advising the Mayor to
revoke the permit “to operate a public market.” Pursuant to said Resolution, Mayor
Banzon, on March 7, 1988, issued Executive Order No. 1, s-88 revoking the permit insofar
as it authorized the operation of a public market.

Petitioner filed the instant petition with a prayer for the issuance of a writ of preliminary
mandatory and prohibitory injunction or restraining order aimed at the reinstatement of
the Mayor’s permit and the curtailment of the municipality’s collection of market fees and
market entrance fees.

Respondent asserted that as the local chief executive, the Mayor may issue, deny or
revoke municipal licenses and permits. They contended that Resolution No. 12, s-88 of
the Sangguniang Bayan, the basis of Executive Order No. 1, s-88, was a legitimate exercise
of local legislative authority and, as such, the revocation of petitioner’s permit was not
tainted with any grave abuse of discretion.

Issues:
A. Whether or not the revocation of the permit issued by the Mayor is valid.
B. Whether or not the landowner can be deprived of the land.

Ruling:
A. No. The powers of municipal corporations are to be construed in strictissimi juris and
any doubt or ambiguity must be construed against the municipality. The authority of
the Mayor to revoke permits is premised on a violation by the grantee of any of its
conditions for its grant. For revocation to be justified under the Balanga Revenue Code,

Page 1 of 166
there must be: 1) proof of willful misrepresentation, and 2) deliberate intent to make
a false statement. Good faith is always presumed.

The application for Mayor's permit in the present case requires the applicant to state
the “type of business, profession, occupation, privileges applied for.” Petitioner left this
entry blank in its application form. It is only in the Mayor’s permit itself that petitioner’s
lines of business appear, which in this case are two separate types, one as real estate
dealer and another as public market operator. Revocation is not justified because
Petitioner did not make any false statement therein.

Neither was petitioner’s applying for two businesses in one permit a ground for
revocation. The second paragraph of Section 3A-06(b) does not expressly require two
permits for the conduct of two or more businesses in one place, but only that separate
fees be paid for each business. In addition, the manner by which the Mayor revoked
the permit transgressed petitioner’s right to due process. The alleged violation of
Section 3A-06(b) of the Balanga Revenue Code was not stated in the order of
revocation, and neither was petitioner informed of this specific violation until the
Rejoinder was filed in the instant case. In fact, with all the more reason should due
process have been observed in view of the questioned Resolution of the Sangguniang
Bayan.

B. No. The Sangguniang Bayan's Resolution merely mentioned the plan to acquire the
Lot for expansion of the Balanga Public Market adjacent thereto. The Sangguniang
Bayan did not actually establish or maintain a public market on the area. Until
expropriation proceedings are instituted in court, the landowner cannot be deprived
of its right over the land. Of course, the Sangguniang Bayan has the duty in the exercise
of its police powers to regulate any business subject to municipal license fees and
prescribe the conditions under which a municipal license already issued may be
revoked.

Page 2 of 166
CASE NO. 2
AQUILINO Q. PIMENTEL, JR. VS. Hon. ALEXANDER AGUIRRE
G.R. No. 132988
July 19, 2000

Facts:
On December 27, 1997, the President Fidel V. Ramos issued AO 372. Section 1 and Section
4 of which provides respectively:

“ xxx All government departments and agencies, including state universities and
colleges, government-owned and controlled corporations and local governments
units will identify and implement measures in FY 1998 that will reduce total
expenditures for the year by at least 25% of authorized regular appropriations for
non-personal services items, along the following suggested areas: xxx ”

“Pending the assessment and evaluation by the Development Budget


Coordinating Committee of the emerging fiscal situation, the amount equivalent
to 10% of the internal revenue allotment to local government units shall be
withheld.”

Subsequently, on December 10, 1998, President Joseph E. Estrada issued AO 43, amending
Section 4 of AO 372, by reducing to five percent (5%) the amount of internal revenue
allotment (IRA) to be withheld from the LGUs.

Petitioner contends that the President, in issuing AO 372, was in effect exercising the
power of control over LGUs. The Constitution vests in the President, however, only the
power of general supervision over LGUs, consistent with the principle of local autonomy;
and the directive to withhold ten percent (10%) of their IRA is in contravention of Section
286 of the Local Government Code and of Section 6, Article X of the Constitution,
providing for the automatic release to each of these units its share in the national internal
revenue.

The Solicitor General claims on the other hand that AO 372 was issued to alleviate the
“economic difficulties brought about by the peso devaluation” and constituted merely an
exercise of the President’s power of supervision over LGUs; and the withholding is
“temporary in nature pending the assessment and evaluation by the Development
Coordination Committee of the emerging fiscal situation.”

Issues:
A. Whether or not Section 1 of AO 372, insofar as it “directs” LGUs to reduce their
expenditures by 25 percent, violates fiscal autonomy.

Page 3 of 166
B. Whether or not Section 4 of the same issuance, which withholds 10 percent of their
internal revenue allotments, are valid exercises of the President’s power of general
supervision over local governments.

Ruling:
A. No. Section 1 of AO 372 is not violative of the fiscal autonomy. Local fiscal autonomy
does not, however, rule out any manner of national government intervention by way
of supervision, in order to ensure that local programs, fiscal and otherwise, are
consistent with national goals. Significantly, the President, by constitutional fiat, is the
head of the economic and planning agency of the government, primarily responsible
for formulating and implementing continuing, coordinated and integrated social and
economic policies, plans and programs for the entire country. However, under the
Constitution, the formulation and the implementation of such policies and programs
are subject to “consultations with the appropriate public agencies, various private
sectors, and local government units.” The President cannot do so unilaterally.

While the wordings of Section 1 of AO 372 have a rather commanding tone, and while
we agree with petitioner that the requirements of Section 284 of the Local Government
Code have not been satisfied, we are prepared to accept the solicitor general’s
assurance that the directive to “identify and implement measures x x x that will reduce
total expenditures x x x by at least 25% of authorized regular appropriation” is merely
advisory in character, and does not constitute a mandatory or binding order that
interferes with local autonomy. The language used, while authoritative, does not
amount to a command that emanates from a boss to a subaltern. Rather, the provision
is merely an advisory to prevail upon local executives to recognize the need for fiscal
restraint in a period of economic difficulty.

B. No. Section 4 of AO 372 cannot, however, be upheld. A basic feature of local fiscal
autonomy is the automatic release of the shares of LGUs in the national internal
revenue. This is mandated by no less than the Constitution. The Local Government
Code specifies further that the release shall be made directly to the LGU concerned
within five (5) days after every quarter of the year and “shall not be subject to any lien
or holdback that may be imposed by the national government for whatever purpose.”
As a rule, the term “shall” is a word of command that must be given a compulsory
meaning. The provision is, therefore, imperative.

Page 4 of 166
CASE NO. 3
LEAGUE OF CITIES OF THE PHILIPPINES (LCP) VS. COMMISSION ON ELECTIONS
G.R. No. 176951
November 18, 2008

Facts:
These cases were initiated by the consolidated petitions for prohibition filed by the
League of Cities of the Philippines (LCP), City of Iloilo, City of Calbayog, and Jerry P. Treñas,
assailing the constitutionality of the sixteen (16) laws, each converting the municipality
covered thereby into a component city (Cityhood Laws), and seeking to enjoin the
Commission on Elections (COMELEC) from conducting plebiscites pursuant to the subject
laws. In the Decision dated November 18, 2008, the Court En Banc, by a 6-5 vote, granted
the petitions and struck down the Cityhood Laws as unconstitutional for violating Sections
10 and 6, Article X, and the equal protection clause. In another Decision dated December
21, 2009, the Court En Banc, by a vote of 6-4, declared the Cityhood Laws as constitutional.
On August 24, 2010, the Court En Banc, through a Resolution, by a vote of 7-6, resolved
the Ad Cautelam Motion for Reconsideration and Motion to Annul the Decision of
December 21, 2009.

Issues:
A. Whether or not the Cityhood Bills violate Article X, Section 10 of the Constitution
B. Whether or not the Cityhood Bills violate Article X, Section 6 and the equal protection
clause of the Constitution

Ruling:
A. No, the Cityhood Bills are constitutional. The enactment of the Cityhood Laws is an
exercise by Congress of its legislative power. Legislative power is the authority, under
the Constitution, to make laws, and to alter and repeal them. The Constitution, as the
expression of the will of the people in their original, sovereign, and unlimited capacity,
has vested this power in the Congress of the Philippines. The LGC is a creation of
Congress through its law-making powers. Congress has the power to alter or modify
it as it did when it enacted R.A. No. 9009. Such power of amendment of laws was
again exercised when Congress enacted the Cityhood Laws. When Congress enacted
the LGC in 1991, it provided for quantifiable indicators of economic viability for the
creation of local government units—income, population, and land area. However,
Congress deemed it wiser to exempt respondent municipalities from such a belatedly
imposed modified income requirement in order to uphold its higher calling of putting
flesh and blood to the very intent and thrust of the LGC, which is countryside
development and autonomy, especially accounting for these municipalities as engines
for economic growth in their respective provinces.

Page 5 of 166
R.A. No. 9009 amended the LGC. But the Cityhood Laws amended R.A. No. 9009
through the exemption clauses found therein. Since the Cityhood Laws explicitly
exempted the concerned municipalities from the amendatory R.A. No. 9009, such
Cityhood Laws are, therefore, also amendments to the LGC itself.

B. Substantial distinction lies in the capacity and viability of respondent municipalities to


become component cities of their respective provinces. Congress, by enacting the
Cityhood Laws, recognized this capacity and viability of respondent municipalities to
become the State’s partners in accelerating economic growth and development in the
provincial regions, which is the very thrust of the LGC, manifested by the pendency of
their cityhood bills during the 11th Congress and their relentless pursuit for cityhood
up to the present. The Resolution dated August 24, 2010 is REVERSED and SET ASIDE.
The Cityhood Laws are declared CONSTITUTIONAL.

Page 6 of 166
CASE NO. 4
PHILIPPINE ASSOCIATION OF SERVICE EXPORTERS, INC (PASEI) VS. HON. DRILON
G.R. No. 81958
June 30, 1988

Facts:
PASEI, a firm "engaged principally in the recruitment of Filipino workers, male and female,
for overseas placement," challenges the Constitutional validity of Department Order No.
1, Series of 1988, of the Department of Labor and Employment, in the character of
"GUIDELINES GOVERNING THE TEMPORARY SUSPENSION OF DEPLOYMENT OF FILIPINO
DOMESTIC AND HOUSEHOLD WORKERS," in this petition for certiorari and prohibition.
Specifically, the measure is assailed for "discrimination against males or females;" that it
"does not apply to all Filipino workers but only to domestic helpers and females with
similar skills;" and that it is violative of the right to travel. It is held likewise to be an invalid
exercise of the lawmaking power, police power being legislative, and not executive, in
character. DO 1 is finally claimed to be in violation of the Charter's non-impairment clause,
in addition to the "great and irreparable injury" that PASEI members face should the Order
be further enforced.

Issue:
Whether or not Department Order No. 1 is constitutional.

Ruling:
Yes. The concept of police power is well-established in this jurisdiction. It has been defined
as the "state authority to enact legislation that may interfere with personal liberty or
property in order to promote the general welfare. As a general rule, official acts enjoy a
presumed validity. In the absence of clear and convincing evidence to the contrary, the
presumption logically stands. The petitioner has shown no satisfactory reason why the
contested measure should be nullified. There is no question that Department Order No.
1 applies only to "female contract workers," but it does not thereby make an undue
discrimination between the sexes. The Court finds, finally, the impugned guidelines to be
applicable to all female domestic overseas workers. That it does not apply to "all Filipina
workers" is not an argument for unconstitutionality. Had the ban been given universal
applicability, then it would have been unreasonable and arbitrary. For obvious reasons,
not all of them are similarly circumstanced. What the Constitution prohibits is the singling
out of a select person or group of persons within an existing class, to the prejudice of
such a person or group or resulting in an unfair advantage to another person or group of
persons.

Page 7 of 166
Relevant Doctrine:
The non-impairment clause of the Constitution, invoked by the petitioner, must yield to
the loftier purposes targeted by the Government. Freedom of contract and enterprise, like
all other freedoms, is not free from restrictions, more so in this jurisdiction, where laissez
faire has never been fully accepted as a controlling economic way of life.

Page 8 of 166
CASE NO. 5
MUNICIPALITY OF SAN NARCISO VS HON. MENDEZ
G.R. No. 103702
December 06, 1994

Facts:
On 20 August 1959, President Carlos P. Garcia, issued, pursuant to the then Sections 68
and 2630 of the Revised Administrative Code, as amended, Executive Order No. 353
creating the municipal district of San Andres, Quezon, by segregating from the
municipality of San Narciso of the same province, the barrios of San Andres, Mangero,
Alibijaban, Pansoy, Camflora and Tala along with their respective sitios. By virtue of
Executive Order No. 174, dated 05 October 1965, issued by President Diosdado
Macapagal, the municipal district of San Andres was later officially recognized to have
gained the status of a fifth-class municipality. On 05 June 1989, the Municipality of San
Narciso filed a petition for quo warranto with the Regional Trial Court, Branch 62, in
Gumaca, Quezon, against the officials of the Municipality of San Andres. The petition
sought the declaration of nullity of Executive Order No. 353 and prayed that the
respondent local officials of the Municipality of San Andres be permanently ordered to
refrain from performing the duties and functions of their respective offices.

The petitioning municipality contended that Executive Order No. 353, a presidential act,
was a clear usurpation of the inherent powers of the legislature and in violation of the
constitutional principle of separation of powers. Hence, petitioner municipality argued,
the officials of the Municipality or Municipal District of San Andres had no right to exercise
the duties and functions of their respective offices that rightfully belonged to the
corresponding officials of the Municipality of San Narciso.

In their answer, respondents asked for the dismissal of the petition saying that since it
was at the instance of petitioner municipality that the Municipality of San Andres was
given life with the issuance of Executive Order No. 353, petitioner municipality should be
deemed estopped from questioning the creation of the new municipality; that because
the Municipality of San Andres had been in existence since 1959, its corporate personality
could no longer be assailed. On 27 November 1991, the Municipality of San Andres filed
anew a motion to dismiss alleging that the case had become moot and academic with the
enactment of Republic Act No. 7160, otherwise known as the Local Government Code of
1991, which took effect on 01 January 1991. The movant municipality cited Section 442(d)
of the law, reading thusly:

Sec. 442. Requisites for Creation. — . . .


(d) Municipalities existing as of the date of the effectivity of this Code shall
continue to exist and operate as such. Existing municipal districts organized

Page 9 of 166
pursuant to presidential issuances or executive orders and which have their
respective set of elective municipal officials holding office at the time of the
effectivity of this Code shall henceforth be considered as regular
municipalities.

In its Order of 02 December 1991, the lower court finally dismissed the petition for lack
of cause of action on what it felt was a matter that belonged to the State, adding that
"whatever defects present in the creation of municipal districts by the President pursuant
to presidential issuances and executive orders, were cured by the enactment of R.A. 7160.
Hence, this petition "for review on certiorari."

Issue:
Whether or not Sec. 442(d) of RA7160 violates due process and equal protection clause
and does not apply to Quo Warranto proceedings filed prior to enactment of said LGC.

Ruling:
No, it does not violate due process and equal protection clause. No pretension of
unconstitutionality per se of Section 442(d) of the Local Government Code is preferred. It
is doubtful whether such a pretext, even if made, would succeed. The power to create
political subdivisions is a function of the legislature. Congress did just that when it has
incorporated Section 442(d) in the Code. Curative laws, which in essence are retrospective,
and aimed at giving "validity to acts done that would have been invalid under existing
laws, as if existing laws have been complied with," are validly accepted in this jurisdiction,
subject to the usual qualification against impairment of vested rights.

Relevant Doctrine:
“A quo warranto proceeding assailing the lawful authority of a political subdivision must
be timely raised. Public interest demands it.”

Page 10 of 166
CASE NO. 6
EMMANUEL PELAEZ VS. THE AUDITOR GENERAL
G.R. No. L-23825
December 24, 1965

Facts:
The President of the Philippines, purporting to act pursuant to Section 68 of the Revised
Administrative Code, issued Executive Orders Nos. 93 to 121, 124 and 126 to 129; creating
thirty-three (33) municipalities enumerated in the margin. Soon after the date last
mentioned, or on November 10, 1964, petitioner Emmanuel Pelaez, as Vice President of
the Philippines and as taxpayer, instituted the present special civil action, for a writ of
prohibition with preliminary injunction, against the Auditor General, to restrain him, as
well as his representatives and agents, from passing in audit any expenditure of public
funds in implementation of said executive orders and/or any disbursement by said
municipalities.

Petitioner alleges that said executive orders are null and void, upon the ground that said
Section 68 has been impliedly repealed by Republic Act No. 2370 and constitutes an
undue delegation of legislative power. Petitioner’s contends that the third paragraph of
Section 3 of Republic Act No. 2370, reads:

“Barrios shall not be created or their boundaries altered nor their names changed
except under the provisions of this Act or by Act of Congress”

Respondent’s contented that the power of the President to create municipalities under
this section does not amount to an undue delegation of legislative power, relying upon
Municipality of Cardona vs. Municipality of Binañgonan (36 Phil. 547), which, he claims,
has settled it. Such claim is untenable, for said case involved, not the creation of a new
municipality, but a mere transfer of territory — from an already existing municipality
(Cardona) to another municipality (Binañgonan), likewise, existing at the time of and prior
to said transfer— in consequence of the fixing and definition, pursuant to Act No. 1748,
of the common boundaries of two municipalities.

Issue:
Whether or not the executive order that was issued by the President creating 33
municipalities is an exercise of undue delegation of legislative power.

Ruling:
Yes, there is undue delegation of legislative power. The power of control of the President
is denied by the Constitution to the Executive, insofar as local governments are concerned.

Page 11 of 166
Although Congress may delegate to another branch of the Government the power to fill
in the details in the execution, enforcement or administration of a law, it is essential, to
forestall a violation of the principle of separation of powers, that said law: (a) be complete
in itself — it must set forth therein the policy to be executed, carried out or implemented
by the delegate2 — and (b) fix a standard — the limits of which are sufficiently
determinate or determinable — to which the delegate must conform in the performance
of his functions. Indeed, without a statutory declaration of policy, the delegate would in
effect, make or formulate such policy, which is the essence of every law; and, without the
aforementioned standard, there would be no means to determine, with reasonable
certainty, whether the delegate has acted within or beyond the scope of his authority.

Upon the other hand if the President could create a municipality, he could, in effect,
remove any of its officials, by creating a new municipality and including therein the barrio
in which the official concerned resides, for his office would thereby become vacant. Thus,
by merely brandishing the power to create a new municipality (if he had it), without
actually creating it, he could compel local officials to submit to his dictation, thereby, in
effect, exercising over them the power of control denied to him by the Constitution.

Then, also, the power of control of the President over executive departments, bureaus or
offices implies no more than the authority to assume directly the functions thereof or to
interfere in the exercise of discretion by its officials. Manifestly, such control does not
include the authority either to abolish an executive department or bureau, or to create a
new one.

Page 12 of 166
CASE NO. 7
THE SOLICITOR GENERAL VS. THE METROPOLITAN MANILA AUTHORITY
G.R. No. 102782
December 11, 1991

Facts:
In Metropolitan Traffic Command, West Traffic District vs. Hon. Arsenio M. Gonong, G.R.
No. 91023, promulgated on July 13, 1990, the Court held that the confiscation of the
license plates of motor vehicles for traffic violations was not among the sanctions that
could be imposed by the Metro Manila Commission under PD 1605 and was permitted
only under the conditions laid down by LOI 43 in the case of stalled vehicles obstructing
the public streets. It was there also observed that even the confiscation of driver's licenses
for traffic violations was not directly prescribed by the decree nor was it allowed by the
decree to be imposed by the Commission.

Subsequently, the following developments transpired: (1) Several, letters were sent to the
court for the alleged confiscation of licenses; (2) On May 24, 1990, the Metropolitan
Manila Authority issued Ordinance No. 11, Series of 1991, authorizing itself "to detach the
license plate/tow and impound attended/ unattended/ abandoned motor vehicles
illegally parked or obstructing the flow of traffic in Metro Manila."; (3) It was held that the
license plates of motor vehicles may not be detached except only under the conditions
prescribed in LOI 43; and (4) The Court has received several complaints against the
confiscation by police authorities of driver's licenses for alleged traffic violations, which
sanction is, according to the said decision, not among those that may be imposed under
PD 1605.

On July 2, 1991, the Court issued the following resolution:

“The attention of the Court has been called to the enactment by the
Metropolitan Manila Authority of Ordinance No. 11, Series of 1991,
providing inter alia that:

Section 2. Authority to Detach Plate/Tow and Impound. The Metropolitan


Manila Authority, thru the Traffic Operatiom Center, is authorized to detach
the license plate/tow and impound attended/unattended/abandoned
motor vehicles illegally parked or obstructing the flow of traffic in Metro
Manila.”

In its Comment, the Metropolitan Manila Authority defended the said ordinance on the
ground that it was adopted pursuant to the powers conferred upon it by EO 392. It

Page 13 of 166
particularly cited Section 2 thereof vesting in the Council (its governing body) the
responsibility among others of:

1. Formulation of policies on the delivery of basic services requiring coordination or


consolidation for the Authority; and

2. Promulgation of resolutions and other issuances of metropolitan wide application,


approval of a code of basic services requiring coordination, and exercise of its
rule-making powers. (Emphasis supplied)

The Authority argued that there was no conflict between the decision and the ordinance
because the latter was meant to supplement and not supplant the latter. It stressed that
the decision itself said that the confiscation of license plates was invalid in the absence of
a valid law or ordinance, which was why Ordinance No. 11 was enacted. The Authority
also pointed out that the ordinance could not be attacked collaterally but only in a direct
action challenging its validity.

Solicitor General expressed the view that the ordinance was null and void because it
represented an invalid exercise of a delegated legislative power. The flaw in the measure
was that it violated existing law, specifically PD 1605, which does not permit, and so
impliedly prohibits, the removal of license plates and the confiscation of driver's licenses
for traffic violations in Metropolitan Manila. He made no mention, however, of the alleged
impropriety of examining the said ordinance in the absence of a formal challenge to its
validity.

CA ruled that there is a valid delegation of legislative power to promulgate such measures,
it appearing that the requisites of such delegation are present. These requisites are. 1) the
completeness of the statute making the delegation; and 2) the presence of a sufficient
standard.

Issue:
Whether or not the exercise of the delegated power is valid.

Ruling:
No, the exercise of delegated power is not valid. The requirement that the municipal
enactment must not violate existing law explains itself. Local political subdivisions are able
to legislate only by virtue of a valid delegation of legislative power from the national
legislature (except only that the power to create their own sources of revenue and to levy
taxes is conferred by the Constitution itself).

Page 14 of 166
They are mere agents vested with what is called the power of subordinate legislation. As
delegates of the Congress, the local government unit cannot contravene but must obey
at all times the will of their principal. In the case before us, the enactments in question,
which are merely local in origin, cannot prevail against the decree, which has the force
and effect of a statute.

The self-serving language of Section 2 of the challenged ordinance is worth noting.


Curiously, it is the measure itself, which was enacted by the Metropolitan Manila Authority,
that authorizes the Metropolitan Manila Authority to impose the questioned sanction. At
any rate, the fact is that there is no statutory authority for — and indeed there is a
statutory prohibition against — the imposition of such penalties in the Metropolitan
Manila area. Hence, regardless of their merits, they cannot be impose by the challenged
enactments by virtue only of the delegated legislative powers.

It is for Congress to determine, in the exercise of its own discretion, whether or not to
impose such sanctions, either directly through a statute or by simply delegating authority
to this effect to the local governments in Metropolitan Manila. Without such action, PD
1605 remains effective and continues prohibit the confiscation of license plates of motor
vehicles (except under the conditions prescribed in LOI 43) and of driver licenses as well
for traffic violations in Metropolitan Manila.

WHEREFORE, judgment is hereby rendered: (1) declaring Ordinance No.11, Seriesof


l991,of theMetropolitan Manila Authority and Ordinance No. 7, Series of 1988 of the
Municipality of Mandaluyong, NULL and VOID; and (2) enjoining all law enforcement
authorities in Metropolitan Manila from removing the license plates of motor vehicles
(except when authorized under LOI 43) and confiscating driver licenses for traffic
violations within the said area.

Page 15 of 166
CASE NO. 8
MUNICIPALITY OF KAPALONG VS. MOYA
G.R. No. L-41322
September 29, 1988

Facts:
From portions of the Municipality of Kapalong, President Carlos P. Garcia created
respondent Municipality of Santo Tomas, and the latter now asserts jurisdiction over eight
(8) barrios of petitioner. For many years and on several occasions, this conflict of
boundaries between the two municipalities was brought, at the instance of Municipality
of Sto. Tomas thru Solis, to the Provincial Board of Davao for it to consider and decide.
However, no action was taken on the same. Mayor Solis then filed a complaint with the
then CFI of Davao, presided by Judge Moya against the Municipality of Kapalong, for
settlement of the municipal boundary dispute, recovery of collected taxes and damages,
docketed therein as Civil Case No. 475. Municipality of Kapalong thru Mayor Royo, filed a
Motion to Dismiss on the ground of lack of jurisdiction of the lower court and lack of legal
personality of the Municipality of Santo Tomas which Judge Moya, denied.

Issue:
Whether or not the Municipality of Santo Tomas legally exists.

Ruling:
No. The Court finds the contention of the petitioner meritorious. Petitioner contends that
the ruling of this Court in Pelaez v. Auditor General (15 SCRA 569) is clear that the
President has no power to create municipalities. Thus, there is no Municipality of Santo
Tomas to speak of. It has no right to assert, no cause of action, no corporate existence at
all, and it must perforce remain part and parcel of Kapalong. Based on this premise, it
submits that respondent Judge should have dismissed the case. Rule 3, Section 1 of the
Rules of Court expressly provides that only "entities authorized by law may be parties in
a civil action." Now then, as ruled in the Pelaez case supra, the President has no power to
create a municipality. Since private respondent has no legal personality, it can not be a
party to any civil action, and as such, respondent Judge should have dismissed the case,
since further proceedings would be pointless.

Relevant Doctrine:
President has no power to create municipalities. (Pursuant to the Court ruling in Pelaez v
Auditor General 15 SCRA 569.)

Page 16 of 166
CASE NO. 9
BENJAMIN E. CAWALING, JR., VS. THE COMMISSION ON ELECTIONS
G.R. No. 146319
October 26, 2001

Facts:
On August 16, 2000, former President Joseph E. Estrada signed into law R.A. No. 8806, an
"Act Creating The City of Sorsogon By Merging The Municipalities Of Bacon And Sorsogon
In The Province Of Sorsogon, And Appropriating Funds Therefor."

Pursuant to Section 10, Article X of the Constitution, 2 the Commission on Elections


(COMELEC), on December 16, 2000, conducted a plebiscite in the Municipalities of Bacon
and Sorsogon and submitted the matter for ratification. Consequently, the Plebiscite City
Board of Canvassers (PCBC) proclaimed the creation of the City of Sorsogon as having
been ratified and approved by the majority of the votes cast in the plebiscite.

Invoking his right as a resident and taxpayer of the former Municipality of Sorsogon,
Benjamin E. Cawaling, Jr. filed the petition for certiorari seeking the annulment of the
plebiscite on the following grounds: (a) plebiscite was conducted beyond the required
120-day period from the approval of R.A. 8806, in violation of Section 54 thereof; and (b)
COMELEC failed to observe the legal requirement of twenty (20) day extensive information
campaign in the Municipalities of Bacon and Sorsogon before conducting the plebiscite.
Two days after Cawaling also instituted another petition for prohibition, seeking to enjoin
the further implementation of R.A. No. 8806 for being unconstitutional.

Issues:
A. Whether or not the creation of Sorsogon City by merging two municipalities violates
Section 450(a) of the Local Government Code of 1991 (in relation to Section 10, Article
X of the Constitution);

B. Wheather or not the plebiscite conducted pursuant thereto was valid.

Ruling:
A. No. The Court found no reason to rule RA No. 8806 as unconstitutional. Under Section
450(a) of the Local Government Code, a municipality or a cluster of barangays may be
converted into a component city. This is allowed under Sec. 10, Art. X of the
Constitution. Petitioner questioned the creation of the City of Sorsogon by RA No.
8806. The Court, however, is not competent to rule on the wisdom of the law. The
Court also rejected the allegation that RA No. 8806 contravened the "one subject-one
bill rule." The only subject embraced in RA No. 8806 is the creation of the City of

Page 17 of 166
Sorsogon; the abolition of the two municipalities concerned are the inevitable
consequence of the merger.

B. Yes. On the plebiscite conducted within 120 days from the "effectivity" of the law, that
is, the completion of its publication, the Court found the same proper. The provision
in RA No. 8806 requiring a plebiscite within 120 days from the "approval" of the Act
should be read in harmony with the fundamental law to avoid inconsistencies or
repugnancy to established jurisprudence.

Relevant Doctrine:

The creation of a city is prescribed in Sec. 450 of the Local Gov’t. Code of 1991. Sec. 10,
Art. X of the Constitution allows the merger of local government units to create a province,
city, municipality or barangay in accordance with the criteria established by the Code.

The creation of an entirely new local government unit through a division or a merger of
existing local government units is recognized under the Constitution, provided that such
merger or division shall comply with the requirements prescribed by the Code.

Page 18 of 166
CASE NO. 10
ARDONA VS. REYES AND THE PHILLIPPINE TOURISM AUTHORITY
G.R. Nos. L-60549 60553 to 60555
October 26, 1983

Facts:
The Philippine Tourism Authority (PTA) filed four (4) complaints with the Court of First
Instance (CFI) of Cebu City for the expropriation of some 282 hectares of rolling land
situated in barangay Malubog and Babag, Cebu City, under PTA’s express authority “to
acquire by purchase, by negotiation or by condemnation proceedings any private land
within and without the tourist zones” for the purposes indicated in Section 5, paragraph
B (2), of its Revised Charter (PD 564).

The heirs of Juancho Ardona et. al, filed their oppositions, and had a common allegation
in that the taking is allegedly not impressed with public use under the Constitution,
alleging that there is no specific constitutional provision authorizing the taking of private
property for tourism purposes; and that assuming that PTA has such power, the intended
use cannot be paramount to the determination of the land as a land reform area; that
limiting the amount of compensation by legislative fiat is constitutionally repugnant; and
that since the land is under the land reform program, it is the Court of Agrarian Relations
and not the CFI, that has jurisdiction over the expropriation cases.

The Philippine Tourism Authority having deposited with the Philippine National Bank,
Cebu City Branch, an amount equivalent to 10% of the value of the properties pursuant
to Presidential Decree No. 1533, the lower court issued separate orders authorizing PTA
to take immediate possession of the premises and directing the issuance of writs of
possession. The Heirs of Ardona, et. al. then filed a petition for certiorari with preliminary
injunction before the Supreme Court.

Issue:
Whether of not the exporpriation of parcels of land for the purpose of contructing a sports
complex by the Philippine Tourism Authority be considered taking for public use.

Ruling:
There are three provisions of the Constitution which directly provide for the exercise of
the power of eminent domain, Section 2, Article IV states that private property shall not
be taken for public use without just compensation. Section 6, Article XIV allows the State,
in the interest of national welfare or defense and upon payment of just compensation to
transfer to public ownership, utilities and other private enterprises to be operated by the
government. Section 13, Article XIV states that the Batasang Pambansa may authorize

Page 19 of 166
upon payment of just compensation the expropriation of private lands to be subdivided
into small lots and conveyed at cost to deserving citizens.

In the leading case of Visayan Refining Co. v. Camus, this Court emphasized that the power
of eminent domain is inseparable from sovereignty being essential to the existence of the
State and inherent in government even in its most primitive forms. The only purpose of
the provision in the Bill of Rights is to provide some form of restraint on the sovereign
power. The constitutional restraints are public use and just compensation.

There can be no doubt that expropriation for such traditional purposes as the construction
of roads, bridges, ports, waterworks, schools, electric and telecommunications systems,
hydroelectric power plants, markets and slaughterhouses, parks, hospitals, government
office buildings, and flood control or irrigation systems is valid. However, the concept of
public use is not limited to traditional purposes. Here as elsewhere the idea that "public
use" is strictly limited to clear cases of "use by the public" has been discarded.

In addition, the petitioner's contention that the promotion of tourism is not "public use"
because private concessioners would be allowed to maintain various facilities such as
restaurants, hotels, stores, etc. inside the tourist complex is impressed with even less merit.
Private bus firms, taxicab fleets, roadside restaurants, and other private businesses using
public streets and highways do not diminish in the least bit the public character of
expropriations for roads and streets. The lease of store spaces in underpasses of streets
built on expropriated land does not make the taking for a private purpose. Airports and
piers catering exclusively to private airlines and shipping companies are still for public
use. The expropriation of private land for slum clearance and urban development is for a
public purpose even if the developed area is later sold to private homeowners,
commercial firms, entertainment and service companies, and other private concerns.

Page 20 of 166
CASE NO. 11
MUNICIPALITY OF PARAÑAQUE VS. V.M. REALTY CORPORATION
G.R. No. 127820
July 20, 1998

Facts:
Pursuant to Sangguniang Bayan Resolution No. 93-95, Series of 1993, petitioner
Municipality of Parañaque filed with the Regional Trial Court of Makati, Branch 134, on
September 20, 1993 a complaint for expropriation against private respondent V.M. Realty
Corporation over two parcels of land with a combined area of about 10,000 square meters
located at Wakas, San Dionisio, Parañaque, Metro Manila and covered by Torrens
Certificate of Title No. 48700. Allegedly, the complaint was filed for the purpose of
alleviating the living conditions of the underprivileged by providing homes for the
homeless through a socialized housing project.

The Municipality of Parañaque, pursuant to its Sangguniang Bayan Resolution No. 577,
Series of 1991, previously made an offer to enter into a negotiated sale of the property
with private respondent, which the latter did not accept. In an Order dated February 4,
1994, the trial court authorized petitioner to take possession of the subject property upon
deposit with its clerk of court of an amount equivalent to 15 percent of its fair market
value based on its current tax declaration. V.M. Realty Corporation filed its answer alleging
in the main that (a) the complaint failed to state a cause of action because it was filed
pursuant to a resolution and not to an ordinance as required by the Local Government
Code and (b) the cause of action, if any, was barred by prior judgment or res judicata.

On March 24, 1994, petitioner filed its opposition, stressing that the trial court's Order
dated February 4, 1994 was in accord with Section 19 of RA 7160, and that the principle
of res judicata was not applicable. Petitioner contends that a resolution approved by the
municipal council for the purpose of initiating an expropriation case "substantially
complies with the requirements of the law" because the terms "ordinance" and
"resolution" are synonymous for "the purpose of bestowing authority [on] the local
government unit through its chief executive to initiate the expropriation proceedings in
court in the exercise of the power of eminent domain."

The trial court then nullified its February 4, 1994 order and dismissed the case. On
appeal, the Court of Appeals affirmed the trial court's resolution. Hence, this petition.

Issues:
A. Whether or not a resolution duly approved by the municipal council has the same
force and effect of an ordinance and will not deprive an expropriation case of a valid
cause of action.

Page 21 of 166
B. Whether or not the principle of res judicata as a ground for dismissal of the case is not
applicable when public interest is primarily involved.

Ruling:
A. No. The power of eminent domain is lodged in the legislative branch of government,
which may delegate the exercise thereof to LGUs, other public entities and public
utilities. An LGU may therefore exercise the power to expropriate private property only
when authorized by Congress and subject to the latter's control and restraints imposed
"through the law conferring the power or in other legislations." Section 19 of RA 7160,
which delegates to LGUs the power of eminent domain, also lays down the parameters
for its exercise. It provides as follows: "Section 19. Eminent Domain. A local
government unit may, through its chief executive and acting pursuant to an ordinance,
exercise the power of eminent domain for public use, or purpose, or welfare for the
benefit of the poor and the landless, upon payment of just compensation, pursuant to
the provisions of the Constitution and pertinent laws: . . .

In addition, the following essential requisites must concur before an LGU can exercise
the power of eminent domain:

1. An ordinance is enacted by the local legislative council authorizing the local


chief executive, in behalf of the LGU, to exercise the power of eminent
domain or pursue expropriation proceedings over a particular private
property.
2. The power of eminent domain is exercised for public use, purpose or
welfare, or for the benefit of the poor and the landless.
3. There is payment of just compensation, as required under Section 9, Article
III of the Constitution, and other pertinent laws.
4. A valid and definite offer has been previously made to the owner of the
property sought to be expropriated, but said offer was not accepted.

In the case at bar, the local chief executive sought to exercise the power of eminent
domain pursuant to a resolution of the municipal council. Thus, there was no
compliance with the first requisite that the mayor be authorized through an ordinance.
If Congress intended to allow LGUs to exercise eminent domain through a mere
resolution, it would have simply adopted the language of the previous Local
Government Code. But Congress did not. In a clear divergence from the previous Local
Government Code, Section 19 of RA 7160 categorically requires that the local chief
executive act pursuant to an ordinance.

Page 22 of 166
B. Yes. The Court holds that the principle of res judicata, which finds application in
generally all cases and proceedings, cannot bar the right of the State or its agent to
expropriate private property. The very nature of eminent domain, as an inherent power
of the State, dictates that the right to exercise the power be absolute and unfettered
even by a prior judgment or res judicata. The scope of eminent domain is plenary and,
like police power, can "reach every form of property which the State might need for
public use." All separate interests of individuals in property are held by the government
under this tacit agreement or implied reservation. Notwithstanding the grant to
individuals, the eminent domain, the highest and most exact idea of property, remains
in the government, or in the aggregate body of the people in their sovereign capacity;
and they have the right to resume the possession of the property whenever the public
interest requires it." Thus, the State or its authorized agent cannot be forever barred
from exercising said right by reason alone of previous non-compliance with any legal
requirement.

Page 23 of 166
CASE NO. 12
BELUSO VS. THE MUNICIPALITY OF PANAY (CAPIZ)
G.R. No. 153974
August 7, 2006

Facts:
Petitioners Beluso are owners of parcels of land with a total area of about 20,424 square
meters. The Sangguniang Bayan of the Municipality of Panay issued Resolution No. 95-
29 authorizing the municipal government through the mayor to initiate expropriation
proceedings. A petition for expropriation was thereafter filed on April 14, 1997 by the
respondent Municipality of Panay before the Regional Trial Court (RTC), Branch 18 of
Roxas City. Petitioners filed a Motion to Dismiss alleging that the taking is not for public
use but only for the benefit of certain individuals; that it is politically motivated because
petitioners voted against the incumbent mayor and vice-mayor; and that some of the
supposed beneficiaries of the land sought to be expropriated have not actually signed a
petition asking for the property but their signatures were forged or they were misled into
signing the same. RTC denied petitioners’ Motion to Dismiss and declared that the
expropriation in this case is for "public use" and the respondent has the lawful right to
take the property upon payment of just compensation.

Petitioners then filed a Petition for Certiorari before the Court of Appeals (CA) claiming
that they were denied due process when the trial court declared that the taking was for
public purpose without receiving evidence on petitioners’ claim that the Mayor of Panay
was motivated by politics in expropriating their property and in denying their Motion to
Hold in Abeyance the Hearing of the Court Appointed Commissioners; and that the trial
court also committed grave abuse of discretion when it disregarded the affidavits of
persons denying that they signed a petition addressed to the municipal government of
Panay. Then CA dismissed the Petition for Certiorari. It held that the petitioners were not
denied due process as they were able to file an answer to the complaint and were able to
adduce their defenses therein; and that the purpose of the taking in this case constitutes
"public use". Petitioners filed for a motion for reconsideration but it was also denied.
Hence this petition.

Issue:
Whether or not the power of eminent domain is being exercised in accordance with the
delegating law under the existence of legislative grant in favor or local governments.

Ruling:
No. Eminent domain, which is the power of a sovereign state to appropriate private
property to particular uses to promote public welfare, is essentially lodged in the
legislature. While such power may be validly delegated to local government units (LGUs),

Page 24 of 166
other public entities and public utilities the exercise of such power by the delegated
entities is not absolute. In fact, the scope of delegated legislative power is narrower than
that of the delegating authority and such entities may exercise the power to expropriate
private property only when authorized by Congress and subject to its control and
restraints imposed through the law conferring the power or in other legislations

Despite the existence of legislative grant in favor of local governments, it is still the duty
of the courts to determine whether the power of eminent domain is being exercised in
accordance with the delegating law. Section 19 of RA 7160, which delegates to LGUs the
power of eminent domain lays down the parameters for its exercise. It provides as follows:
"Section 19. Eminent Domain. A local government unit may, through its chief executive
and acting pursuant to an ordinance, exercise the power of eminent domain for public
use, or purpose, or welfare for the benefit of the poor and the landless, upon payment of
just compensation, pursuant to the provisions of the Constitution and pertinent laws: . . .
It is clear therefore that several requisites must concur before an LGU can exercise the
power of eminent domain, to wit:

1. An ordinance is enacted by the local legislative council authorizing the local


chief executive, in behalf of the LGU, to exercise the power of eminent domain
or pursue expropriation proceedings over a particular private property.
2. The power of eminent domain is exercised for public use, purpose or welfare,
or for the benefit of the poor and the landless.
3. There is payment of just compensation, as required under Section 9, Article III
of the Constitution, and other pertinent laws.
4. A valid and definite offer has been previously made to the owner of the
property sought to be expropriated, but said offer was not accepted.

The Court in no uncertain terms have pronounced that a local government unit cannot
authorize an expropriation of private property through a mere resolution of its lawmaking
body. The Local Government Code expressly requires an ordinance for the purpose and a
resolution that merely expresses the sentiment of the municipal council will not suffice.

A resolution will not suffice for an LGU to be able to expropriate private property; and the
reason for this is settled:

. . . A municipal ordinance is different from a resolution. An ordinance is a


law, but a resolution is merely a declaration of the sentiment or opinion of a
lawmaking body on a specific matter. An ordinance possesses a general and
permanent character, but a resolution is temporary in nature. Additionally, the two
are enacted differently — a third reading is necessary for an ordinance, but not for

Page 25 of 166
a resolution, unless decided otherwise by a majority of all the Sanggunian
members.

If Congress intended to allow LGUs to exercise eminent domain through a


mere resolution, it would have simply adopted the language of the previous Local
Government Code. But Congress did not. In a clear divergence from the previous
Local Government Code, Sec. 19 of R.A. [No.] 7160 categorically requires that the
local chief executive act pursuant to an ordinance. . . .

As respondent's expropriation in this case was based merely on a resolution, such


expropriation is clearly defective. While the Court is aware of the constitutional policy
promoting local autonomy, the court cannot grant judicial sanction to an LGU's exercise
of its delegated power of eminent domain in contravention of the very law giving it such
power.

Page 26 of 166
CASE 13
FILSTREAM INTERNATIONAL INCORPORATED VS. COURT OF APPEALS
G.R. No. 125218.
January 23, 1998

Facts:
Filstream International Inc., is the registered owner of the properties subject of this dispute
consisting of adjacent parcels of land. Petitioner filed an ejectment suit against the
occupants of the abovementioned parcels of land private respondents in G.R. No. 128077)
on the grounds of termination of the lease contract and non-payment of
rentals. Judgment was rendered by the MTC ordering private respondents to vacate the
premises and pay back rentals to petitioner. Private respondents appealed the decision
to the Regional Trial Court of Manila, which in turn affirmed the decision of the MTC in its
decision. Thereafter, no further action was taken by the private respondents, as a result of
which the decision in the ejectment suit became final and executory.

However, it appeared that during the pendency of the ejectment proceedings private
respondents filed a complaint for Annulment of Deed of Exchange against petitioner
Filstream. It was at this stage that respondent City of Manila came into the picture when
the city government approved Ordinance No. 7813, authorizing Mayor Alfredo S. Lim to
initiate the acquisition by negotiation, expropriation, purchase, or other legal means
certain parcels of land registered under T.C.T. Nos. 169193, 169198, 169190, 169200,
169202, and 169192 of the Registry of Deeds of Manila which formed part of the
properties of petitioner then occupied by private respondents. Subsequently, the City of
Manila approved Ordinance No. 7855 declaring the expropriation of certain parcels of
land situated along Antonio Rivera and Fernando Ma. Guerero streets in Tondo, Manila
which were owned by Mr. Enrique Quijano Gutierez, petitioners predecessor-in-
interest. The said properties were to be sold and distributed to qualified tenants of the
area pursuant to the Land Use Development Program of the City of Manila. Respondent
City of Manila filed a complaint for eminent domain seeking to expropriate the a forecited
parcels of land owned by petitioner Filstream.

Filstream filed a motion to dismiss the complaint for eminent domain as well as a motion
to quash the writ of possession. The motion to dismiss was premised on the following
grounds: no valid cause of action; the petition does not satisfy the requirements of public
use and a mere clandestine maneuver to circumvent the writ execution issued by the RTC
of Manila, Branch 4 in the ejectment suit; violation of the constitutional guarantee against
non-impairment of obligation and contract; price offered was too low hence violative of
the just compensation provision of the constitution and the said amount is without the
certification of the City Treasurer for availability of funds.

Page 27 of 166
Issue:
Whether or not The City of Manila has the power to expropriate private property or the
so called has the power to exercise the power of eminent domain.

Ruling:
Yes, City of Manila has an undeniable right to exercise its power of eminent domain within
its jurisdiction. The right to expropriate private property for public use is expressly granted
to it under Section 19 of the 1991 Local Government Code. More specifically, the City of
Manila has the power to expropriate private property in the pursuit of its urban land
reform and housing program as explicitly laid out in the Revised Charter of the City of
Manila (R.A. No. 409). In fact, the City of Manilas right to exercise these prerogatives
notwithstanding the existence of a final and executory judgment over the property to be
expropriated has been upheld by this Court in the case of Philippine Columbian
Association vs. Panis, G.R. No. 106528, December 21, 1993. Relying on the
aforementioned provisions of the Revised Charter of the City of Manila, the Court declared
that: The City of Manila, acting through its legislative branch, has the express power to
acquire private lands in the city and subdivide these lands into home lots for sale to bona-
fide tenants or occupants thereof, and to laborers and low-salaried employees of the city.

Page 28 of 166
CASE NO. 14
LAGCAO VS. JUDGE LABRA
G.R. No. 155746
October 13, 2004

Facts:
The Province of Cebu donated 210 lots to the City of Cebu. One of these lots was Lot
1029, with an area of 4,048 square meters. Petitioners purchased Lot 1029 on installment
basis. But then, in late 1965, the 210 lots, including Lot 1029, reverted to the Province of
Cebu. Consequently, the province tried to annul the sale of Lot 1029 by the City of Cebu
to the petitioners. This prompted the latter to sue the province for specific performance
and damages in the then Court of First Instance. The court a quo ruled in favor of
petitioners and ordered the Province of Cebu to execute the final deed of sale in favor of
petitioners. The Court of Appeals affirmed the decision of the trial court. Pursuant to the
ruling of the appellate court, the Province of Cebu executed a deed of absolute sale over
Lot 1029 in favor of petitioners. Thereafter, Transfer Certificate of Title (TCT) No. 129306
was issued in the name of petitioners and Crispina Lagcao.

After acquiring title, petitioners tried to take possession of the lot only to discover that it
was already occupied by squatters. Thus, petitioners instituted ejectment proceedings
against the squatters. The Municipal Trial Court in Cities (MTCC), rendered a decision
ordering the squatters to vacate the lot. On appeal, the RTC affirmed the MTCCs decision
and issued a writ of execution and order of demolition.

When the demolition order was about to be implemented, Cebu City Mayor Alvin Garcia
wrote two letters to the MTCC, requesting the deferment of the demolition on the ground
that the City was still looking for a relocation site for the squatters. Acting on the mayors
request, the MTCC issued two orders suspending the demolition for a period of 120 days.
Unfortunately for petitioners, during the suspension period, the Sangguniang
Panlungsod (SP) of Cebu City passed a resolution which identified Lot 1029 as a socialized
housing site pursuant to RA 7279. The SP of Cebu City passed Ordinance No. 1772 which
included Lot 1029 among the identified sites for socialized housing. Ordinance No.
1843 was enacted by the SP of Cebu City authorizing the mayor of Cebu City to initiate
expropriation proceedings for the acquisition of Lot 1029 which was registered in the
name of petitioners. The intended acquisition was to be used for the benefit of the
homeless after its subdivision and sale to the actual occupants thereof. For this purpose,
the ordinance appropriated the amount of P6,881,600 for the payment of the subject lot.
This ordinance was approved by Mayor Garcia.

Petitioners filed with the RTC an action for declaration of nullity of Ordinance No. 1843
for being unconstitutional. The trial court rendered its decision dismissing the complaint

Page 29 of 166
filed by petitioners whose subsequent motion for reconsideration was likewise denied. In
this appeal, petitioners argue that Ordinance No. 1843 is unconstitutional as it sanctions
the expropriation of their property for the purpose of selling it to the squatters, an
endeavor contrary to the concept of public use contemplated in the Constitution. They
allege that it will benefit only a handful of people. The ordinance, according to petitioners,
was obviously passed for politicking, the squatters undeniably being a big source of votes.

Issue:
Whether or not the expropriation of City of Cebu of a 4,048-square-meter parcel of land
owned by petitioners contravenes the Constitution and applicable laws.

Ruling:
No, Local government units have no inherent power of eminent domain and can exercise
it only when expressly authorized by the legislature. By virtue of RA 7160, Congress
conferred upon local government units the power to expropriate. However, while we
recognize that housing is one of the most serious social problems of the country, local
government units do not possess unbridled authority to exercise their power of eminent
domain in seeking solutions to this problem.

There are two legal provisions which limit the exercise of this power: (1) no person shall
be deprived of life, liberty, or property without due process of law, nor shall any person
be denied the equal protection of the laws; and (2) private property shall not be taken for
public use without just compensation. Thus, the exercise by local government units of the
power of eminent domain is not absolute. In fact, Section 19 of RA 7160 itself explicitly
states that such exercise must comply with the provisions of the Constitution and
pertinent laws. The exercise of the power of eminent domain drastically affects a
landowners right to private property, which is as much a constitutionally-protected right
necessary for the preservation and enhancement of personal dignity and intimately
connected with the rights to life and liberty. Whether directly exercised by the State or by
its authorized agents, the exercise of eminent domain is necessarily in derogation of
private rights. For this reason, the need for a painstaking scrutiny cannot be
overemphasized.
The foundation of the right to exercise eminent domain is genuine necessity and that
necessity must be of public character. Government may not capriciously or arbitrarily
choose which private property should be expropriated. In this case, there was no showing
at all why petitioners property was singled out for expropriation by the city ordinance or
what necessity impelled the particular choice or selection. Ordinance No. 1843 stated no
reason for the choice of petitioners property as the site of a socialized housing project.

Page 30 of 166
CASE NO. 15
ANTONIO vs HON. ISAGANI A. GERONIMO
G.R. No. 124779
November 29, 2005

Facts:
A complaint for unlawful detainer was filed by Alexander Catolos (private respondent),
who alleged that he was the registered owner of four (4) parcels of land. The defendants
therein were the petitioners, who were occupying the said properties. Private respondent
claimed he allowed petitioners to occupy portions of his land without requiring them to
pay rent, on the condition that the latter would immediately vacate the same in the event
that the former would need the premises. However, when private respondent did notify
petitioners of his need to use the premises, petitioners refused to vacate the land even
after demand.

The complaint was resolved in favor of private respondent. respondent judge ordered
petitioners to vacate the subject properties and pay the amount of Two Hundred Pesos
(P200.00) as reasonable compensation for the use and occupation of the properties, as
well as Twenty Thousand Pesos (P20,000.00) for litigation expenses and attorneys fees.
Private respondent filed a motion for issuance of a writ of demolition. The lower court
granted the motion and directed the issuance of a writ of demolition. Partial demolition
had already taken place Private respondent filed an urgent ex parte motion, seeking the
full implementation of the writ of demolition, thus was granted.

Sangguniang Bayan of Antipolo, Rizal passed Resolution No. 61-95, authorizing Mayor
Daniel Garcia to acquire thru expropriation or purchase the subject properties for public
purposes/socialized housing. Another resolution, No. 88-95, was issued amending the
previous resolution by further authorizing the municipal mayor to secure financing for the
acquisition of the said parcel of land subject of this case. Sangguniang Bayan informed
respondent court of the expropriation and the fact that the funds required for the same
have already been included in the 1996 budget, and requested that the demolition be
held in abeyance. At this point, the writ of demolition had not yet been fully implemented.

Petitioners filed a motion to stay execution with the MTC on the ground that supervening
events have rendered execution unjust and inequitable. A Supplement to the Motion to
Stay Execution was filed. Petitioners invoked Commonwealth Act No. 538 in asking
respondent judge to suspend the action for ejectment in view of the announced
expropriation of the subject properties. Petitioners argue that Commonwealth Act No.
538 clearly provides that for the purposes of the Act, the action shall be considered
instituted from the time the competent authority advises in writing the owner of the
intention of the government to acquire his land.

Page 31 of 166
In its Comment, private respondent posits that Commonwealth Act No. 538 applies only
to cases wherein expropriation proceedings are filed. The Sangguniang Bayans
resolutions expressing the intent to expropriate the properties evinced merely an
intention to expropriate and not the actual expropriation proceeding. According to
private respondent, assuming that there exists an expropriation proceeding, petitioners
still cannot avail of the automatic suspension of the ejectment case because they failed
to pay their current rentals and deposit them with the court. Regarding the issue of non-
payment of rentals, petitioners point out that facts of record show that they were not
required by contract to pay rents, thereby rendering the requirement cited by private
respondent inapplicable.

Issue:
Whether or not a resolution for expropriation by a local government unit can suspend the
writ of execution and demolition in an ejectment case.

Ruling:
No. The power of eminent domain, inherent in the State, employs private property for
public use upon payment of just compensation. The enactment of the principle of social
justice falls within the parameters of public use. Local government units may exercise the
power of eminent domain, subject to the limitation embodied under the law. There are
two relevant laws to consider, the Local Government Code (LGC) and Republic Act No.
7279 (UDHA). Clearly, the Sangguniang Bayan, being a local legislative body, may exercise
the power to expropriate private properties, subject to the following requisites, all of
which must concur:

1. An ordinance is enacted by the local legislative council authorizing the local chief
executive, in behalf of the local government unit, to exercise the power of eminent
domain or pursue expropriation proceedings over a particular private property.
2. The power of eminent domain is exercised for public use, purpose or welfare, or
for the benefit of the poor and the landless.
3. There is payment of just compensation, as required under Section 9, Article III of
the Constitution, and other pertinent laws.
4. A valid and definite offer has been previously made to the owner of the property
sought to be expropriated, but said offer was not accepted.

In the instant case, no ordinance was passed by the Sangguniang Bayan of Antipolo. In
its stead were Resolution Nos. 61-95 and 88-95.

These resolutions cannot partake of a supervening event so as to suspend the writ of


execution in the ejectment proceedings. They merely express at most an intention to

Page 32 of 166
expropriate. Private respondent correctly maintained that there was no positive act of
instituting the intended expropriation proceedings. There is no dispute that a local
government unit possesses the power of eminent domain. But the taking of private
properties is not absolute. The power of eminent domain must not be exercised arbitrarily,
even if purposed for resolving a critical problem such as urban squatting. The safeguards
afforded by law require strict observance.

The UDHA is also relevant as it governs the local expropriation of property for purposes
of urban land reform and housing. Sections 9 and 10, in particular, provide the relevant
limitations, thus:

Sec. 9. Priorities in the Acquisition of Land. Lands for socialized housing


shall be acquired in the following order:

(a) Those owned by the government or any of its subdivisions,


instrumentalities, or agencies, including government-owned or -controlled
corporations and their subsidiaries;
(b) Alienable lands of the public domain;
(c) Unregistered or abandoned and idle lands;
(d) Those within the declared Areas for Priority Development, Zonal
Improvement Program sites, and Slum Improvement and Resettlement
Program sites which have not yet been acquired;
(e) Bagong Lipunan Improvement of Sites and Services or BLISS sites which
have not yet been acquired; and
(f) Privately-owned lands.

Where open-site development is found more practicable and


advantageous to the beneficiaries, the priorities mentioned in this section
shall not apply. The local government units shall give budgetary priority to
on-site development of government lands.

Assuming that there was even an attempt by Antipolo City to expropriate the subject
properties for socialized housing, the records do not show compliance with the
abovementioned rules. No attempt was made to acquire the first five (5) lands mentioned
in Section 9. Neither were the other modes of acquisition exhausted, as mandated by
Section 10. An examination of the resolutions readily shows that the purpose for which
they were passed is to save petitioners from the impending demolition. Yet nothing
therein is of binding force as to preclude the ejectment of the petitioners or the
demolition of their houses.

Page 33 of 166
CASE NO. 16
ENRIQUEZ VS. AUDITOR GENERAL OF THE PHILIPPINES
No. L-12817.
April 29, 1960

Facts:
The Municipal Mayor, requested the Provincial Fiscal, through the Provincial Board, to
prosecute a case in court to question and test judicially the legality of Republic Act No.
1383 and to prevent the National Waterworks and Sewerage Authority from exercising its
authority over the waterworks system of the municipality. The provincial fiscal rendered
an opinion holding that Republic Act No. 1383 is valid and constitutional and declined to
represent the municipality of Bauan in an action to be brought against the NWSA.

Now, the Municipal Mayor, authorized by the council, engaged in the services of the
petitioner to represent it in its contemplated action against the NWSA.
 Petitioner claims, through the Municipal Treasurer, reimbursement of his advances
for the payment of docket fees and initial attorney’s fees. And that the Municipal
Council could not request to the Secretary of Justice to appoint an acting provincial
fiscal because the Secretary of Justice, who has executive supervision over the
Government Corporate Counsel, who represented the NWSA in the case filed
against it by the municipality of and direct supervision and control over the
Provincial Fiscal, would be placed in an awkward and absurd position of having
control of both sides of the controversy.
 Respondent disallowed in audit the petitioner's claim for initial attorney's fees,
based upon an opinion rendered by the Secretary of Justice who held that the
Provincial Fiscal was not disqualified to handle and prosecute in court the case of
the municipality of Bauan and that its municipal council had no authority to engage
the services of a special counsel, but offered no objection to the refund to the
petitioner of the sum paid by him to the Court as docket fees.

Issues:
A. Whether or not the Provincial Fiscal is disqualified to represent the municipality.
B. Whether or not the contention of petitioner is tenable.

Ruling:
A. NO. Under the provision of Sections 2241, 1682 and 1683 of the Revised
Administrative Code, the provincial fiscal is the legal adviser of the mayor and council
of the various municipalities of a province and it is his duty to represent the
municipality in any court EXCEPT when he is disqualified by law. The provincial fiscal
is disqualified to represent in court the municipality: 1. if and when original jurisdiction
of the case involving the municipality is vested in the Supreme Court; 2. when the

Page 34 of 166
municipality is a party adverse to the provincial government or to some other
municipality in the same province; and 3. when in the case involving the municipality,
he, or his wife, or child, is pecuniarily involved as heir, legatee, creditor or otherwise.

In the present case, the fact that the provincial fiscal entertains a hostile belief and
attitude on the theory involved in the litigation and, therefore, would not be in a
position to prosecute the case of the municipality with earnestness and vigor, could
not justify the act of the municipal council in engaging the services of a special counsel.
Bias or prejudice and animosity or hostility on the part of a fiscal not based on any of
the conditions enumerated in the law and the Rules of Court do not constitute a legal
and valid excuse for inhibition or disqualification.

B. NO. Section 83 of the Revised Administrative Code, as amended by Executive Order


No. 94, series and further amended by Executive Order No. 392, provides that the
Secretary of Justice shall have executive supervision over the Government Corporate
Counsel and supervision and control over Provincial Fiscals. In administrative law,
supervision means overseeing or the power or authority of an officer to see that
subordinate officers perform their duties. If the latter fail or neglect to fulfill them the
former may take such action or step as prescribed by law to make them perform their
duties. Control, on the other hand, means the power of an officer to alter or modify or
nullify or set aside what a subordinate officer had done in the performance of his
duties and to substitute the judgment of the former for that of the latter.

Instead of engaging the services of a special attorney, the municipal council should
have requested the Secretary of Justice to appoint an acting provincial fiscal in place
of the provincial fiscal who had declined to handle and prosecute its case in court
pursuant to Section 1679 of the Revised Administrative Code.

Page 35 of 166
CASE NO. 17
MUNICIPALITY OF SAN NARCISO VS HON. MENDEZ
G.R. No. 103702
December 6, 1994

Facts:
On 20 August 1959, President Carlos P. Garcia, issued, pursuant to the then Sections 68
and 2630 of the Revised Administrative Code, as amended, Executive Order No. 353
creating the municipal district of San Andres, Quezon, by segregating from the
municipality of San Narciso of the same province, the barrios of San Andres, Mangero,
Alibijaban, Pansoy, Camflora and Tala along with their respective sitios.
EO No. 353 was issued upon the request, addressed to the President and coursed through
the Provincial Board of Quezon, of the municipal council of San Narciso, Quezon.

By virtue of EO No. 174, dated 05 October 1965, issued by President Diosdado Macapagal,
the municipal district of San Andres was later officially recognized to have gained the
status of a fifth class municipality beginning 01 July 1963 by operation of Section 2 of
Republic Act No. 1515. 2 The executive order added that “the conversion of this municipal
district into a municipality as proposed in House Bill No. 4864 was approved by the House
of Representatives.”

Petitioner Municipality of San Narciso: filed a petition for quo warranto with RTC which
petition sought the declaration of nullity of EO No. 353 Invoking the ruling of this Court
in Pelaez v. Auditor General.

Respondent San Andres: San Narciso is estopped from questioning the creation of the
new municipality and that the case had become moot and academic with the enactment
of Republic Act No. 7160 (Sec. 442. Requisites for Creation. — . . .(d) Municipalities existing
as of the date of the effectivity of this Code shall continue to exist and operate as such.)

Petitioner: The above provision of law was inapplicable to the Municipality of San Andres
since the enactment referred to legally existing municipalities and not to those whose
mode of creation had been void ab initio.

Issue:
Whether or not the Municipality of San Andres is a de jure or de facto municipal
corporation.

Ruling:
Executive Order No. 353 creating the municipal district of San Andres was issued on 20
August 1959 but it was only after almost thirty (30) years, or on 05 June 1989, that the

Page 36 of 166
municipality of San Narciso finally decided to challenge the legality of the executive order.

Granting the Executive Order No. 353 was a complete nullity for being the result of an
unconstitutional delegation of legislative power, the peculiar circumstances obtaining in
this case hardly could offer a choice other than to consider the Municipality of San Andres
to have at least attained a status uniquely of its own closely approximating, if not in fact
attaining, that of a de facto municipal corporation. Conventional wisdom cannot allow it
to be otherwise. Created in 1959 by virtue of Executive Order No. 353, the Municipality of
San Andres had been in existence for more than six years when, on 24 December 1965,
Pelaez v. Auditor General was promulgated. The ruling could have sounded the call for a
similar declaration of the unconstitutionality of Executive Order No. 353 but it was not to
be the case. On the contrary, certain governmental acts all pointed to the State’s
recognition of the continued existence of the Municipality of San Andres. Thus, after more
than five years as a municipal district, Executive Order No. 174 classified the Municipality
of San Andres as a fifth-class municipality after having surpassed the income requirement
laid out in Republic Act No. 1515.

At the present time, all doubts on the de jure standing of the municipality must be
dispelled. Under the Ordinance (adopted on 15 October 1986) apportioning the seats of
the House of Representatives, appended to the 1987 Constitution, the Municipality of San
Andres has been considered to be one of the twelve (12) municipalities composing the
Third District of the province of Quezon. Equally significant is Section 442(d) of the Local
Government Code to the effect that municipal districts “organized pursuant to
presidential issuances or executive orders and which have their respective sets of elective
municipal officials holding office at the time of the effectivity of the Code shall henceforth
be considered as regular municipalities.” All considered, the de jure status of the
Municipality of San Andres in the province of Quezon must now be conceded.

Page 37 of 166
CASE NO. 18
MUNICIPALITY OF JIMENEZ VS BAZ
GR No. 105746
December 2, 1996

Facts:
The Municipality of Sinacaban was created by EO 258 of then Pres. Quirino pursuant to
Sec. 68 of the Revised Admin. Code.Sinacaban laid claim to several barrios based on the
technical description in EO 258. The Municipality of Jimenez asserted jurisdiction based
on an agreement with Sinacaban which was approved by the Provincial Board of Misamis
Occidental which fixed the common boundary of Sinacaban and Jimenez. The Provincial
Board declared the disputed area to be part of Sinacaban. It held that the earlier resolution
approving the agreement between the municipalities was void since the Board had no
power to alter the boundaries of Sinacaban as fixed in EO 258. Jimenez argued that the
power to create municipalities is essentially legislative (as held in Pelaez v Auditor
General), then Sinacaban, which was created thru and EO, had no legal personality and no
right to assert a territorial claim.

Issue:
Whether or not Sinacaban has juridical personality.

Ruling:
Where a municipality created as such by EO is later impliedly recognized and its acts are
accorded legal validity, its creation can no longer be questioned. In the case of
Municipality of San Narciso v Mendez, the SC laid the factors to consider in validating the
creation of a municipal corporation: 1. The fact that for 30 years, the validity of the
corporation has not been challenged; 2. The fact that no quo warranto suit was filed to
question the validity of the EO creating the municipality; and 3. The fact that the
municipality was later classified as a 5th class municipality, organized as part of a
municipal circuit court and considered part of a legislative district in the Constitution
apportioning the seats in the House. In this case, the following factors are present:

1. Sinacaban has been in existence for 16 years when Pelaez was decided in 1965 and yet
the validity of EO 258 creating it had never been questioned. 2. It was only 40 years later
that its existence was questioned. 3. Rule 66, Sec. 16 of the Rules of COurt provides that
a quo warranto suit against a corporation for forfeiture of its charter must be commenced
within 5 years from the time the act complaned of was done or committed. 4. The State
and even Jimenez recognized Sinacaban’s corporate existence by entering into an
agreement with it regarding the boundary. Ex.: AO 33, Judiciary Reorganization Act of
1980, etc. 5. Sinacaban is constituted as part of a municipal circuit for purposes of the
establishment of MTCs in the country. Moreover, the LGC of 1991, Sec. 442(d) provides

Page 38 of 166
that “municipal districts organized pursuant to presidential issuances or executive orders
and which have their respective sets of elective officials holding office at the time of the
effectivity of this Code shall henceforth be considered as regular municipalities.”
Sinacaban has attained de jure status by virtue of the Ordinance appended to the 1987
Constitution, apportioning legislative districts throughout the country, which considered
Sinacaban as part of the 2nd District of Misamis Occidental.

II. Sinacaban had attained de facto status at the time the 1987 Constitution took effect. It
is not subject to the plebiscite requirement. It applies only to new municipalities created
for the first time under the Constitution. The requirement of plebiscite was originally
contained in Art. XI, Section 3 of the previous Constitution. It cannot be applied to
municipal corporations created before, such as Sinacaban.

Page 39 of 166
CASE NO. 19
SAN JUAN VS. CIVIL SERVICE COMMISSION
G.R. No. 92299
April 19, 1991

Facts:
On March 22, 1988 the position of Provincial Budget Officer for the province of Rizal was
left vacant by a certain Hemadima Del Rosario. In a letter dated April 18, 1988. Gov.
Reynaldo R. San Juan informed Director Reynaldo Abella that Ms. Dalisay Santos assumed
office as Acting PBO pursuant to memorandum issued by San Juan. In a memorandum
dated July 26, 1988 addressed to the DBM Secretary then Director Abella recommended
the appointment of Cecila Almajose as Provincial Budget Officer of Rizal. According to
Abella Almajose was the most qualified since she was the only CPA among the contenders.
On August 1, 1988 DBM Undersecretary Cabuquit signed the appointment papers of the
private respondent as PBO of Rizal upon the recommendation of Abella. On Aug 31, 1988.
DBM Regional Director Agripino G. Galvez wrote San Juan that Dalisay Santod did not
meet the minimum requirements under Local Budget Circular No. 31 for the position of a
Local Budget Officer. San Juan after having been informed of the private respondent's
appointment wrote Sec. Carague protesting against the said appointment on the grounds
that Cabuquit is not legally authorize to appoint the PBO.

Issue:
Whether or not Almajose is lawfully entitled to discharge the functions of PBO of Rizal

Ruling:
When the Civil Service Commission interpreted the recommending power of the
Provincial Governor as purely directory, it went against the letter and spirit of the
constitutional provisions on local autonomy. If the DBM Secretary jealously hoards the
entirety of budgetary powers and ignores the right of local governments to develop self-
reliance and resoluteness in the handling of their own funds, the goal of meaningful local
autonomy is frustrated and set back.The right given by Local Budget Circular No. 31 which
states: Sec. 6.0 — The DBM reserves the right to fill up any existing vacancy where none
of the nominees of the local chief executive meet the prescribed requirements. is ultra
vires and is, accordingly, set aside. The DBM may appoint only from the list of qualified
recommendees nominated by the Governor. If none is qualified, he must return the list of
nominees to the Governor explaining why no one meets the legal requirements and ask
for new recommendees who have the necessary eligibilities and qualifications. The PBO
is expected to synchronize his work with DBM. More important, however, is the proper
administration of fiscal affairs at the local level. Provincial and municipal budgets are
prepared at the local level and after completion are forwarded to the national officials for
review. They are prepared by the local officials who must work within the constraints of

Page 40 of 166
those budgets. They are not formulated in the inner sanctums of an all-knowing DBM and
unilaterally imposed on local governments whether or not they are relevant to local needs
and resources. It is for this reason that there should be a genuine interplay, a balancing
of viewpoints, and a harmonization of proposals from both the local and national officials.
It is for this reason that the nomination and appointment process involve a sharing of
power between the two levels of government.

Page 41 of 166
CASE NO. 20
MACASIANO VS. DIOKNO
G.R. No. 97764
August 10, 1992

Facts:
On June 13, 1990 respondent municipally passed Ordinance No. 86 Series of 1990 which
authorized the closure of J. Gabriel, GG Cruz, Bayanihan, L.T Garcia Extension and Opera
st located at Baclaran Paranaque and the establishment of a flea market therein. On July
20, 1990, the Metropolitan Manila Authority approved Ordinance No. 86 1990 of the
municipal counself of respondent the municipal council of Paranaque also issued a
resolution authorizing Paranaque Mayor Wilfredo N. Ferrer to enter into contract with any
service cooperative for the establishment, operation, maintenance and management of
flea markets and vending areas. It entered into a service cooperative with respondent
Palanyag whereby the latter shall operate, maintain and manage the flea market with the
obligation to remit dues to the treasury of the municipal government of Paranaque.
Consequently, market stalls were put up by respondent Palanyag on the said streets.

On September 13, 1990, Petitioner Brig. Gen Macasiano ordered the destruction and
confiscation of stalls along G.G Cruz and J. Gabriel in Baclaran. These stalls were later
returned to respondent Palanyag. Petitioner Macasiano wrote a letter to respondent
Palanyag giving the latter 10 days.
Issue:
Whether or not an ordinance or resolution issued by the municipal council of Paranaque
authorizing the lease and use of public streets or thoroughfares as sites for flea markets
is valid.

Ruling:
We find the petition meritorious. In resolving the question of whether the disputed
municipal ordinance authorizing the flea market on the public streets is valid, it is
necessary to examine the laws in force during the time the said ordinance was enacted,
namely, Batas Pambansa Blg. 337, otherwise known as Local Government Code, in
connection with established principles embodied in the Civil Code an property and settled
jurisprudence on the matter.
The property of provinces, cities and municipalities is divided into property for public use
and patrimonial property (Art. 423, Civil Code). As to what consists of property for public
use, Article 424 of Civil Code states:

Art. 424. Property for public use, in the provinces, cities and municipalities, consists
of the provincial roads, city streets, the squares, fountains, public waters,

Page 42 of 166
promenades, and public works for public service paid for by said provinces, cities
or municipalities.

All other property possessed by any of them is patrimonial and shall be governed
by this Code, without prejudice to the provisions of special laws.

Based on the foregoing, J. Gabriel G.G. Cruz, Bayanihan, Lt. Garcia Extension and Opena
streets are local roads used for public service and are therefore considered public
properties of respondent municipality. Properties of the local government which are
devoted to public service are deemed public and are under the absolute control of
Congress (Province of Zamboanga del Norte v. City of Zamboanga, L-24440, March 28,
1968, 22 SCRA 1334). Hence, local governments have no authority whatsoever to control
or regulate the use of public properties unless specific authority is vested upon them by
Congress. One such example of this authority given by Congress to the local governments
is the power to close roads as provided in Section 10, Chapter II of the Local Government
Code, which states:

Sec. 10. Closure of roads. — A local government unit may likewise, through its head
acting pursuant to a resolution of its sangguniang and in accordance with existing
law and the provisions of this Code, close any barangay, municipal, city or
provincial road, street, alley, park or square. No such way or place or any part of
thereof shall be close without indemnifying any person prejudiced thereby. A
property thus withdrawn from public use may be used or conveyed for any purpose
for which other real property belonging to the local unit concerned might be
lawfully used or conveyed. (Emphasis ours).

However, the aforestated legal provision which gives authority to local government units
to close roads and other similar public places should be read and interpreted in
accordance with basic principles already established by law. These basic principles have
the effect of limiting such authority of the province, city or municipality to close a public
street or thoroughfare. Article 424 of the Civil Code lays down the basic principle that
properties of public dominion devoted to public use and made available to the public in
general are outside the commerce of man and cannot be disposed of or leased by the
local government unit to private persons. Aside from the requirement of due process
which should be complied with before closing a road, street or park, the closure should
be for the sole purpose of withdrawing the road or other public property from public use
when circumstances show that such property is no longer intended or necessary for public
use or public service. When it is already withdrawn from public use, the property then
becomes patrimonial property of the local government unit concerned (Article 422, Civil
Code; Cebu Oxygen, etc. et al. v. Bercilles, et al., G.R. No. L-40474, August 29, 1975, 66
SCRA 481). It is only then that the respondent municipality can "use or convey them for

Page 43 of 166
any purpose for which other real property belonging to the local unit concerned might
be lawfully used or conveyed" in accordance with the last sentence of Section 10, Chapter
II of Blg. 337, known as Local Government Code. In one case, the City Council of Cebu,
through a resolution, declared the terminal road of M. Borces Street, Mabolo, Cebu City
as an abandoned road, the same not being included in the City Development Plan.
Thereafter, the City Council passes another resolution authorizing the sale of the said
abandoned road through public bidding. We held therein that the City of Cebu is
empowered to close a city street and to vacate or withdraw the same from public use.
Such withdrawn portion becomes patrimonial property which can be the object of an
ordinary contract (Cebu Oxygen and Acetylene Co., Inc. v. Bercilles, et al., G.R. No.
L-40474, August 29, 1975, 66 SCRA 481). However, those roads and streets which are
available to the public in general and ordinarily used for vehicular traffic are still
considered public property devoted to public use. In such case, the local government has
no power to use it for another purpose or to dispose of or lease it to private persons. This
limitation on the authority of the local government over public properties has been
discussed and settled by this Court en banc in "Francisco V. Dacanay, petitioner v. Mayor
Macaria Asistio, Jr., et al., respondents, G.R. No. 93654, May 6, 1992." This Court ruled:

There is no doubt that the disputed areas from which the private respondents'
market stalls are sought to be evicted are public streets, as found by the trial court
in Civil Case No. C-12921. A public street is property for public use hence outside
the commerce of man (Arts. 420, 424, Civil Code). Being outside the commerce of
man, it may not be the subject of lease or others contract (Villanueva, et al. v.
Castañeda and Macalino, 15 SCRA 142 citing the Municipality of Cavite v. Rojas, 30
SCRA 602; Espiritu v. Municipal Council of Pozorrubio, 102 Phil. 869; And Muyot v.
De la Fuente, 48 O.G. 4860).
As the stallholders pay fees to the City Government for the right to occupy portions
of the public street, the City Government, contrary to law, has been leasing portions
of the streets to them. Such leases or licenses are null and void for being contrary
to law. The right of the public to use the city streets may not be bargained away
through contract. The interests of a few should not prevail over the good of the
greater number in the community whose health, peace, safety, good order and
general welfare, the respondent city officials are under legal obligation to protect.
The Executive Order issued by acting Mayor Robles authorizing the use of Heroes
del '96 Street as a vending area for stallholders who were granted licenses by the
city government contravenes the general law that reserves city streets and roads
for public use. Mayor Robles' Executive Order may not infringe upon the vested
right of the public to use city streets for the purpose they were intended to
serve: i.e., as arteries of travel for vehicles and pedestrians.

Page 44 of 166
Even assuming, in gratia argumenti, that respondent municipality has the authority to
pass the disputed ordinance, the same cannot be validly implemented because it cannot
be considered approved by the Metropolitan Manila Authority due to non-compliance by
respondent municipality of the conditions imposed by the former for the approval of the
ordinance, to wit:

1. That the aforenamed streets are not used for vehicular traffic, and that the
majority of the residents do(es) not oppose the establishment of the flea
market/vending areas thereon;
2. That the 2-meter middle road to be used as flea market/vending area shall be
marked distinctly, and that the 2 meters on both sides of the road shall be used by
pedestrians;
3. That the time during which the vending area is to be used shall be clearly
designated;
4. That the use of the vending areas shall be temporary and shall be closed once
the reclaimed areas are developed and donated by the Public Estate Authority. (p.
38, Rollo)

Respondent municipality has not shown any iota of proof that it has complied with the
foregoing conditions precedent to the approval of the ordinance. The allegations of
respondent municipality that the closed streets were not used for vehicular traffic and
that the majority of the residents do not oppose the establishment of a flea market on
said streets are unsupported by any evidence that will show that this first condition has
been met. Likewise, the designation by respondents of a time schedule during which the
flea market shall operate is absent.

Further, it is of public notice that the streets along Baclaran area are congested with
people, houses and traffic brought about by the proliferation of vendors occupying the
streets. To license and allow the establishment of a flea market along J. Gabriel, G.G. Cruz,
Bayanihan, Lt. Garcia Extension and Opena streets in Baclaran would not help in solving
the problem of congestion. We take note of the other observations of the Solicitor General
when he said:

. . . There have been many instances of emergencies and fires where


ambulances and fire engines, instead of using the roads for a more direct
access to the fire area, have to maneuver and look for other streets which
are not occupied by stalls and vendors thereby losing valuable time which
could, otherwise, have been spent in saving properties and lives.
Along G.G. Cruz Street is a hospital, the St. Rita Hospital. However, its
ambulances and the people rushing their patients to the hospital cannot
pass through G.G. Cruz because of the stalls and the vendors. One can only

Page 45 of 166
imagine the tragedy of losing a life just because of a few seconds delay
brought about by the inaccessibility of the streets leading to the hospital.
The children, too, suffer. In view of the occupancy of the roads by stalls and
vendors, normal transportation flow is disrupted and school children have
to get off at a distance still far from their schools and walk, rain or shine.
Indeed one can only imagine the garbage and litter left by vendors on the
streets at the end of the day. Needless to say, these cause further pollution,
sickness and deterioration of health of the residents therein. (pp. 21-
22, Rollo)

Respondents do not refute the truth of the foregoing findings and observations of
petitioners. Instead, respondents want this Court to focus its attention solely on the
argument that the use of public spaces for the establishment of a flea market is well within
the powers granted by law to a local government which should not be interfered with by
the courts.
Verily, the powers of a local government unit are not absolute. They are subject to
limitations laid down by toe Constitution and the laws such as our Civil Code. Moreover,
the exercise of such powers should be subservient to paramount considerations of health
and well-being of the members of the community. Every local government unit has the
sworn obligation to enact measures that will enhance the public health, safety and
convenience, maintain peace and order, and promote the general prosperity of the
inhabitants of the local units. Based on this objective, the local government should refrain
from acting towards that which might prejudice or adversely affect the general welfare.
As what we have said in the Dacanay case, the general public have a legal right to demand
the demolition of the illegally constructed stalls in public roads and streets and the
officials of respondent municipality have the corresponding duty arising from public office
to clear the city streets and restore them to their specific public purpose.

The instant case as well as the Dacanay case, involves an ordinance which is void and
illegal for lack of basis and authority in laws applicable during its time. However, at this
point, We find it worthy to note that Batas Pambansa Blg. 337, known as Local
Government Lode, has already been repealed by Republic Act No. 7160 known as Local
Government Code of 1991 which took effect on January 1, 1992. Section 5(d) of the new
Code provides that rights and obligations existing on the date of effectivity of the new
Code and arising out of contracts or any other source of prestation involving a local
government unit shall be governed by the original terms and conditions of the said
contracts or the law in force at the time such rights were vested.

Page 46 of 166
CASE NO. 21
LEYNES vs. COA
G.R. No. 143596
December 11, 2003

Facts:

Judge Tomas Leynes is the presiding judge of the RTC of Calapan City Oriental Mindoro
Branch 40 was formerly assigned to the Municipality of Naujan, Oriental Mindoro. As such
his RATA were drawn from the budget of the Supremre Court, in addition Judge Leynes
received a monthly allowance of 944 from the local funds of Naujan starting 1984. On
March 15, 1993 the Sangguniang Bayan of Naujan sought the opinion of the Provincial
Auditor and the Provincial Budget Officer regarding any budgetary limitation on the grant
of a monthly allowance by the petitioner judge increasing petitioner judge monthly
allowance from 944 to 1600. On Feb 17, 1994. Provincial Auditor Salvacion M. Dalisay sent
a letter to the Mayor directing them to stop the payment of the 1,600 monthly allowance
or RATA to judge and to require the immediate refund of the amounts previously paid to
the latter. Petitioner judge appealled to COA Regional Director Gregoria S. Ong who
upheld the opinion of Provincial Auditor Dalisay.

Issue:

Whether or not the municipality of Naujan Oriental Mindoro can validly provide RATA to
its Municipal Judge.

Ruling:
Yes. In this case, RA 7160 (the LGC of 1991) is a special law which exclusively deals with
local government units (LGUs), outlining their powers and functions in consonance with
the constitutionally mandated policy of local autonomy. RA 7645 (the GAA of 1993), on
the other hand, was a general law which outlined the share in the national fund of all
branches of the national government. RA 7645 therefore, being a general law, could not
have, by mere implication, repealed RA 7160. Rather, RA 7160 should be taken as the
exception to RA 7645 in the absence of circumstances warranting a contrary conclusion.

The controversy actually centers on the seemingly sweeping provision in NCC No. 67
which states that no one shall be allowed to collect RATA from more than one source.
Does this mean that judges cannot receive allowances from LGUs in addition to the RATA
from the Supreme Court For reasons that will hereinafter be discussed, we answer in the
negative.

Page 47 of 166
The pertinent provisions of NCC No. 67 read:

3.1.1 Payment of RATA, whether commutable or reimbursable, shall be in


accordance with the rates prescribed for each of the following officials and
employees and those of equivalent ranks, and the conditions enumerated
under the pertinent sections of the General Provisions of the annual General
Appropriations Act (GAA)

Sangguniang Panlalawigan ofOriental Mindoro approved Resolution No. 101 of


the Sangguniang Bayan of Naujan granting theP1,600 monthly allowance to petitioner
judge as well as the corresponding budgets of the municipality providing for the said
monthly allowance to petitioner judge. Under Section 327 of the Local Government Code
of 1991, the Sagguniang Panlalawigan was specifically tasked to review the appropriation
ordinances of its component municipalities to ensure compliance with Sections 324 and
32 5of the Code. Considering said duty of the Sangguniang Panlalawigan, we will assume,
in the absence of proof to the contrary, that the Sangguniang Panlalawigan of
Oriental Mindoro performed what the law required it to do, that is, review the resolution
and the corresponding budgets of the Municipality ofNaujan to make sure that they
complied with Sections 324 and 325 of the Code.

We presume the regularity of the Sangguniang Panlalawigans official act. Moreover, it is


well-settled that an ordinance must be presumed valid in the absence of evidence
showing that it is not in accordance with the law. Respondent COA had the burden of
proving that Resolution No. 101 of the Sangguniang Bayan of Naujan did not comply
with the condition provided in Section 447 of the Code, the budgetary requirements and
general limitations on the use of municipal funds provided in Sections 324 and 325 of the
Code and the implementing guidelines issued by the DBM, i.e., paragraphs (a) to (d),
Section 3 of LBC No. 53. Respondent COA also had the burden of showing that
the Sangguniang Panlalawigan ofOriental Mindoro erroneously approved said resolution
despite its non-compliance with the requirements of the law. It failed to discharge such
burden. On the contrary, wefindthat the resolution of the Municipality of Naujan granting
the P1,600 monthly allowance to petitioner judge fully complied with the law. Thus, we
uphold its validity.

Page 48 of 166
CASE NO. 22
MUYOT VS. DE LA FUENTE
48 O.G. 4860

Relevant Doctrine:
The City of Manila could not lease a portion of a public sidewalk on Plaza Sta. Cruz, being
likewise beyond the commerce of man. Property for public use is outside the commerce
of man (Arts. 420, 424, Civil Code). Being outside the commerce of man, it may not be the
subject of lease or other contract.

Page 49 of 166
CASE NO. 23
THE MUNICIPALITY OF CAVITE VS. HILARIA ROJAS
G.R. No. L-9069
March 31, 1915

Facts:
In 1907, the municipal council of Cavite leased to Rojas a part of Plaza Soledad, on the
condition that she will pay rent. The defendants constructed thereon a house, through
payment to the plaintiff for occupation thereof of a rental of P5. 58 a quarter in advance,
and with a condition that the defendants being obligated to vacate the leased land within
60 days after the demand; However, the defendants refused to do so, claiming that they
had acquired the right of possession thereof. They further claimed that according to the
lease, they could only be ordered to vacate the land leased when the plaintiff municipality
might need it for decoration or other public use.

Petitioner’s contention:

1. That the lease secured from the municipality of Cavite, by virtue whereof the
defendants occupy the land, is ultra vires and therefore ipso facto null and void
and of no force or effect, for the said land is an integral portion of a public plaza
of public domain and use,
2. That the municipal council of Cavite has never at any time had any power or
authority to withdraw it from public use, and to lease it to a private party for his
own use, and so the defendants have never had any right or occupy or to retain
the said land under leasehold, or in any other way, their occupation of the parcel
being furthermore illegal;
3. That judgment be rendered declaring that possession of the said land lies with
the plaintiff and ordering the defendants to vacate the land and deliver
possession thereof to said plaintiff, with the costs against the defendants.

Defendant’s contention:

1. Defendants admitted some of the allegations contained in the complaint but


denied that the parcel of land which they occupy and to which the complaint refers
forms and integral part of Plaza Soledad, or that the lease secured by them from
the municipality of Cavite was null and void and ultra vires, stating if they refused
to vacate said land it was because they had acquired the right of possession
thereof.

2. They alleged that, according to the lease, they could only be ordered to vacate the
land leased when the plaintiff municipality might need it for decoration or other

Page 50 of 166
public use, which does not apply in the present case;

3. That on the land which is the subject matter of the complaint the defendants have
erected a house of strong materials, assessed at P3,000, which was constructed
under a license secured from the plaintiff municipality;

4. That if they should be ordered to vacate the said land they would suffer damages
to the extent of P3,000, wherefore they prayed that they be absolved from the
complaint, or in the contrary case that the plaintiff be sentenced to indemnify them
in the sum of P3,000 as damages, and to pay the costs.

Issue:
Whether or not the contract of lease entered into by the parties is valid.

Ruling:
The lease was null and void.

The defendant has no right to continue to occupy the land for it is an integral part of the
plaza which is for public use and is reserved for the common benefit. Property for public
use in provinces and in towns comprises the provincial and town roads, the squares,
streets, fountains, and public waters, the promenades, and public works of general service
supported by said towns or provinces.

The said Plaza being a promenade for public use, the municipal council of Cavite could
not in 1907 withdraw or exclude from public use a portion thereof in order to lease it for
the sole benefit of the defendant Hilaria Rojas.

The plaintiff municipality exceeded its authority in the exercise of its powers by executing
a contract over a thing of which it could not dispose, nor is it empowered so to do. The
Civil Code, articles 1271, prescribes that everything which is not outside the commerce of
man may be the object of a contract, and plazas and streets are outside of this commerce.
Therefore, it must be concluded that the said lease is null and void. Further, the
defendants must restore and deliver possession of the land. On the other hand the
plaintiff must reimburse to the defendants the rentals they have paid as soon as they
restore the land improperly leased.

Page 51 of 166
CASE NO. 24
MUNICIPALITY OF PILILLA, RIZAL VS. COURT OF APPEALS
G.R. No. 105909
June 28, 1994

Facts:
Petitioner questions and seeks the nullification of the resolution of respondent Court of
Appeals in CA-G.R. SP. No. 27504 dated March 31, 1992, dismissing the petition for having
been filed by a private counsel, as well as its succeeding resolution dated June 9, 1992,
denying petitioner's motion for reconsideration.

On March 17, 1989, the Regional Trial Court of Tanay, Rizal, Branch 80, rendered judgment
in Civil Case No. 057-T in favor of plaintiff, now herein petitioner Municipality of Pililla,
Rizal, against defendant, now herein private respondent Philippine Petroleum Corporation
(PPC, for short), ordering therein defendant to pay said plaintiff:

(1) the amount of P5,301,385.00 representing the tax on business due from the defendant
under Section 9(A) of Municipal Tax Ordinance No. 1 of said municipality for the period
from 1979 to 1983, inclusive, plus such amount of tax as may accrue until final
determination of the case;
(2) storage permit fee in the amount of P3,321,730.00 due from the defendant under same
municipal tax ordinance for the period from 1975 to 1986, inclusive, plus the amount of
said fee that may accrue until final determination of the case;

(3) mayor's permit fee due from the defendant from 1975 to 1984, inclusive, in the amount
of P12,120.00, plus such amount of the same fee as may accrue until final determination
of the case;

(4) sanitary inspection fee in the amount of P1,010.00 for the period from 1975 to 1984,
plus the amount of this fee that may accrue until final determination of the case; and
(5) the costs of suit

Atty. Felix E. Mendiola served as counsel for the Municipality of Pililia in a collection suit
for unpaid business taxes, storage permit fee, mayor’s permit fee, sanitary inspection fee,
and the cost of the suit against private respondent Philippine Petroleum Corporation
(PPC).

The municipality won in the trial court, and when PPC elevated the case to the Supreme
Court, the SC affirmed the aforesaid judgment. The judgment became final and executory
and the records were remanded to the trial court for execution.

Page 52 of 166
In connection with the execution of said judgment, Atty. Felix E. Mendiola filed a motion
in behalf of the municipality for the examination of defendant corporation's gross sales
for the years 1976 to 1978 and 1984 to 1991 for the purpose of computing business tax.
Defendant corporation filed a manifestation that Pililla Mayor Nicomedes Patenia
received from it the sum of P11,457,907.00 as full satisfaction of the above-mentioned
judgment of the Supreme Court, as evidence by the release and quitclaim documents
executed by said mayor. The RTC denied the municipality's motion for examination and
execution of judgment on the ground that the judgment had already been satisfied.

It was when the case was only when the case was brought before to the CA that
respondent PPC filed a motion questioning Atty. Mendiola's authority to represent
petitioner municipality. The Court of Appeals dismissed the petition for having been filed
by a private counsel in violation of law and jurisprudence, but without prejudice to the
filing of a similar petition by the Municipality of Pililla through the proper provincial or
municipal legal officer.

Issue:
Whether or not Atty. Mendiola can represent the Municipality of Pilila

Ruling:
No. The Court of Appeals is correct in holding that Atty. Mendiola has no authority to file
a petition in behalf of and in the name of the Municipality of Pililla. Section 1683 of the
Revised Administrative Code provides:
Section 1683. Duty of fiscal to represent provinces and provincial subdivisions in
litigation. — The provincial fiscal shall represent the province and any municipality
or municipal district thereof in any court, except in cases whereof original
jurisdiction is vested in the Supreme Court or in cases where the municipality or
municipal district in question is a party adverse to the provincial government or to
some other municipality or municipal district in the same province. When the
interests of a provincial government and of any political division thereof are
opposed, the provincial fiscal shall act on behalf of the province.

When the provincial fiscal is disqualified to serve any municipality or other political
subdivision of a province, a special attorney may be employed by its council.

Only the provincial fiscal and the municipal attorney can represent a province or
municipality in their lawsuits. The provision is mandatory. The municipality's
authority to employ a private lawyer is expressly limited only to situations where
the provincial fiscal is disqualified to represent it.

Page 53 of 166
The fact that the provincial fiscal was disqualified to handle the municipality's case must
appear on record. In the case, there is nothing in the records to show that the provincial
fiscal is disqualified to act as counsel for the Municipality of Pililla on appeal, hence the
appearance of herein private counsel is without authority of law.

The submission of Atty. Mendiola that the exception is broad enough to apply to
situations where the provincial fiscal refuses to handle the case cannot be sustained. The
fiscal's refusal to represent the municipality is not a legal justification. A fiscal cannot
refuse to perform his functions on grounds not provided for by law without violating his
oath of office. Instead of engaging the services of a special attorney, the municipal council
should request the Secretary of Justice to appoint an acting provincial fiscal in place of
the provincial fiscal who has declined to handle and prosecute its case in court.

It should also be noted that the lack of authority of Atty. Mendiola, was even raised by
the municipality itself in its comment and opposition to said counsel's motion for
execution of his lien, which was filed by the office of the Provincial Prosecutor of Rizal in
behalf of said municipality.

The contention of Atty. Mendiola that private respondent cannot raise for the first time
on appeal his lack of authority to represent the municipality is untenable. The legality of
his representation can be questioned at any stage of the proceedings.

Also, even assuming that the representation of the municipality by Atty. Mendiola was
duly authorized, said authority is deemed to have been revoked by the municipality when
the latter, through the municipal mayor and without said counsel's participation, entered
into a compromise agreement with PPC.

WHEREFORE, the petition at bar is DENIED for lack of merit and the judgment of
respondent Court of Appeals is hereby AFFIRMED.

Page 54 of 166
CASE NO. 25
ZONSAYDA L. ALINSUG VS. REGIONAL TRIAL COURT
G.R. No. 108232
August 23, 1993

Facts:
Zonsayda L. Alinsug, had been a regular employee of the municipal government of
Escalante, Negros Occidental, when she received a permanent appointment as Clerk III in
the office of the Municipal Planning and Development Coordinator of the same
municipality. On 19 June 1992, Zonsayda absented herself from work allegedly to attend
to family matters. She had asked permission from the personnel officer but not from the
mayor. On 23 June 1992, Mayor Ponsica issued Office Order No. 31, suspending Zonsayda
for one month and one day commencing on 24 June 1992 for "a simple misconduct . . .
which can also be categorized as an act of insubordination." The order also stated that
the suspension "carries with it forfeiture of . . . benefits such as . . . salary and PERA and
leave credits during the duration of its effectivity."

Zonsayda filed with the Regional Trial Court of Negros Occidental, in San Carlos City, a
petition, for "injunction with damages and prayer for temporary restraining order and
preliminary injunction" against Mayor Ponsica and the municipal treasurer.

Mayor Ponsica and the municipal treasurer filed an answer to the petition, through private
practitioner Samuel SM Lezama, alleging that the petitioner had not exhausted
administrative remedies and that her suspension was in accordance with law.

Petitioner claimed that the respondents be all declared in default on the ground that since
the respondents were sued in their official capacities, "not including their private
capacities," they should have been represented by either the municipal legal officer or the
provincial legal officer or prosecutor as provided for by Sec. 481 (b) [i] and [3] of the Local
Government Code. It also cited Sec. 1 of Rep. Act No. 10 and Art. 177 of the Revised Penal
Code which penalizes usurpation of public authority.

While the defendant claimed that the municipality of Escalante has no legal officer, they
asserted that both the Local Government Code and the Administrative Code of 1987 do
not have any provision "relative to the duty of any provincial legal officer or prosecutor
to represent a municipality or its officials in suits filed against them by an employee or a
private individual." They contended that it was "unnecessary to provide such a provision
because there (exist) administrative and judicial rulings sustaining the validity of the
employment of a private counsel by municipal officials. Moreover, since the petitioner
prayed for the award of moral damages," on the strength of this Court's ruling in Albuera
v. Torrens,3 their hiring of a private counsel was justified.

Page 55 of 166
Issue:

Whether or not a private counsel may represent municipal officials sued in their official
capacities

Ruling:
The appointment of a legal officer shall be mandatory for the provincial and city
governments and optional for the municipal government. Section 481, Article 11 of Title
V of the Local Government Code, paragraph (i) states one of the functions of the legal
officer:
(i) Represent the local government unit in all civil actions and special
proceedings wherein the local government unit or any official thereof, in
his official capacity, is a party: Provided, that in actions or proceedings
where a component city or municipality is a party adverse to the
provincial government or to another component city or municipality, a
special legal officer may be employed to represent the adverse party.

Indeed, it appears that the law allows a private counsel to be hired by a municipality only
when the municipality is an adverse party in a case involving the provincial government
or another municipality or city within the province. This provision has its apparent origin
in the ruling in De Guia v. The Auditor General where the Court held that the municipality's
authority to employ a private attorney is expressly limited only to situations where the
provincial fiscal would be disqualified to serve and represent it. With Sec. 1683 of the old
Administrative Code as legal basis, the Court therein cited Enriquez, Sr. v. Gimenez which
enumerated instances when the provincial fiscal is disqualified to represent in court a
particular municipality; if and when original jurisdiction of case involving the municipality
is vested in the Supreme Court, when the municipality is a party adverse to the provincial
government or to some other municipality in the same province, and when, in a case
involving the municipality, he, or his wife, or child, is pecuniarily involved, as heir legatee,
creditor or otherwise.

Page 56 of 166
CASE NO. 26
MUNICIPALITY OF CANDIJAY, BOHOL VS. COURT OF APPEALS
G.R. No. 116702
December 28, 1995
Facts:
The lower court declared barrio/barangay Pagahat within the territorial jurisdiction of the
Municipality of Candijay therefor making said barrio as part and parcel of its territory. Said
order also permanently enjoined the municipality of Alicia to respect Candijay’s control,
possession, and polititcal supervision of barangay Pagahat and never to molest, disturb,
harass its possession and ownership over the same barrio.

On appeal, the respondent reversed the decision of the lower court. According to
respondent court the lower court erred in its decision and rejected the boundary line
being claimed by petitioner because such boundary line would not just place barrio
Pagahat under the territorial jurisdiction of petitioner but would also engulf other
barrios/barangays or portions of them under the Municipality of Mabini.

Also it was stated by the respondent court that there was equiponderance of evidence
making them rule in favor of the defendant Municipality. According to the
equiponderance of evidence rule, the plaintiff must rely on the strength of his evidence
and not on the weakness of defendant's claim. Even if the evidence of the plaintiff may
be stronger than that of the defendant, there is no preponderance of evidence on his side
if such evidence is insufficient in itself to establish his cause of action.

The petitioner then came to the Court alleging (1) improper application by the respondent
Court of Appeals of the so-called principle of "equiponderance of evidence", for having
based its ruling against petitioner on documentary evidence which, petitioner claims, are
void, (2) the respondent municipality's purported lack of juridical personality, as a result
of having been created under a void executive order, and (iii) that the challenged Decision
"does not solve the problem of both towns but throws them back again to their
controversy."

Issues:
A. Whether or not the equiponderance of evidence rule was correctly applied?
B. Whether or not the Municipality of Alicia is a de jure municipality?

Ruling:
A. Yes. The court said in this wise:

“With respect to the first and third grounds, we find that the issues of fact in this
case had been adequately passed upon by respondent Court in its Decision, which

Page 57 of 166
is well-supported by the evidence on record. The determination of equiponderance
of evidence by the respondent Court involves the appreciation of evidence by the
latter tribunal, which will not be reviewed by this Court unless shown to be
whimsical or capricious; here, there has been no such showing.”

B. Yes. Despite the ruling in Pelaez vs Auditor General declaring Section 68 of the Revised
Administrative Code, on which Executive Order 265 was based unconstitutional
because it constituted an undue delegation of legislative powers to the President of
the Philippines, the Court said that the instant case is similar with the case of San
Andres, where the latter municipality was considered as a de jure municipality by the
same court even if it was also created through an Executive Order.

The court said in this wise:

“Inasmuch as respondent municipality of Alicia is similarly situated as the


municipality of San Andres, it should likewise benefit from the effects of Section
442 (d) of the Local Government Code, and should henceforth be considered as a
regular, de jure municipality.”

Page 58 of 166
CASE NO. 27
PROVINCE OF CAMARINES NORTE VS. PROVINCE OF QUEZON, RESPONDENT
G.R. No. 80796
November 8, 1989

Facts:
Historically the Province of Camarines Norte and Camarines Sur was merge as one entity
by the Spanish colonial administration which they called Ambos Camarines. On the other
hand, the Province of Quezon was formerly known as Tayabas.

The boundary between Ambos Camarines and Tayabas was defined and written into law
in 1916, by Section 47 of Act No. 2657 (the Administrative Code). Although Act No. 2657
was repealed the following year by Act No. 2711 (the Revised Administrative Code), the
provisions pertaining to said boundary remained unaltered.

By the powers vested in him by law, Governor-General Francis Burton Harrison on March
30 , 1920 issued Executive Order No. 22, implementing Act No. 2809 and formally re-
establishing Camarines Norte as a province separate and distinct from Camarines Sur,
effective April 15, 1920.

The then Chief of the Executive Bureau, acting upon the authority of the Secretary of the
Interior, rendered on June 16, 1922 a decision delineating that portion of the boundary
between the provinces of Camarines Norte and Tayabas which is here involved.
Despite several official directives from the then Secretary of the Interior and repeated
efforts on the part of petitioner Camarines Norte, over the years, to enforce the same. All
efforts at amicable resolution of the boundary dispute (the last such effort having been
made sometime in 1987) have failed. Respondent Province of Quezon, now as in the past,
has simply refused to recognize as valid, and has frustrated all attempts to locate on the
ground, survey and monument the segment of the Ambos Camarine-Tayabas boundary
line delineated in the 1922 decision.

Issues:
A. Whether or not the boundaries between Ambos Camarines and Quezon Province were
already defined?
B. Whether or not there was legal authority on the June 16, 1922 decision of the Chief of
the Executive Bureau delineating the boundaries between Ambos Camarines and
Tayabas?

Page 59 of 166
Ruling:
A. No. The court said in this wise:

“We consider that to that limited extent, the Ambos Camarines-Quezon boundary
line was "undefined" and that there was thus necessity for the 16 June 1922
decision of the Chief of the Executive Bureau to provide more specific guidance
that would permit actual Identification or location of the Basiad Bay-Mt. Cadig
portion of the boundary line between Ambos Camarines and Quezon Province:

[from the peak of Mt. Cadig] thence a straight line is drawn to the point of
intersection of the inter provincial road between Camarines Norte and Tayabas
with the Tabugon River, thence following the course of the river to its mouth at the
Basiad Bay.

B. Yes. The court said that should it be assumed, finally, that prior legislative authority
was nonetheless necessary for the legal effectivity and enforceability of the 16 June
1922 decision of the Chief of the executive Bureau, we believe and so hold that that
prior legislative authority was supplied by Act No. 2809.

The Court also said in this wise:

In sum, we hold that the decision of the Chief of the Executive Bureau dated 16
June 1922 was lawfully issued and is binding upon the parties. We hold further that
prohibition and mandamus will lie for the enforcement of that decision, an
enforcement unjustifiably resisted and delayed for sixty-seven (67) years.

Page 60 of 166
CASE NO. 28
PHYSICAL THERAPY ORGANIZATION OF THE PHILIPPINES, INC., VS. MANILA
G.R. No. L-10448
August 30, 1957
Facts:
Physical Therapy Organization of the Philippines, an association of registered massagists
and licensed operators of massage clinics in the City of Manila and other parts of the
country, filed an action in the Court of First Instance of Manila for declaratory judgment
regarding the validity of Municipal Ordinance No. 3659, promulgated by the Municipal
Board and approved by the City Mayor. To stop the City from enforcing said ordinance,
the petitioner was able to secure an injunction. A hearing was held, but the parties without
introducing any evidence submitted the case for decision on the pleadings, although they
submitted written memoranda. Thereafter, the trial court dismissed the petition and later
dissolved the writ of injunction previously issued.

The petitoners filed an appeal directly to the Supreme Court questioning the decision of
the trial court in upholding the validity of Municipal Ordinance No. 3659 and holding that
the Municipal Board has the power to enact the Municipal ordinance in question.

Issues:
A. Whether or not Municipal Ordinance No. 3659 is a valid ordinance?
B. Whether or not the Municipal Board has the authority to enact Municipal Ordinance
No. 3659?
C. Whether or not the permit fee of 100 pesos imposed upon operators unreasonable?

Ruling:
A. Yes. The ordinance is a valid ordinance and was not enacted to prohibit trade but
rather to control illegal acts such as prostitution. According to the Court the intention
of the Ordinance was correctly ascertained by Judge Hermogenes Concepcion,
presiding in the trial court, in his order of dismissal where he said: "What the Ordinance
tries to avoid is that the massage clinic run by an operator who may not be a masseur
or massagista may be used as cover for the running or maintaining a house of
prostitution."

B. Yes. The Court agrees with the City Fiscal calling their attention to Section 18 of the
New Charter of the City of Manila, Act No. 409, which gives legislative powers to the
Municipal Board to enact all ordinances it may deem necessary and proper for the
promotion of the morality, peace, good order, comfort, convenience and general
welfare of the City and its inhabitants. This is generally referred to as the General
Welfare Clause, a delegation in statutory form of the police power, under which

Page 61 of 166
municipal corporations, are authorized to enact ordinances to provide for the health
and safety, and promote the morality, peace and general welfare of its inhabitants.

C. Yes. The court said in this wise: “As regards the permit fee of P100.00, it will be seen
that said fee is made payable not by the masseur or massagist, but by the operator of
a massage clinic who may not be a massagist himself. Compared to permit fees
required in other operations, P100.00 may appear to be too large and rather
unreasonable. However, much discretion is given to municipal corporations in
determining the amount of said fee without considering it as a tax for revenue
purposes:

The amount of the fee or charge is properly considered in determining whether it is a


tax or an exercise of the police power. The amount may be so large as to itself show
that the purpose was to raise revenue and not to regulate, but in regard to this matter
there is a marked distinction between license fees imposed upon useful and beneficial
occupations which the sovereign wishes to regulate but not restrict, and those which
are inimical and dangerous to public health, morals or safety. In the latter case the fee
may be very large without necessarily being a tax. (Cooley on Taxation, Vol. IV, pp.
3516-17; underlining supplied.)”

Page 62 of 166
CASE NO. 29
MMDA VS. BEL-AIR VILLAGE ASSOCIATION, INC.,
G.R. No. 135962
March 27, 2000
Facts:
Petitioner MMDA is a government agency tasked with the delivery of basic services in
Metro Manila. Respondent Bel-Air Village Association, Inc. (BAVA) is a non-stock, non-
profit corporation whose members are homeowners in Bel-Air Village, a private
subdivision in Makati City. Respondent received from petitioner, through its Chairman, a
notice requesting respondent to open Neptune Street to public vehicular traffic starting
January 2, 1996. CA held that MMDA has no authority to order the opening of Neptune
Street, a private subdivision road and cause the demolition of its perimeter walls. It held
that the authority is lodged in the City Council of Makati by ordinance.

Issue:
Whether or not Metropolitan Manila Development Authority (MMDA) has the mandate
to open Neptune street to public traffic pursuant to its regulatory and police powers?

Ruling:
NO. Police power is an inherent attribute of sovereignty. It has been defined as the power
vested by the Constitution in the legislature to make, ordain, and establish all manner of
wholesome and reasonable laws, statutes and ordinances, either with penalties or without,
not repugnant to the Constitution, as they shall judge to be for the good and welfare of
the commonwealth, and for the subjects of the same. The power is plenary and its scope
is vast and pervasive, reaching and justifying measures for public health, public safety,
public morals, and the general welfare.

It bears stressing that police power is lodged primarily in the National Legislature. It
cannot be exercised by any group or body of individuals not possessing legislative
power. The National Legislature, however, may delegate this power to the President and
administrative boards as well as the lawmaking bodies of municipal corporations or local
government units. Once delegated, the agents can exercise only such legislative powers
as are conferred on them by the national lawmaking body.

A local government is a "political subdivision of a nation or state which is constituted by


law and has substantial control of local affairs." The Local Government Code of 1991
defines a local government unit as a "body politic and corporate." — one endowed with
powers as a political subdivision of the National Government and as a corporate entity
representing the inhabitants of its territory. Local government units are the provinces,
cities, municipalities and barangays. They are also the territorial and political subdivisions
of the state.

Page 63 of 166
Our Congress delegated police power to the local government units in the Local
Government Code of 1991. This delegation is found in Section 16 of the same Code,
known as the general welfare clause, viz:
Sec. 16. General Welfare. — Every local government unit shall exercise the powers
expressly granted, those necessarily implied therefrom, as well as powers
necessary, appropriate, or incidental for its efficient and effective governance, and
those which are essential to the promotion of the general welfare. Within their
respective territorial jurisdictions, local government units shall ensure and support,
among other things, the preservation and enrichment of culture, promote health
and safety, enhance the right of the people to a balanced ecology, encourage and
support the development of appropriate and self-reliant scientific and
technological capabilities, improve public morals, enhance economic prosperity
and social justice, promote full employment among their residents, maintain peace
and order, and preserve the comfort and convenience of their inhabitants. 21

Local government units exercise police power through their respective legislative bodies.
The legislative body of the provincial government is the sangguniang panlalawigan, that
of the city government is the sangguniang panlungsod, that of the municipal government
is the sangguniang bayan, and that of the barangay is the sangguniang barangay. The
Local Government Code of 1991 empowers the sangguniang panlalawigan, sangguniang
panlungsod and sangguniang bayan to "enact ordinances, approve resolutions and
appropriate funds for the general welfare of the [province, city or municipality, as the case
may be], and its inhabitants pursuant to Section 16 of the Code and in the proper exercise
of the corporate powers of the [province, city municipality] provided under the Code.
" The same Code gives the sangguniang barangay the power to "enact ordinances as may
be necessary to discharge the responsibilities conferred upon it by law or ordinance and
to promote the general welfare of the inhabitants thereon."

Page 64 of 166
CASE NO. 30
MAGTAJAS VS. PRYCE PROPERTIES CORPORATION, INC.
G.R. No. 111097
July 20, 1994

Facts:
When PAGCOR announced the opening of a casino in Cagayan de Oro City, civic
organizations angrily denounced the project. The religious elements echoed the objection
and so did the women's groups and the youth. Demonstrations were led by the mayor
and the city legislators. The media trumpeted the protest, describing the casino as an
affront to the welfare of the city.

AN ORDINANCE PROHIBITING THE ISSUANCE OF BUSINESS PERMIT AND CANCELLING


EXISTING BUSINESS PERMIT TO ANY ESTABLISHMENT FOR THE USING AND ALLOWING
TO BE USED ITS PREMISES OR PORTION THEREOF FOR THE OPERATION OF CASINO was
passed by the Sangguniang Panlungsod. AN ORDINANCE PROHIBITING THE OPERATION
OF CASINO AND PROVIDING PENALTY FOR VIOLATION THEREFOR was also enacted.
PRYCE PROPERTIES is the lessor of PAGCOR. Pryce and PAGCOR assailed the ordinances.
PAGCOR is a corporation created directly by P.D. 1869 to help centralize and regulate all
games of chance, including casinos on land and sea within the territorial jurisdiction of
the Philippines. Cagayan de Oro City, like other local political subdivisions, is empowered
to enact ordinances for the purposes indicated in the Local Government Code. It is
expressly vested with the police power under what is known as the General Welfare
Clause.now embodied in Section 16 as follows:

Sec. 16. — General Welfare. — Every local government unit shall exercise the powers
expressly granted, those necessarily implied therefrom, as well as powers necessary,
appropriate, or incidental for its efficient and effective governance, and those which are
essential to the promotion of the general welfare. Within their respective territorial
jurisdictions, local government units shall ensure and support, among other things, the
preservation and enrichment of culture, promote health and safety, enhance the right of
the people to a balanced ecology, encourage and support the development of
appropriate and self-reliant scientific and technological capabilities, improve public
morals, enhance economic prosperity and social justice, promote full employment among
their residents, maintain peace and order, and preserve the comfort and convenience of
their inhabitants.

The petitioners argue that by virtue of these provisions, the Sangguniang Panlungsod may
prohibit the operation of casinos because they involve games of chance, which are
detrimental to the people. Gambling is not allowed by general law and even by the
Constitution itself. The legislative power conferred upon local government units may be

Page 65 of 166
exercised over all kinds of gambling and not only over "illegal gambling" as the
respondents erroneously argue. Even if the operation of casinos may have been permitted
under P.D. 1869, the government of Cagayan de Oro City has the authority to prohibit
them within its territory pursuant to the authority entrusted to it by the Local Government
Code.

Issue:
Whether or not the Ordinance is valid.

Ruling:
No. The tests of a valid ordinance are well established. A long line of decisions 9 has held
that to be valid, an ordinance must conform to the following substantive requirements:
1) It must not contravene the constitution or any statute.
2) It must not be unfair or oppressive.
3) It must not be partial or discriminatory.
4) It must not prohibit but may regulate trade.
5) It must be general and consistent with public policy.
6) It must not be unreasonable.

The ordinances violate P.D. 1869, which has the character and force of a statute, as well
as the public policy expressed in the decree allowing the playing of certain games of
chance despite the prohibition of gambling in general. The rationale of the requirement
that the ordinances should not contravene a statute is obvious. Municipal governments
are only agents of the national government. Local councils exercise only delegated
legislative powers conferred on them by Congress as the national lawmaking body. The
delegate cannot be superior to the principal or exercise powers higher than those of the
latter. It is a heresy to suggest that the local government units can undo the acts of
Congress, from which they have derived their power in the first place, and negate by mere
ordinance the mandate of the statute.

The power of PAGCOR to centralize and regulate all games of chance, including casinos
on land and sea within the territorial jurisdiction of the Philippines, remains unimpaired.
P.D. 1869 has not been modified by the Local Government Code, which empowers the
local government units to prevent or suppress only those forms of gambling prohibited
by law.

Page 66 of 166
CASE NO. 31
SEN. HEHERSON T. ALVAREZ VS. HON. TEOFISTO T. GUINGONA, JR
G.R. No. 118303
January 31, 1996

Facts:
On April 18, 1993, House Bill No. 8817 (HB No. 8871), entitled “An Act Converting the
Municipality of Santiago into an Independent Component City to be known as the City of
Santiago,” was filed in the House of Representatives. Meanwhile, a counterpart of HB No.
8817, Senate Bill No. 1243, entitled, An Act Converting the Municipality of Santiago into
an Independent Component City to be Known as the City of Santiago, was filed in the
Senate.
On February 23, 1994, or a little less than a month after HB No. 8817 was transmitted to
the Senate, the Senate Committee on Local Government conducted public hearings on
SB No. 1243.
On March 22, 1994, the House of Representatives, upon being apprised of the action of
the Senate, approved the amendments proposed by the Senate.
On April 12, 1994, the enrolled bill was submitted to the President, and was signed by the
Chief Executive on May 5, 1994 as Republic Act No. 7720 (RA No. 7720). When a plebiscite
on the Act was held on July 13, 1994, a great majority of the registered voters of Santiago
voted in favor of the conversion of Santiago into a city.

Issue:
A. Whether or not the Internal Revenue Allotments (IRAs) are to be included in the
computation of the average annual income of a municipality for purposes of its
conversion into an independent component city.
B. Whether or not, considering that the Senate passed SB No. 1243, its own version of
HB No. 8817, RA No. 7720 can be said to have originated in the House of
Representatives.

Ruling:
A. Yes, the IRAs are to be included in the computation of the average annual income of
a municipality for purposes of its conversion into an independent component city. The
court held that petitioners’ asseverations are untenable because Internal Revenue
Allotments form part of the income of Local Government Units. Section 450 (c) of the
Local Government Code provides that "the average annual income shall include the
income accruing to the general fund, exclusive of special funds, transfers, and non-
recurring income." To reiterate, IRAs are a regular, recurring item of income; nil is there
a basis, too, to classify the same as a special fund or transfer, since IRAs have a
technical definition and meaning all its own as used in the Local Government Code
that unequivocally makes it distinct from special funds or transfers referred to when

Page 67 of 166
the Code speaks of "funding support from the national government, its
instrumentalities and government-owned-or-controlled corporations".

B. Yes, RA No. 7720 can be said to have originated in the House of Representatives. The
Court ruled that HB No. 8817 was filed in the House of Representatives first before SB
No. 1243 was filed in the Senate. Petitioners themselves cannot disavow their own
admission that HB No. 8817 was filed on April 18, 1993 while SB No. 1243 was filed on
May 19, 1993. The filing of HB No. 8817 was thus precursive not only of the said Act
in question but also of SB No. 1243. Thus, HB No. 8817, was the bill that initiated the
legislative process that culminated in the enactment of Republic Act No. 7720. No
violation of Section 24, Article VI, of the 1987 Constitution is perceptible under the
circumstances attending the instant controversy.

Furthermore, petitioners themselves acknowledge that HB No. 8817 was already


approved on Third Reading and duly transmitted to the Senate when the Senate
Committee on Local Government conducted its public hearing on HB No. 8817. HB
No. 8817 was approved on the Third Reading on December 17, 1993 and transmitted
to the Senate on January 28, 1994; a little less than a month thereafter, or on February
23, 1994, the Senate Committee on Local Government conducted public hearings on
SB No. 1243. Clearly, the Senate held in abeyance any action on SB No. 1243 until it
received HB No. 8817, already approved on the Third Reading, from the House of
Representatives. The filing in the Senate of a substitute bill in anticipation of its receipt
of the bill from the House, does not contravene the constitutional requirement that a
bill of local application should originate in the House of Representatives, for as long
as the Senate does not act thereupon until it receives the House bill.

The Court reiterated a ruling in Tolentino vs. Secretary of Finance, that what the
Constitution simply means is that the initiative for filing revenue, tariff, or tax bills, bills
authorizing an increase of the public debt, private bills and bills of local application
must come from the House of Representatives on the theory that, elected as they are
from the districts, the members of the House can be expected to be more sensitive to
the local needs and problems. On the other hand, the senators, who are elected at
large, are expected to approach the same problems from the national perspective.
Both views are thereby made to bear on the enactment of such laws. Nor does the
Constitution prohibit the filing in the Senate of a substitute bill in anticipation of its
receipt of the bill from the House, so long as action by the Senate as a body is withheld
pending receipt of the House bill.

Page 68 of 166
CASE NO. 32
CITY OF PASIG VS. THE HON. COMMISSION ON ELECTION (COMELEC)
G.R. No. 125646
September 10, 1999

Facts:
On April 22, 1996, residents of Karangalan Village filed a petition that they be segregated
from its mother Barangays Manggahan and Dela Paz, City of Pasig, and to be converted
and separated into a distinct barangay to be known as Barangay Karangalan. Thus, the
City Council of Pasig passed and approved Ordinance No. 21, Series of 1996, creating
Barangay Karangalan in Pasig City. Plebiscite on the creation of said barangay was
thereafter set for June 22, 1996.

Meanwhile, on September 9, 1996, the City of Pasig similarly issued Ordinance No. 52,
Series of 1996, creating Barangay Napico in Pasig City. Plebiscite for this purpose was set
for March 15, 1997.

Immediately upon learning of such ordinances, the Municipality of Cainta filed two (2)
Petitions with the COMELEC calling its attention to a pending case before the Regional
Trial Court of Antipolo, Rizal for the settlement of boundary disputes which claimed that
the proposed barangays involve areas included in the boundary dispute subject of said
pending case; hence, the scheduled plebiscite should be suspended or cancelled until
after the said case shall have been finally decided by the court.

The COMELEC accepted the position of the Municipality of Cainta and ordered the
plebiscite on the creation of Barangay Karangalan to be held in abeyance until after the
court has settled with finality the boundary dispute involving the two municipalities.
However, the COMELEC ruled differently in the other petition, dismissing the same for
being moot since the creation of Barangay Napico was already ratified and approved by
the majority of the votes cast in the plebiscite. Hence, these two (2) petitions by the City
of Pasig and the Municipality of Cainta.

Issue:
Whether or not the plebiscites scheduled for the creation of Barangays Karangalan and
Napico should be suspended or cancelled in view of the pending boundary dispute
between the two local governments.

Ruling:
The Court ruled that the pending civil case on boundary dispute presents a prejudicial
question which must first be decided before the creation of the proposed barangays.
While the City of Pasig argues that there is no prejudicial question since the same

Page 69 of 166
contemplates a civil and criminal action and does not come into play where both cases
are civil, as in the instant case, still in the interest of good order, the Court can suspend
action on one case pending the final outcome of another case closely interrelated or
linked to the first.

The decision on whose territorial jurisdiction the areas fall has material bearing to the
creation of the proposed Barangays. A requisite for the creation of a barangay is properly
identified territorial jurisdiction for these define the limits of the exercise of the
governmental powers of the LGU. Beyond these limits, its acts are ultra vires (beyond the
legal capacity). Moreover, considering the expenses entailed in the holding of plebiscites,
it is far more prudent to hold in abeyance the conduct of the same until the resolution of
the boundary dispute.

In the case of Barangay Napico, the Court does not agree that the petition of the
Municipality of Cainta has been rendered moot and academic because the plebiscite was
already held. The issues raised are still pending and must first be resolved.

Therefore, the plebiscite on the creation of Barangay Karangalan should be held in


abeyance; and the plebiscite held on March 15, 1997 ratifying the creation of Barangay
Napico should be annulled and set aside, and any plebiscite thereto is hold in abeyance
pending final resolution of the boundary dispute.

Page 70 of 166
CASE NO. 33
SANGALANG VS. INTERMEDIATE APPELLATE COURT
G.R. No. 71169
December 22, 1988

Facts:
Jose Sangalang and wife, herein petitioners are residents of Jupiter Street, Makati, Metro
Manila. Sangalang and the other petitioners who are also residents of Jupiter Street
initially filed a case against Ayala (appellee) to enforce by specific performance restrictive
easement upon property, specifically the Bel-Air subdivision in Makati, Metro Manila
pursuant to stipulations embodied in the deeds of sale covering the subdivision, and for
damages.

The lots which were acquired by appellees Sangalang and others were all sold by MDC
subject to certain conditions and easements contained in Deed Restrictions which form a
part of each deed of sale. When MDC sold the lots to appellees predecessors-in-interest,
the whole stretch of the commercial block between Buendia Avenue and Jupiter Street
was still undeveloped. Although it was not part of the original plan, MDC constructed a
fence or wall on the commercial block along Jupiter that was destroyed by typhoon but
was then rebuilt by appellee.

In 1975, the municipal council of Makati enacted its ordinance No. 81, providing for the
zonification of Makati. Under this Ordinance, Bel-Air Village was classified as a Class A
Residential Zone, with its boundary in the south extending to the center line of Jupiter
Street. Under the zoning classifications, Jupiter Street, therefore, is a common boundary
of Bel-Air Village and the commercial zone. Gates had been installed by Bel-Air Village
Association in strategic locations across Jupiter Street which were manned and operated
by its own security guards who were employed to maintain, supervise and enforce traffic
regulations in the roads and streets of the village.

On 1977, the office of the Mayor of Makati wrote to Bel-Air Village Association(BAVA)
directing that, in the interest of public welfare and for the purpose of easing traffic
congestion, some streets should be opened which BAVA complied with except for one,
the Jupiter street because residents are concern about the opening of Jupiter Street to
the general public. The Municipal Engineer of Makati wrote a letter to BAVA advising them
to open the gates of the entire portion of Jupiter Street and on August 12, 1977, the
municipal officials of Makati concerned allegedly opened, destroyed and removed the
gates constructed/located at the corner of Reposo Street and Jupiter Street as well as the
gates/fences located/constructed at Jupiter Street and Makati Avenue forcibly, and then
opened the entire length of Jupiter Street to public traffic.

Page 71 of 166
Then, on January 27, 1978, appellant donated the entire Jupiter Street from Metropolitan
Avenue to Zodiac Street to BAVA. However, even before 1978, the Makati Police and the
security force of BAVA were already the ones regulating the traffic along Jupiter Street
after the gates were opened in 1977. Thus, with the opening of the entire length of Jupiter
Street to public traffic, the different residential lots located in the northern side of Jupiter
Street ceased to be used for purely residential purposes. They became, for all purposes,
commercial in character.

Petitioners brought the present action for damages against the defendant-appellant
Ayala Corporation predicated on both breach of contract and on tort or quasi-delict. After
trial on the merits, the then Court of First Instance ruled in favor of the petitioners and
awarded damages, Defendant is Further ordered to restore/reconstruct the perimeter wall
at its original position in 1966 from Reposo Street in the west to Zodiac Street in the east,
at its own expense.

On appeal, CA reversed the lower court, finding the decision appealed from as not
supported by the facts and the law on the matter, it was set aside and another one entered
dismissing the case for lack of a cause of action.

Issue:
Whether the ordinances allowing the use of Jupiter Street both for residentian and
commercial purposes were a valid exercise of Police Power?

Ruling:
YES, while non-impairment of contracts is constitutionally guaranteed, the rule is not
absolute, since it has to be reconciled with the legitimate exercise of police power, i.e.,
"the power to prescribe regulations to promote the health, morals, peace, education,
good order or safety and general welfare of the people.' Invariably described as "the most
essential, insistent, and illimitable of powers" and "in a sense, the greatest and most
powerful attribute of government," the exercise of the power may be judicially inquired
into and corrected only if it is capricious, whimsical, unjust or unreasonable, there having
been a denial of due process or a violation of any other applicable constitutional
guarantee. As this Court held through Justice Jose P. Bengson in Philippine Long Distance
Company vs. City of Davao, et al. police power 'is elastic and must be responsive to various
social conditions; it is not confined within narrow circumscriptions of precedents resting
on past conditions; it must follow the legal progress of a democratic way of life.' We were
even more emphatic in Vda. de Genuino vs. The Court of agrarian Relations, et al., when
We declared: "We do not see why public welfare when clashing with the individual right
to property should not be made to prevail through the state's exercise of its police power."
Undoubtedly, the MMC Ordinance represents a legitimate exercise of police power. The
petitioners have not shown why we should hold otherwise other than for the supposed

Page 72 of 166
"non-impairment" guaranty of the Constitution, which, as we have declared, is secondary
to the more compelling interests of general welfare. The Ordinance has not been shown
to be capricious or arbitrary or unreasonable to warrant the reversal of the judgments so
appealed. In that connection, we find no reversible error to have been committed by the
Court of Appeals.

Page 73 of 166
CASE NO. 34
BALACUIT VS. COURT OF FIRST INSTANCE OF AGUSAN DEL NORTE
G.R. No. L-38429
June 30, 1988

Facts:
Ordinance No. 640 was passed by the Municipal Board of the City of Butuan on April 21,
1969 with the title of “ORDINANCE PENALIZING ANY PERSON, GROUP OF PERSONS,
ENTITY OR CORPORATION ENGAGED IN THE BUSINESS OF SELLING ADMISSION TICKETS
TO ANY MOVIE OR OTHER PUBLIC EXHIBITIONS, GAMES, CONTESTS OR OTHER
PERFORMANCES TO REQUIRE CHILDREN BETWEEN SEVEN (7) AND TWELVE (12) YEARS
OF AGE TO PAY FULL PAYMENT FOR TICKETS INTENDED FOR ADULTS BUT SHOULD
CHARGE ONLY ONE-HALF OF THE SAID TICKET”

The ordinance provides for the reduction to ½ of the ticket price given to minors from 7-
12 years old with a fine from 200-600 pesos or a 2-6 month imprisonment if violated.
Petitioners Carlos Balacuit, et al as managers of theaters, affected by the ordinance, filed
a Complaint before the CFI of Agusan del Norte and Butuan City praying that the subject
ordinance be declared unconstitutional and, therefore, void and unenforceable. A TRO
was then issued to prevent the law from being enforced but later the Court rendered
judgment declaring Ordinance No. 640 of the City of Butuan constitutional and Valid.
Petitioners then attack the validity and constitutionality of Ordinance No. 640 on the
grounds that it is ultra vires and an invalid exercise of police power. Petitioners contend
that Ordinance No. 640 is not within the power of' the Municipal Board to enact as
provided for in Section 15(n) of Republic Act No. 523 where it states that the Muncipal
board can only fix license fees for theaters and not admission rates.

The respondent attempts to justify the enactment of the ordinance by invoking the
general welfare clause embodied in Section 15 of the cited law.

Issue:
Whether or not Ordinance 640 prohibiting selling of theatre admission tickets to children
7-12 years old at full price is constitutional and a valid exercise of police power?

Ruling:
No, Ordinance 640 is declared unconstitutional. Its is already settled that the operation of
theaters, cinematographs and other places of public exhibition are subject to regulations
by the municipal council in the exercise of delegated police power by the local
government. However, to invoke the exercise of police power, not only must it appear
that the interest of the public generally requires an interference with private rights but
the means adopted must be reasonably necessary for the accomplishment of the purpose

Page 74 of 166
and not unduly oppressive upon individuals. The legislature may not, under the guise of
protecting the public interest, arbitrarily interfere with private business, or impose unusual
and unnecessary restrictions upon lawful occupations. In other words, the determination
as to what is a proper exercise of its police power is not final and conclusive, but is subject
to the supervisions of the courts.

The Court likewise ruled in the negative as to the question of the subject ordinance being
a valid exercise of police power. While it is true that a business may be regulated, it is
equally true that such regulation must be within the bounds of reason, that is, the
regulatory ordinance must be reasonable, and its provisions cannot be oppressive
amounting to an arbitrary interference with the business or calling subject of regulation.
The proprietors of a theater have a right to manage their property in their own way, to fix
what prices of admission they think most for their own advantage, and that any person
who did not approve could stay away.

The exercise of police power by the local government is valid unless it contravenes the
fundamental law of the land, or an act of the legislature, or unless it is against public policy
or is unreasonable, oppressive, partial, discriminating or in derogation of a common right.
Ordinance No. 640 clearly invades the personal and property rights of petitioners for even
if We could assume that, on its face, the interference was reasonable, from the foregoing
considerations, it has been fully shown that it is an unwarranted and unlawful curtailment
of the property and personal rights of citizens. For being unreasonable and an undue
restraint of trade, it cannot, under the guise of exercising police power, be upheld as valid.

Page 75 of 166
CASE NO. 35
ORTIGAS & CO., LIMITED PARTNERSHIP VS. FEATI BANK AND TRUST CO.
G.R. No. L-24670
December 14, 1979

Facts:
Plaintiff is engaged in real estate business, developing and selling lots to the public,
particularly the Highway Hills Subdivision along EDSA, Mandaluyong, Rizal. On March 4,
1952, plaintiff entered into separate agreements of sale with Augusto Padillay Angeles
and Natividad Angeles over 2 parcels of land (Lot nos. 5 and 6, Block 31, of the Highway
Hills Subdivision). On July 19, 1962 the vendees transferred their rights and interests over
the said lots to Emma Chavez. The plaintiff executed the corresponding deeds of sale in
favor of Emma Chaves upon payment of the purchase price. Both the agreements and the
deeds of sale thereafter executed contained the stipulation that the parcels of land subject
of the deeds of sale “shall be used by the Buyer exclusively for residential purposes”. The
restrictions were later annotated in the Transfer Certificate of Titles covering the said lots
issued in the name of Chavez. Eventually, defendant-appellee acquired Lots no. 5 and 6
with the building restrictions also annotated in their corresponding TCT’s. Lot No. 5 was
aught directly from Chavez “free from all liens and encumbrances” while Lot No.6 was
acquired through a “Deed of Exchange” from Republic Flour Mills.

Plaintiff claims that the restrictions were imposed as part of its general building scheme
designed for the beautification and development of the Highway Hills Subdivision which
forms part of its big landed estate where commercial and industrial sites are also
designated or established. Defendant maintains that the area along the western part of
EDSA from Shaw Boulevard to the Pasig River, has been declared a commercial and
industrial zone, per Resolution No.27 of the Municipal Council of Mandaluyong. It alleges
that plaintiff “completely sold and transferred to third persons all lots in said subdivision
facing EDSA” and the subject lots thereunder were acquired by it “only on June 23, 1962
or more than 2 years after the area xxx had been declared a commercial and industrial
zone”.
On or about May 5, 1963, defendant-appellee began construction of a building devoted
to banking purposes but which it claims could also be used exclusively for residential
purposes. The following day, the plaintiff demanded in writing that the construction of
the commercial building be stopped but the defendant refused to comply contending
that the construction was in accordance with the zoning regulations.

Issue:
A. Whether or not Resolution No. 27 is a valid exercise of police power

Page 76 of 166
B. Whether or not Resolution No. 27 declaring lot 5 and 6 to be part of an industrial and
commercial zone is valid considering the contract stipulation in the Transfer Certificate
Title.

Ruling:
A. Yes. The validity of Resolution No.27 was never questioned. In fact, it was impliedly
admitted in the stipulation of facts, when plaintiff-appellant did not dispute the same.
Having admitted the validity of the subject resolution, plaintiff-appellant cannot now
change its position on appeal.

However, assuming that it is not yet too late to question the validity of the said
resolution, the posture is unsustainable. Municipalities are empowered by law through
Sec.3 of RA 2264 (Local Autonomy Act) to to adopt zoning and subdivision ordinances
or regulations for the municipality. The law does not restrict the exercise of the power
through an ordinance. Therefore, granting that Resolution No.27 is not an ordinance,
it certainly is a regulatory measure within the intendment of the word “regulation”
under the provision. An examination of Sec.12 of the same law reveals that the implied
power of a municipality should be “liberally construed in its favor” and that “any fair
and reasonable doubt as to the existence of the power should be interpreted in favor
of the local government and it shall be presumed to exist.” An exception to the general
welfare powers delegated to municipalities is when the exercise of its powers will
conflict with vested rights arising from contracts. The exception does not apply to the
case at bar.

B. Yes, Resolution No. 27 prevails over the contract stipulations. Section 3 of RA 2264 of
the Local Autonomy Act empowers a Municipal Council to adopt zoning and
subdivision ordinances or regulations for the Municipality. Section 12 or RA 2264
states that implied power of the municipality should be “liberally construed in it’s
favour”, “to give more power to the local government in promoting economic
conditions, social welfare, and material progress in the community”. This is found in
the General Welfare Clause of the said act. Although non-impairment of contracts is
constitutionally guaranteed, it is not absolute since it has to be reconciled with the
legitimate exercise of police power, e.g. the power to promote health, morals, peace,
education, good order or safety and general welfare of the people. Resolution No. 27
was obviously passed in exercise of police power to safeguard health, safety, peace
and order and the general welfare of the people in the locality as it would not be a
conducive residential area considering the amount of traffic, pollution, and noise
which results in the surrounding industrial and commercial establishments.

Page 77 of 166
CASE NO. 36
SUGUITAN VS. CITY OF MANDALUYONG
G.R. No. 135087
March 14, 2000
Facts:
 October 13, 1994, the Sangguniang Panlungsod of Mandaluyong City issued
Resolution No. 396, S-1994 authorizing then Mayor Benjamin Abalos to institute
expropriation proceedings over the property of Alberto Suguitan
 Mayor Abalos wrote Suguitan a letter dated January 20, 1995 offering to buy his
property, but Suguitan refused to sell
 March 13, 1995 - Mandaluyong filed a complaint for expropriation with the Pasig
RTC
 Suguitan moved to dismiss the complaint on the following grounds
o (1) the power of eminent domain is not being exercised in accordance with
law
o (2) there is no public necessity to warrant expropriation of subject property
o (3) Mandaluyong seeks to expropriate the said property without payment
of just compensation
o (4) Mandaluyong has no budget and appropriation for the payment of the
property being expropriated
o (5) expropriation of Suguitan' s property is but a ploy of Mayor Abalos to
acquire the same for his personal use. Respondent filed its comment and
opposition to the motion
 October 24, 1995 - RTC denied Suguitan's motion to dismiss
 November 14, 1995 - Upon motion, RTC issued an order allowing Mandaluyong to
take immediate possession of Suguitan's property upon the deposit of P621,000
representing 15% of the fair market value of the lot based upon its current tax
declaration
 December 15, 1995 - Mandaluyong assumed possession of the subject property
by virtue of a writ of possession issued by the RTC on December 14, 1995
 July 28, 1998 - RTC granted the assailed order of expropriation. Hence, this petition

Issue:
Whether or not the expropriation was valid (NO)

Ruling:
 Suguitan: Mandaluyong may only exercise its delegated power of eminent domain
by means of an ordinance as required by LGC §19 and not by means of a mere
resolution
 Mandaluyong: Exercise of eminent domain power was valid and legal. Pursuant to
Art. 36, Rule VI of the LGC’s IRR, a resolution is a sufficient antecedent for the filing

Page 78 of 166
of expropriation proceedings. A "resolution" empowering the City Mayor to initiate
such expropriation proceedings [is sufficient] and thereafter when the court has
already determine[d] with certainty the amount of just compensation to be paid
for the property expropriated, then follows an Ordinance of the Sanggunian
Panlungsod appropriating funds for the payment of the expropriated property
 SC: LGU power of eminent domain comes from legislative delegation. It is therefore
subject to the same limitations as if the power was being exercised by the
legislature itself. The exercise by the LGUs has been guarded even more closely by
the courts. The nature of the LGU’s eminent domain power must be analyzed
 Eminent domain is the right or power of a sovereign state to appropriate private
property to particular uses to promote public welfare
 It is an indispensable attribute of sovereignty; a power grounded in the primary
duty of government to serve the common need and advance the general welfare
 City of Manila vs. Chinese Community of Manila: “The exercise of the right of
eminent domain, whether directly by the State, or by its authorized agents, is
necessarily in derogation of private rights, and the rule in that case is that the
authority must be strictly construed. No species of property is held by individuals
with greater tenacity, and none is guarded by the constitution and the laws more
sedulously, than the right to the freehold of inhabitants. When the legislature
interferes with that right, and, for greater public purposes, appropriates the land
of an individual without his consent, the plain meaning of the law should not be
enlarged by doubtful interpretation.”
 The power of eminent domain is essentially legislative in nature. However, it may
be validly delegated to LGUs, other public entities and public utilities
 Of course the scope of this delegated legislative power is necessarily narrower than
that of the delegating authority and may only be exercised in strict compliance
with the terms of the delegating law

REQUISITES OF VALID EXERCISE OF EMINENT DOMAIN POWER BY LGUs


1. An ordinance is enacted by the local legislative council authorizing the
local chief executive, in behalf of the local government unit, to exercise
the power of eminent domain or pursue expropriation proceedings
over a particular private property
2. The power of eminent domain is exercised for public use, purpose or
welfare, or for the benefit of the poor and the landless.
3. There is payment of just compensation, as required under § 9, Art. III
of the Constitution, and other pertinent laws
4. A valid and definite offer has been previously made to the owner of
the property sought to be expropriated, but said offer was not
accepted

Page 79 of 166
 Mandaluyong expropriated Suguitan’s property on the basis of a mere resolution,
in contravention of the first requisite. The law in this case is clear and free from
ambiguity. §19 of the Code requires an ordinance, not a resolution. An ordinance
is needed only to appropriate funds after the court has determined the amount of
just compensation.

 SC: Untenable. Ordinance is necessary to authorize the filing of a complaint with


the proper court since, beginning at this point, the power of eminent domain is
already being exercised

 Mandaluyong: A resolution is enough under the LGC-IRR. Art. 36 (a), Rule VI


provides that if the LGU fails to acquire a private property for public use, purpose,
or welfare through purchase, it may expropriate said property through a resolution
of the sanggunian authorizing its chief executive to initiate expropriation
proceedings

 SC remains aware of the constitutional policy of promoting local autonomy, but


judicial sanction cannot be granted to a LGU's exercise of its delegated power of
eminent domain in contravention of the very law giving it such power

 Mandaluyong is not precluded from enacting the necessary ordinance and


thereafter reinstituting expropriation proceedings, for so long as it has complied
with all other legal requirements

 Petition granted. RTC order reversed and set aside

Page 80 of 166
CASE NO. 37
ASSOCIATION OF SMALL LANDOWNERS VS. SEC OF AGRARIAN REFORM
G.R. No. 78742
July 14, 1989

Facts:
 RA 3844 was enacted in 1963. P.D. No. 27 was promulgated in 1972 to provide for
the compulsory acquisition of private lands for distribution among tenant-farmers
and to specify maximum retention limits for landowners. In 1987, President
Corazon Aquino issued E.O. No. 228, declaring full land ownership in favor of the
beneficiaries of PD 27 and providing for the valuation of still unvalued lands
covered by the decree as well as the manner of their payment. In 1987, P.P. No.
131, instituting a comprehensive agrarian reform program (CARP) was enacted;
later, E.O. No. 229, providing the mechanics for its (PP131’s) implementation, was
also enacted. Afterwhich is the enactment of R.A. No. 6657, Comprehensive
Agrarian Reform Law in 1988. This law, while considerably changing the earlier
mentioned enactments, nevertheless gives them suppletory effect insofar as they
are not inconsistent with its provisions.

G.R. No. 78742: (Association of Small Landowners vs Secretary)


 The Association of Small Landowners in the Philippines, Inc. sought exception from
the land distribution scheme provided for in R.A. 6657. The Association is
comprised of landowners of ricelands and cornlands whose landholdings do not
exceed 7 hectares. They invoke that since their landholdings are less than 7
hectares, they should not be forced to distribute their land to their tenants under
R.A. 6657 for they themselves have shown willingness to till their own land. In short,
they want to be exempted from agrarian reform program because they claim to
belong to a different class

G.R. No. 79777: (Manaay vs Juico)


 Nicolas Manaay questioned the validity of the agrarian reform laws (PD 27, EO 228,
and 229) on the ground that these laws already valuated their lands for the agrarian
reform program and that the specific amount must be determined by the
Department of Agrarian Reform (DAR). Manaay averred that this violated the
principle in eminent domain which provides that only courts can determine just
compensation. This, for Manaay, also violated due process for under the
constitution, no property shall be taken for public use without just compensation
 Manaay also questioned the provision which states that landowners may be paid
for their land in bonds and not necessarily in cash. Manaay averred that just
compensation has always been in the form of money and not in bonds

Page 81 of 166
Issues:
A. Whether or not there was a violation of the equal protection clause
B. Whether or not there is a violation of due process
C. Whether or not just compensation, under the agrarian reform program, must be in
terms of cash

Ruling:
A. No. The Association had not shown any proof that they belong to a different class
exempt from the agrarian reform program. Under the law, classification has been
defined as the grouping of persons or things similar to each other in certain particulars
and different from each other in these same particulars. To be valid, it must conform
to the following requirements:
(1) it must be based on substantial distinctions;
(2) it must be germane to the purposes of the law;
(3) it must not be limited to existing conditions only; and
(4) it must apply equally to all the members of the class.

Equal protection simply means that all persons or things similarly situated must be treated
alike both as to the rights conferred and the liabilities imposed. The Association have not
shown that they belong to a different class and entitled to a different treatment. The
argument that not only landowners but also owners of other properties must be made to
share the burden of implementing land reform must be rejected. There is a substantial
distinction between these two classes of owners that is clearly visible except to those who
will not see. The Congress is allowed a wide leeway in providing for a valid classification.
Its decision is accorded recognition and respect by the courts of justice except only where
its discretion is abused to the detriment of the Bill of Rights. In the contrary, it appears
that Congress is right in classifying small landowners as part of the agrarian reform
program

B. No. It is true that the determination of just compensation is a power lodged in the
courts. However, there is no law which prohibits administrative bodies like the DAR
from determining just compensation. In fact, just compensation can be that amount
agreed upon by the landowner and the government – even without judicial
intervention so long as both parties agree. The DAR can determine just compensation
through appraisers and if the landowner agrees, then judicial intervention is not
needed. What is contemplated by law however is that, the just compensation
determined by an administrative body is merely preliminary. If the landowner does not
agree with the finding of just compensation by an administrative body, then it can go
to court and the determination of the latter shall be the final determination

Page 82 of 166
C. No. Money as payment for just compensation is merely a concept in traditional
exercise of eminent domain. The agrarian reform program is a revolutionary exercise
of eminent domain. The program will require billions of pesos in funds if all
compensation have to be made in cash – if everything is in cash, then the government
will not have sufficient money hence, bonds, and other securities, i.e., shares of stocks,
may be used for just compensation

Page 83 of 166
CASE NO. 38
MODAY VS. COURT OF APPEALS
G.R. No. 107916
February 20, 1997

Facts:
 Percival Moday is a landowner in Bunawan, Agusan del Sur
 In 1989, the Sangguniang Bayan of Bunawan passed a resolution authorizing the
mayor to initiate an expropriation case against a 1 hectare portion of Moday’s land.
Purpose of which was to erect a gymnasium and other public buildings
 The mayor approved the resolution and the resolution was transmitted to the
Sangguniang Panlalawigan which disapproved the said resolution ruling that the
expropriation is not necessary because there are other lots owned by Bunawan that
can be used for such purpose
 The mayor pushed through with the expropriation nonetheless

Issue:
Whether or not a municipality may expropriate private property by virtue of a municipal
resolution which was disapproved by the Sangguniang Panlalawigan

Ruling:
 Yes. Eminent domain, the power which the Municipality of Bunawan exercised in
the instant case, is a fundamental State power that is inseparable from sovereignty
 It is government’s right to appropriate, in the nature of a compulsory sale to the
State, private property for public use or purpose. Inherently possessed by the
national legislature, the power of eminent domain may be validly delegated to
local governments, other public entities and public utilities
 For the taking of private property by the government to be valid, the taking must
be for public use and there must be just compensation. The only ground upon
which a provincial board may declare any municipal resolution, ordinance, or order
invalid is when such resolution, ordinance, or order is “beyond the powers
conferred upon the council or president making the same.”
 This was not the case in the case at bar as the Sangguniang Panlalawigan merely
stated that there are other available lands for the purpose sought, they did not
even bother to declare the SB resolution as invalid. Hence, the expropriation case
is valid

Page 84 of 166
CASE NO. 39
FLAVIANO MEJIA VS. PEDRO U. BALOLONG
81 Phil. 486
September 16, 1948

Facts:
This is an action of quo warranto instituted by the petitioners, Flaviano Mejia, Teofilo P.
Guadiz, Ruperto Z. Tandoc and Policronio de Venecia against the respondents, Pedro U.
Balolong, Ricardo Villamil, Toribio Quimosing and Crisologo Zarate on the ground that
the appointments of the latter by the President as councilors of the City of Dagupan were
null and void, and therefore they are unlawfully holding their offices, and that the former
are entitled to said offices because they were elected as such in the general election for
provincial, municipal, and city officials on November, 1947.

Issue:
Whether to effect Act No. 170 or Executive Order 96 in the appointment of officials of
Dagupan.

Ruling:
The appointments of the respondents effected on December 30, 1947, are null and void.
The validity of the appointment of the respondents as councilors of the City of Dagupan
by the President of the Philippines depends upon whether the City of Dagupan was
created and came into existence on June 20, 1947, the date Act No. 170 became effective,
or on January 1, 1948, when the city government was organized by Executive Order No.
96.The City of Dagupan created by Act 170 came into existence as a legal entity or a public
corporation upon the approval of said Act, on June 20, 1947. Because a statute like Act
No. 170 is to take effect upon its approval, it is operative from the exact instance upon its
approval or becoming a law. The City of Dagupan created by Act No. 170 came into
existence as a legal entity or a public corporation upon the approval of Act No. 170, on
June 20, 1947, because a statute which, like Act No. 170, is to take effect upon its approval,
is operative from the exact instance upon its approval or becoming a law. The date of the
organization of the city government of Dagupan which the President is authorized to fix
by the provisions of section 88, is not and cannot be the date of the creation of the city,
not only because the City of Dagupan came into existence on the same date June 20 in
which Act No. 170 creating the said city became effective, but because what was to be
organized, according to said section 88, is the city government, and not the city as an
entity, and the word “organize” means “to prepare (the city) for transaction of business,
as assembly, by choosing officers, committees, etc.” It is obvious that to create a public
corporation or city is one thing ‘and to organize the government thereof is another. A
public corporation is created and comes into existence from the moment the law or
charter that creates it becomes effective. Since the election of the members of the

Page 85 of 166
Municipal Board of the City of Dagupan created on June 20, 1947, was to take and took
place at the general election held on November11, 1947, and the President of the
Philippines was empowered by section 88 of Act 170 to appoint those members only if
the organization of the city government had taken place pending or before the said
election.

Page 86 of 166
CASE NO. 40
MUNICIPALITY OF JIMENEZ VS. BAZ
G.R. No. 105746
December 2, 1996

Facts:
By virtue of Executive Order No. 258, the Municipality of Sicacaban was creadted, pursuant
to 68 of the revised Administrative Code of 1917.

By virtue of Municipal Council Resolution No. 171, dated November 22, 1988, Sinacaban
laid claim to a portion of Barrio Tabo-o and to Barrios Macabayao, Adorable, Sinara, Baja,
and Sinara Alto, based on the technical description in E.O. No. 258. The claim was filed
with the Provincial Board of Misamis Occidental against the Municipality of Jimenez.

While conceding that the disputed area is part of Sinacaban, the Municipality of Jimenez,
in its answer, nonetheless asserted jurisdiction on the basis of an agreement it had with
the Municipality of Sinacaban. This agreement was approved by the Provincial Board of
Misamis Occidental in its Resolution No. 77 dated February 18, 1950, fixed the common
boundary of Sinacaban and Jimenez.

On October 11, 1989, the Provincial Board declared the disputed area to be part of
Sinacaban. It held that the previous resolution approving the agreement between the
parties was void since the Board had no power to alter the boundaries of Sinacaban as
fixed in E.O. 258, that power being vested in Congress pursuant to the Constitution and
the LGC of 1983 (BP 337), Sec. 134. The Provincial Board denied the motion of Jimenez
seeking reconsideration.

On March 20, 1990, Jimenez filed a petition for certiorari, prohibition, and mandamus in
the RTC of Oroquieta City, Branch 14 against Sinacaban, the Province of Misamis
Occidental and its Provincial Board, the Commission on Audit, the Departments of Local
Government, Budget and Management, and the Executive Secretary alleging that the
power to create municipalities is legislative in nature and since Sinacaban is created by
virtue of Executive Order does not have a legal personality.

Issues:
A. Whether or not Sinacaban has legal personality to file a claim.
B. If Sinacaban has legal personality, whether it is the boundary provided for in E.O. 258
or in Resolution No. 77 of the Provincial board of Misamis Occidental which should be
used as basis for adjudicating its territorial claim.

Ruling:

Page 87 of 166
A. The principal basis for the view that Sinacaban was not validly created as a municipal
corporation is the ruling in Pelaez vs. Auditor General that the creation of municipal
corporations is essentially a legislative matter and therefore the President was without
power to create by executive order the Municipality of Sinacaban. However, where a
municipality created as such by executive order is later impliedly recognized and its
acts are accorded legal validity, its creation can no longer be questioned.

A municipality has been conferred the status of at least a de facto municipal


corporation where its legal existence has been recognized and acquiesced publicly
and officially.

A quo warranto suit against a corporation for forfeiture of its charter must be
commenced within 5 years from the act complained of was done or committed as
provided for under Rule 66, section 16 of Rules of Court. The validity of E.O. No. 258
creating it had never been questioned. Created in 1949, it was only 40 years later that
its existence was questioned and only because it had laid claim to an area that is
apparently desired for its revenue. The State and even the Municipality of Jimenez
itself has recognized Sinacaban’s corporate existence. Sinacaban is constituted part of
a municipal circuit for purposes of the establishment of MTCs in the country. Jimenez
had earlier recognized Sinacaban in 1950 by entering into an agreement with it
regarding their common boundary.

The Municipality of Sinacaban attained a de jure status by virtue of the Ordinance


appended to the 1987 Constitution, apportioning legislative districts throughout the
country, which considered Sinacaban part of the Second District of Misamis
Occidental. Sec. 442(d) of the Local Government Code of 1991 must be deemed
to have cured any defect in the creation of Sinacaban since it states that:

“Municipalities existing as of the date of the effectivity of this Code shall continue to
exist and operate as such. Existing municipal districts organized pursuant to
presidential issuances/executive orders and which have their respective set of
municipal officials holding office at the time of the effectivity of this Code shall
henceforth be regular municipalities.”

B. E.O. No. 258 does not say that Sinacaban comprises only the barrios therein
mentioned. What it says is that “Sinacaban contains” those barrios. The reason for this
is that the technical description, containing the metes and bounds of a municipality’s
territory, is controlling. The trial court correctly ordered a relocation survey as the only
means of determining the boundaries of the municipality & consequently to which
municipality the barangays in question belong.

Page 88 of 166
The question of whether the Provincial Board had authority to approve agreement or
whether it had power to declare certain barrios part of one or other municipalities, the
court held that, it has no power if the effect would be to amend the areas described
in EO No. 258.

Sinacaban is likewise not subject to the plebiscite requirement since it attained de


facto status at the time the 1987 Constitution took effect. The plebiscite requirement
for the creation of municipalities applies only to new municipalities created for the first
time under the Constitution – it cannot be applied to municipalities created before.

Page 89 of 166
CASE NO. 41
TERRADO VS. COURT OF APPEALS
131 SCRA 373
August 24, 1984

Facts:
Pursuant to Act No. 4041 of the Philippine Legislature the Fisheries situated in the locality
known as Mangabul, Bayambang, Pangasinan, recently declared by the courts as public
land was reserved and the usufruct thereof ceded to the municipality of Bayambang,
Province of Pangasinan, to be used or disposed of in accordance with the general
municipal law relative to the letting of fisheries in municipal waters.

Provided however that the timber and other forest products therein shall be placed under
the administration and control of the forest service. Provided further, that the cession shall
not be interpreted as limiting the power of the Secretary of Agriculture and Natural
Resources to prescribe rules and regulations for the protection of game birds, mammals
or fish within the area ceded to the municipality of Bayambang.

On May 15, 1974, the Sanggunian Bayan of Bayambang, Pangasinan passed Resolution
No. 35 enacting Ordinance NO. 8, series of 1974, establishing the Bayambang Fishery and
Hunting Park and Municipal Water Shed embracing all the vast area of the Mangabul
Fisheries consisting of about 2,061 hectares with 19 fishponds and not less than 1,500
hectares of watershed area.

In the said ordinance, the municipality designated appointed and constituted private
respondent Geruncio Lacuesta as Manager-Administrator for a period of 25 years,
renewable for another 25 years, under the condition that said respondent shall pay the
municipality. a sum equivalent to 10% of the annual gross income that may be derived
from the sale of forest products, wild game and fish, which amount shall not be less than
P200,000.00 annually. He was further required to post a bond in the amount of
P200,000.00 to guaranty payment of the 10% due the municipality.

Municipal Ordinance No. 8 was approved by the Provincial Board of Pangasinan and
thereafter was forwarded to the then Secretary of Agriculture and Natural Resources for
approval pursuant to the provisions of the Fisheries Act, Act No. 4003.

Later, the Secretary disapproved the Ordinance because it grants fishery privileges to
respondent Lacuesta without the benefit of competitive public hearing in contravention
of the provisions of Act 4003 as amended.

Page 90 of 166
Respondent Lacuesta interposed an appeal from the disapproval by the Secretary of
Agriculture and Natural Resources to the Office of the President but the appeal was
withdrawn by said respondent in his letter dated July 14, 1977.

The Municipality then informed respondent Lacuesta of the disapproval of the Ordinance
by the Secretary of Agriculture and Natural Resources and directed him to refrain and
desist from acting as Administrator-Manager under the contract but the latter refused
and insisted in maintaining possession of the fisheries.

Despite such refusal, the Sanggunian Bayan of Bayambang, Pangasinan passed Resolution
No. 31, series of 1977, resolving to advertise for public bidding all fisheries at the
Mangabul area for four years and to direct the Municipal Treasurer to prepare the
necessary notices of public bidding, and accordingly, the Municipal Mayor and the
Municipal Treasurer caused to issue a Notice of Public Bidding.

Among the winning bidders were the petitioners herein, the spouses Lydia Terrado and
Martin Rosario and Domingo Fernandez who were immediately placed in possession of
the Mangabul fisheries as of July 6, 1977.

Private respondent Geruncio Lacuesta immediately filed a petition for prohibition and
mandamus with damages with the CFI of Pangasinan against the Municipal Mayor, the
Municipal Treasurer, the Sanggunian Bayan and the members thereof, praying that the
respondent municipal officials named therein be prohibited from executing any contract
of lease with the winning bidders and from enforcing Resolution No. 31, series of 1977,
and further asked that a temporary restraining order be issued against said respondent
officials from performing the acts enjoined.

The situation became serious as the Sanggunian Bayan passed Resolution No. 34, series
of 1977 "requesting the assistance from the Department of Natural Resources, the
Philippine Constabulary, Department of Justice, the Provincial Fiscal, the Provincial
Governor and other agencies, for them to enjoin respondent from disturbing and
interfering with the administration by the Municipality of Mangabul Fisheries and other
areas."

Issue:
Whether or not the Municipal Order granting Lacuesta administration is valid?

Ruling:
No. It granted the administration without the benefit of public bidding.

Page 91 of 166
The Ordinance is clearly against the provisions of the law for it granted exclusive fishery
privileges to the private respondent without benefit of public bidding. Under the Fisheries
Act, the Municipality may not delegate to a private individual as Manager-Administrator
to "use or dispose of the fisheries portion in accordance with the general law on municipal
waters" nor to charge foes for fishing and hunting in the park, much less sell forest
products, wild games and fish from the area.

Neither can the Municipality grant the exclusive privilege of fishing for a period more than
five (5) years, whereas in the instant case, the period granted the Manager-Administrator
was for twenty-five (25) years, renewable for another twenty-five years.

Moreover, under the specific provision of Act No. 4041, there is the proviso that the timber
and other forest products therein shall be placed under the administration and control of
the forest service so that insofar as the ordinance relates to the timber and other forest
products and the reforestation of the timberland portions indicated in Plan Ipd-92
including the powers, duties and responsibilities of the Manager-Administrator affecting
the forestry portions are violative of Act No. 4041.

The Ordinance is illegal and contrary to law, the contract executed in pursuance thereto
is consequently illegal. Acts executed against the provisions of mandatory or prohibitory
laws shall be void, except when the law itself authorizes their validity.

Since Ordinance No. 8 granted fishery privileges exclusively to the private respondent
without benefit of public bidding and for a period exceeding five (5) years, the said
ordinance and the contract of management executed in accordance therewith were null
and void ab initio, such that the failure of the Secretary of Agriculture & Natural Resources
to disapprove the same within 30 days from its submission does not render validity to the
illegal legislation of the municipal council nor to the contract executed under the same.

Essentially, the contract of management and administration between the Municipality and
Lacuesta is one of agency whereby a person binds himself to render some service or to
do something in representation or on behalf of another, with the consent or authority of
the latter. Here in the case at bar, Lacuesta bound himself as Manager-Administrator of
the Bayambang Fishing & Hunting Park and Municipal Watershed to render service or
perform duties and responsibilities in representation or on behalf of the Municipality of
Bayambang, with the consent or authority of the latter pursuant to Ordinance No. 8. Under
Article 1919, New Civil Code, agency is extinguished by the death of the agent. His rights
and obligations arising from the contract are not transmittable to his heirs.

“We hereby pronounce the nullity of Ordinance No. 8, series of 1974 of the Municipal
Council of Bayambang, Pangasinan and the contract of management and supervision

Page 92 of 166
executed between the Municipality of Bayambang and Geruncio Lacuesta as Manager-
Administrator of the Bayambang Fishery & Hunting Park and Municipal Watershed”

Since Ordinance No. 8 and the contract of management and supervision are both null and
void, the Alias Writ of Execution and Possession dated November 6, 1981 and the Order
of October 8, 1982 for the issuance of writ of execution and possession to place and
restore possession of the Mangabul Fisheries, of portions thereof or fisheries therein to
Geruncio Lacuesta, his agents, men and/or representatives under the said contract and by
virtue of the ordinance are, including the writ also issued without legal force and effect.

Page 93 of 166
CASE NO. 42
LAGUNA LAKE DEVELOPMENT AUTHORITY vs. COURT OF APPEALS
G.R. Nos. 120865-71
December 7, 1995
Facts:
The Laguna Lake Development Authority (LLDA) was created through RA No. 4850 in
order to execute the policy towards environmental protection and sustainable
development so as to accelerate the development and balanced growth of the Laguna
Lake area and the surrounding provinces and towns. Upon implementation of RA 7160
(Local Government Code of 1991), the municipalities assumed exclusive jurisdiction &
authority to issue fishing privileges within their municipal waters since Sec.149 thereof
provides: “Municipal corporations shall have the authority to grant fishery privileges in
the municipal waters and impose rental fees or charges therefore…” Big fishpen operators
took advantage of the occasion to establish fishpens & fish cages to the consternation of
the LLDA. The implementation of separate independent policies in fish cages & fish pen
operation and the indiscriminate grant of fishpen permits by the lakeshore municipalities
have saturated the lake with fishpens, thereby aggravating the current environmental
problems and ecological stress of Laguna Lake. A month later, the LLDA sent notices
advising the owners of the illegally constructed fishpens, fishcages and other aqua-culture
structures advising them to dismantle their respective structures otherwise demolition
shall be effected. The issue now arose after there was a conflict as to who has the authority
to file an action to eject the informal settlers in the area.

Issue:
Whether or not LLDA (Laguna Lake Development Authority) should exercise jurisdiction
over the Laguna Lake insofar as the issuance of permits for fishery privileges is concerned.

Ruling:
Yes, LLDA has jurisdiction over such matters because the charter of the LLDA prevails over
the Local Government Code of 1991. The said charter constitutes a special law, while the
latter is a general law. The Local Government Code of 1991, has not repealed the
provisions of the charter of the Laguna Lake Development Authority, Republic Act No.
4850, as amended. Thus, the Authority has the exclusive jurisdiction to issue permits for
the enjoyment of fishery privileges in Laguna de Bay to the exclusion of municipalities
situated therein and the authority to exercise such powers as are by its charter vested on
it. In addition, the charter of the LLDA embodies a valid exercise of police power for the
purpose of protecting and developing the Laguna Lake region, as opposed to the Local
Government Code, which grants powers to municipalities to issue fishing permits for
revenue purposes. Thus, it has to be concluded that the charter of the LLDA should prevail
over the Local Government Code of 1991 on matters affecting Laguna de Bay.

Page 94 of 166
CASE NO. 43
PILAPIL VS. COURT OF APPEALS
G.R. No. 97619
November 26, 1992

Facts:
Spouses Pilapil own a parcel of land in Bahak, Poblacion, Liloan, Cebu. Spouses Colomida
on the other hand bought a parcel of land located in Bahak. The Colomidas claim that
they had acquired from Sesenando Longkit a road right of way which leads towards the
National Road. However, this road right of way ends at that portion of the property of
the Pilapils where a “camino vicinal” (barrio road) exists all the way to the said National
Road. The Colonidas tried to improve the road of camino vecinal for the convenience of
the public, but the Pilapils harassed and threatened them with bodily harm from making
said improvement. The Pilapil also threatened to fence off the camino vecinal. Thus, the
Colomidas filed a complaint against them. The Pilapils denied the existence of the camino
vecinal. They presented several witnesses among them was Engineer Epifanio Jordan,
Municipal Planning and Development Coordinator of Liloan. Engr Jordan testified on
Liloan’s Urban Land Use Plan or Zoning Map which he prepared upon the instruction of
Mayor Cesar Butai which was approved by Sangguniang Bayan of Liloan. Per the said
plan, the camino vecinal does not traverse but runs along the side of the Pilapil property.
The Colonidas on the other hand relied on old-timers as witnesses such as Florentino
Pepito, who attested to the existence of the camino vecinal and its availability to the
general public since time immemorial.

Issue:
Whether or not the Municipality of Liloan’s camino vecinal should traverse the property
of the Pilapils.

Ruling:
No. A camino vecinal is a municipal road and property for public use. Pursuant to the
powers of the Local Government Unit, the Municipality of Liloan had the unassailable
authority to: (a) prepare and adopt a land use map, (b) promulgate a zoning ordinance
which may consider, among other things the municipal roads to be constructed,
maintained, improved or repaired, and (c) close municipal road. It is beyond dispute that
the establishment, closure, or abandonment of the camino vecinal is the sole prerogative
of the Municipality of Liloan with respect to the said camino vecinal in Sitio Bahak must
prevail. Moreover, the Municipality of Liloan through the Sangguniang Bayan, approved
the Urban Land Use Plan, the plan was duly signed by the Municipal Mayor. By doing so,
the said legislative body determined among others, the location of the caminal vecinal in
Sitio Bahak.

Page 95 of 166
CASE NO. 44
CABRERA VS. COURT OF APPEALS
G.R. No. 78673
August 21, 1987

Facts:
The Provincial Board of Catanduanes adopted Resolution No. 158 (Closing the old road
leading to the new Capitol Building and giving owners of properties traversed by the new
road an area form the old road). Pursuant thereto, Deeds of Exchange were executed
under which the Province conveyed to Remedios R. Bagadiong, Fredeswindo F. Alcala,
Elena S. Latorre, Baldomero Tolentino, Eulogia T.Alejandro, Angeles S. Vargas, and Juan S.
Reyes portions of the closed road in exchange for their own respective properties,
on which was subsequently laid a new concrete road leading to the Capitol Building.
Learning about Resolution 158, the petitioner filed a complaint with the CFI
of Catanduanes for"Restoration of Public Road and/or Abatement of Nuisance,
Annulment of Resolutions and Documents with Damages." He alleged that the land
fronting his house was a public road owned by the Province in its governmental capacity
and therefore beyond the commerce of man. He contended that Resolution No. 158and
the deeds of exchange were invalid, as so too was the closure of the road. The judge
sustained the authority of the provincial board to enact said Resolution. The CA affirmed
and found that the road was not a public road but just a trail. Also, pursuant to RA
5185, municipal authorities, subject to the approval of the Provincial Board, can close
thoroughfares pursuant to Sec 2246of the Revised Administrative Code. Petitioner insists
that Sec. 2246 is not applicable because Resolution No. 158 is not an order for the closure
of the road in question but an authority to barter or exchange it with private properties.
He maintains that the public road was owned by the province in its governmental capacity
and, without a prior order of closure, could not be the subject of a barter. Control over
public roads, he insists, is with Congress and not with the provincial board.

Issue:
Whether or not the Provincial Board can validly enact said resolution

Ruling:
Yes. Resolution 158 clearly says that it is "hereby resolved to close the old road." The
closure is as plain as day except that the petitioner, with the blindness of those who will
not see, refuses to acknowledge it. The Court has little patience with such puerile
arguments. They border dangerously on a trifling with the administration of justice and
can only prejudice the pleader's cause. The authority of the provincial board to close that
road and use or convey it for other purposes is derived from the following provisions of
Republic Act No. 5185 in relation to Section 2246 of the Revised Administrative Code: It
sustained the subsequent sale of the land as being in accordance not only with the charter

Page 96 of 166
but also with Article 422 of the Civil Code, which provides: "Property of public dominion,
when no longer intended for public use or for public service, shall form part of the
patrimonial property of the State."

While it is true that the cases dealt with city councils and not the provincial board, there
is no reason for not applying the doctrine announced therein to the provincial board in
connection with the closure of provincial roads. The provincial board has, after all, the
duty of maintaining such roads for the comfort and convenience of the inhabitants of the
province. Moreover, this authority is inferable from the grant by the national legislature
of the funds to the Province for the construction of provincial roads. The lower court
found the petitioner's allegation of injury and prejudice to be without basis because he
had "easy access anyway to the national road, for in fact the vehicles used by the Court
and the parties during the ocular inspection easily passed and used it, reaching beyond
plaintiff's house." However, the CA ruled that the he "was prejudiced by the closure of the
road which formerly fronted his house. He and his family were undoubtedly
inconvenienced by the loss of access to their place of residence for which we believe they
should be compensated." On this issue, the governing principle was laid down in Favis v
City of Baguio, thus: The general rule is that one whose property does not abut on the closed
section of a street has no right to compensation for the closing or vacation of the street, if he still
has reasonable access to the general system of streets. The circumstances in some cases may be
such as to give a right to damages to a property owner, even though his property does not abut
on the closed section. But to warrant recovery in any such case the property owner must show that
the situation is such that he has sustained special damages differing in kind, and not merely in
degree, from those sustained by the public generally.

Petitioner is not entitled to damages because the injury he has incurred, such as it is, is
the price he and others like him must pay for the welfare of the entire community. This is
not a case where his property has been expropriated and he is entitled to just
compensation. The construction of the new road was undertaken under the general
welfare clause. As the trial judge acutely observed, whatever inconvenience the petitioner
has suffered "pales in significance compared to the greater convenience the new road,
which is wide and concrete, straight to the veterans fountain and down to the pier, has
been giving to the public, plus the fact that the new road adds beauty and color not only
to the town of Virac but also to the whole province of Catanduanes." For the enjoyment
of those benefits, every individual in the province, including the petitioner, must
be prepared to give his share.

Page 97 of 166
CASE NO. 45
CRUZ VS. COURT OF APPEALS
G.R. No. L-44178
August 21, 1987

Facts:
On May 26, 1970, the management of Padre Rada Market in Tondo, Manila represented
by Ricardo Cruz wrote Mayor Villegas that the management is withdrawing three-fourths
of the area of the market "from the direct supervision and control of the City Treasurer's
Office effective on June 15, 1970, and from said date the withdrawn portion shall cease to
function and operate as a public market." The respondent-vendors, who were likewise
notified of such withdrawal, protested such move. After several exchanges of referrals,
indorsements, and communications, Mayor Villegas allowed the withdrawal in the light of
the Court of Appeals' decision upholding the right of the operators of the Market to
withdraw their property from its use as a public market stating, among others, that
approval for the withdrawal by the City of Manila is not even necessary. Motions for
reconsiderations were denied. Thus, the private respondents instituted a civil case against
Mayor Villegas, Cruz and others.

The lower Court ruled in favor of the latter, however, the Court of Appeals reversed the
decision and denied the withdrawal by the Manila City Mayor of government-control and
supervision "until legal conditions and equitable justification for the withdrawal by private
parties obtain."

Issue:
Whether or not the City Mayor may validly withdraw Padre Rada Market as a public
market.

Held:
No. The Padre Rada Market is a public market and as such should be subject to the local
government's supervision and control. Its conversion into a private market or its closure
must follow the procedures laid down by law.

The Municipal Board of Manila with the approval of then Mayor Manuel de la Fuente
pursuant to its legislative power authorized the disputed premises to be operated as a
public market under its direct control and supervision as embodied in Resolution No. 230,
amended by Resolution No. 406, both series of 1949. Thus, Mayor Villegas had no legal
authority by himself to allow the petitioner to withdraw the major portion of Padre Rada
Market from its use as a public market. It must be subject to the same joint action of the
Board and the Mayor.

Page 98 of 166
CASE NO. 46
CEBU OXYGEN VS. BERCILLES
G.R. No. L-40474
August 29, 1975

Facts:
Cebu Oygen and Acetylene assailed the judgment of Judge Pascual Bercilles dismissing
their application to register the land once part of M. Borces Street, Mabolo Cebu. It upheld
the argument of Assistant Provincial Fiscal Jose Espeleta that the land belongs to the
government, not an abandoned road or land, and not alienable. On September 23, 1968,
the City Council of Cebu, through Resolution No. 2193, approved on October 03,1968,
declared the terminal portion of Borces Street, Mabolo, Cebu City, as an abandoned road,
the same not being included in the City development plan. Subsequently, on December
19, 1968, the City Council of Cebu passed another resolution (Resolution No.2755)
authorizing the Acting City Mayor to sell the land through a public bidding. In pursuance
to the said resolution, the lot was awarded to Cebu Oygen and Acetylene, being the
highest bidder. Subsequently, a deed of absolute sale was executed on March 03, 1969 by
the City of Cebu via its Acting City Mayor to Cebu Oxygen and Acetylene for a total
consideration of P10,800.00.

Issue:
Whether Cebu Oygen and Acetylene can own and register the land in their name?

Held:
Yes. Since sec. 31 of RA No.357 (The City Charter of Cebu) authorized the Mayor “to
close any city road, street or alley, boulevard, avenue, park or square. Also, under Art.
422 of the New Civil Code, property of public dominion, when no longer intended for
public use or for public service, shall form part of the patrimonial property of the State.
Thus, property withdrawn from public servitude may be used or conveyed for any purpose
for which other real property belonging to the City may be lawfully used or conveyed, the
petitioners Cebu Oxygen and Acetylene acquired valid ownership on the abandoned part
of M Borces Street, Mabolo, Cebu City since the land is no longer under public dominion
or public use.

Page 99 of 166
CASE NO. 47
MUNICIPALITY OF SAN JOAQUIN VS. SIVA, ET AL.
GR No. L-19870
March 18, 1967

Facts:
Municipality of San Joaquin, seeks the reversal of a decision of the Court of First Instance
of Iloilo dismissing the former’s petition for prohibition, contesting the legality of
Executive Order No. 436 of the President of the Philippines, dated July 10, 1961, creating
the municipality of Lawigan out of twenty-one (21) barrios theretofore forming part of
said municipality of San Joaquin. Respondents-appellees are the persons appointed by
the President as mayor, vicemayor and councilors of Lawigan, who are sought to be
restrained from performing their functions as such, upon the ground that Section 68 of
the Revised Administrative Code, on which said Executive Order is based, constitutes an
undue delegation of legislative powers, and, hence, unconstitutional. The lower court,
however, held otherwise.

Issue:
Whether Executive Order No. 436 constitutes an undue delegation of legislative power.

Ruling:
Executive Order No. 436 of the President of the Philippines, creating the municipality of
Lawigan out of twenty-one (21) barrios theretofore forming part of the municipality of
San Joaquin, is void ab initio, on the ground that Section 68 of the Revised Administrative
Code, on which said Executive Order is based, constitutes an undue delegation of
legislative powers to the President of the Philippines. Hence, it is unconstitutional. This
issue has been settled in Pelaez vs Auditor General G.R, No. L-23825 which states that
Section 68 of the Revised Administrative Code, insofar as it grants to the President the
power to create municipalities, does not meet the well-settled requirements for a valid
delegation of the power to fix the details in the enforcement of a law. It does not enunciate
any policy to be carried out or implemented by the President. If the validity of said
delegation of powers, made in Section 68 of the Revised Administrative Code, were
upheld. there would no longer be any legal impediment to a statutory grant of authority
to the President to do anything which, in his opinion, may be required by public welfare
or public interest. Such grant of authority would be a virtual abdication of the powers of

Page 100 of 166


Congress in favor of the Executive, and would bring about a total collapse of the
democratic system established by the Constitution.

Page 101 of 166


CASE NO. 48
TOBIAS VS. ABALOS
G.R. No. 114783
December 8, 1994

Facts:
Invoking their rights as taxpayers and as residents of Mandaluyong, petitioners assail the
constitutionality of Republic Act No. 7675, otherwise known as “An Act Converting the
Municipality of Mandaluyong into a Highly Urbanized City to be Known as the City of
Mandaluyong.” Prior to the enactment of the assailed statute, the municipalities of
Mandaluyong and San Juan belonged to only one legislative district. Hon. Ronaldo
Zamora, the incumbent congressional representative of this legislative district, sponsored
the bill which eventually became R.A. No. 7675 which the President Ramos signed into
law. Pursuant to the Local Government Code of 1991, a plebiscite was held on April 10,
1994. The people of Mandaluyong were asked whether they approved of the conversion
of the Municipality of Mandaluyong into a highly urbanized city as provided under R.A.
No. 7675. The turnout at the plebiscite was only 14.41% of the voting population.
Nevertheless, 18,621 voted “yes” whereas 7,911 voted “no.” By virtue of these results, R.A.
No. 7675 was deemed ratified and in effect.

Issues:
A. Whether statutory conversion of Mandaluyong into a highly urbanized city indubitably
complies with the “one city-one representative” proviso in the Constitution
B. Whether the creation of a separate congressional district for Mandaluyong is not a
subject separate and distinct from the subject of its conversion into a highly urbanized
city
C. Whether the present composition of Congress may be increased

Ruling:
A. Yes, we agree with the observation of the Solicitor General that the statutory
conversion of Mandaluyong into a highly urbanized city with a population of not less
than two hundred fifty thousand indubitably ordains compliance with the “one city-
one representative” proviso in the Constitution: “x x x Each city with a population of at
least two hundred fifty thousand, or each province, shall have at least one
representative” (Article VI, Section 5(3), Constitution). Hence, it is in compliance with
the aforestated constitutional mandate that the creation of a separate congressional

Page 102 of 166


district for the City of Mandaluyong is decreed under Article VIII, Section 49 of R.A.
No. 7675.

B. Yes, Contrary to petitioners’ assertion, the creation of a separate congressional district


for Mandaluyong is not a subject separate and distinct from the subject of its
conversion into a highly urbanized city but is a natural and logical consequence of its
conversion into a highly urbanized city. Verily, the title of R.A. No. 7675, “An Act
Converting the Municipality of Mandaluyong Into a Highly Urbanized City of Manda-
luyong” necessarily includes and contemplates the subject treated under Section 49
regarding the creation of a separate congressional district for Mandaluyong.
Moreover, a liberal construction of the “one title-one subject” rule has been invariably
adopted by this court so as not to cripple or impede legislation. Thus, in Sumulong v.
Comelec (73 Phil. 288 [1941]), we ruled that the constitutional requirement as now
expressed in Article VI, Section 26(1) “should be given a practical rather than a
technical construction. It should be sufficient compliance with such requirement if the
title expresses the general subject and all the provisions are germane to that general
subject.”

C. Yes, if the Congress itself so mandates through legislative enactment. As to the


contention that the assailed law violates the present limit on the number of
representatives as set forth in the Constitution, a reading of the applicable provision,
Article VI, Section 5 (1), as aforequoted, shows that the present limit of 250 members
is not absolute. The Constitution clearly provides that the House of Representatives
shall be composed of not more than 250 members, “unless otherwise provided by
law.” The inescapable import of the latter clause is that the present composition of
Congress may be increased, if Congress itself so mandates through a legislative
enactment. Therefore, the increase in congressional representation mandated by R.A.
No. 7675 is not unconstitutional

Page 103 of 166


CASE NO. 49
MATALIN COCONUT CO., INC. VS. MUNICIPAL COUNCIL OF MALABANG
G.R. No. L-28138
August 13, 1986

Facts:
On August 24, 1966, the Municipal Council of Malabang, Lanao del Sur, invoking the
authority of Section 2 of Republic Act No. 2264, otherwise known as the Local Autonomy
Act, enacted Municipal Ordinance No. 45-46, entitled “AN ORDINANCE IMPOSING A
POLICE INSPECTION FEE OF P.30 PER SACK OF CASSAVA STARCH PRODUCED AND
SHIPPED OUT OF THE MUNICIPALITY OF MALABANG AND IMPOSING PENALTIES FOR
VIOLATIONS THEREOF.” The ordinance made it unlawful for any person, company or
group of persons “to ship out of the Municipality of Malabang, cassava starch or flour
without paying to the Municipal Treasurer or his authorized representatives the
corresponding fee fixed by (the) ordinance.” The validity of the ordinance was challenged
by the Matalin Coconut, Inc. in a petition for declaratory relief filed with the then Court of
First Instance of Lanao del Sur against the Municipal Council, the Municipal Mayor and
the Municipal Treasurer of Malabang, Lanao del Sur. Alleging among others that the
ordinance is not only ultra vires, being violative of Republic Act No. 2264, but also
unreasonable, oppressive and confiscatory, the petitioner prayed that the ordinance be
declared null and void ab initio

Issue:
Whether or not Ordinance No. 45-66 enacted by respondent Municipal Council of
Malabang, Lanao del Sur, is valid

Ruling:
No, as correctly held by the trial court, the so-called “police inspection fee” levied by the
ordinance is “unjust and unreasonable.”

It has been proven that the only service rendered by the Municipality of Malabang, by
way of inspection, is for the policeman to verify from the driver of the trucks of the
petitioner passing by at the police checkpoint the number of bags loaded per trip which
are to be shipped out of the municipality based on the trip tickets for the purpose of
computing the total amount of tax to be collect (sic) and for no other purpose. The
pretention of respondents that the police, aside from counting the number of bags

Page 104 of 166


shipped out, is also inspecting the cassava flour starch contained in the bags to find out
if the said cassava flour starch is fit for human consumption could not be given credence
by the Court because, aside from the fact that said purpose is not so stated in the
ordinance in question, the policemen of said municipality are not competent to determine
if the cassava flour starch are fit for human consumption. The further pretention of
respondents that the trucks of the petitioner hauling the bags of cassava flour starch from
the mill to the bodega at the beach of Malabang are escorted by a policeman from the
police checkpoint to the beach for the purpose of protecting the truck and its cargoes
from molestation by undesirable elements could not also be given credence by the Court
because it has been shown, beyond doubt, that the petitioner has not asked for the said
police protection because there has been no occasion where its trucks have been
molested, even for once, by bad elements from the police checkpoint to the bodega at
the beach, it is solely for the purpose of verifying the correct number of bags of cassava
flour starch loaded on the trucks of the petitioner as stated in the trip tickets, when
unloaded at its bodega at the beach. The imposition, therefore, of a police inspection fee
of P. 30 per bag, imposed by said ordinance is unjust and unreasonable.

Page 105 of 166


CASE NO. 50
LEOVILO C. AGUSTIN VS. HON. ROMEO F. EDU
G.R. No. L-49112
February 02, 1979

Facts:
This is a petition for prohibition and/or Mandatory Injunction on the Letter of Intent (LOI)
No.229 of the president which recommended the enactment of local legislation for the
installation of road safety sign and devices. The assailed letter was made Dec 2,1974 based
on a study by the Department that majority of the fatal or serious accidents In Land
Transportation is the presence of disabled stalled or parked motor vehicles along streets
or highways without an appropriate early warning device to signal approaching vehicle of
their presence which was fully recognized by the Vienna Convention on Road Signs and
Signals as well as the UN in 1968 I which the Philippines was a signatory.

The petitioner setting forth that he is an owner of a Volkswagen Bettle Car Model 13035
and is already equipped with needed early warning device therefore such legislation is
not necessary. He believes that LOI 229 violates the provision and the delegation of police
power. He finds it oppressive unreasonable, arbitrary and unconstitutional. It is also one-
sided, onerous and patently illegal and immoral because it will make manufacturers and
dealers instant millionaires at the expense of car owners who are required to buy sets of
early warning devices. This led to such resolution.

Issue:
Whether or Not LOI 229 violates the constitutional provision of due process , undue
delegation of police power and at the same time oppressive, arbitrary, confisticatory,
onerous, immoral, unreasonable and illegal.

Ruling:
The decision of the court on the petition is to dismissed based on the following grounds:

1. The LOI was issued on the exercise of Police Power. It is the state authority to enact
legislation that may interfere with personal liberty and property in order to
promote general welfare. The purpose of LOI 229 is to promote public Safety by
giving emphasis on safe transit and avoid obstruction of roads and streets
designated as national roads.

2. The petitioner failed to nullify the LoI and the implementing rules and regulation
for the president had with him the necessary statistical information and data which
provides factual foundation in the implementation of such law compared with
allegations of the petitioner.

Page 106 of 166


3. It is not oppressive in nature for being a signatory to the 1968 Vienna Convention
which mandates to make visible by 400 meters disabled and stationary vehicle by
providing a reflectorized rectangular early warning device this does not obstruct
and endangered traffic. This is an International Law that should also implemented
in our local legislation.

4. As to compel motor vehicle owner to purchase early warning device, procuring or


obtaining is left on the will and ingenuity of the vehicle owner. It is conclusive to
say that it will make manufacturers instant millionaires out of it.

5. The alleged infringement of the fundamental principles of non-delegation of police


power is without support of well settled doctrines. The purpose of the Law is “Safe
Transit upon Roads and Subordinate Legislation”.

Page 107 of 166


CASE NO. 51
JEJOMAR BINAY VS. DOMINGO
G.R. No. 92389
September 11, 1991

Facts:
This is a petition for Special Civil Action of the Ordinance Resolution No.60 and setting
aside the COA’s decision as null and void.
On Sept 27,1988 petitioner which is the Municipality of Makati through its council
approved the recreation of Resolution no.60 which is the ratification of the on-going
Burial Assistance Program initiated by the Office of the Mayor extending Php 500.00
financial assistance to berieved families who’s family gross monthly income does not
exceed Php 2,000. The amount of disbursement totalling Php 400,000. This was refered
to COA for expected allowance in audit but was disapproved.
Two letters for reconsideration were denied under COA’s decision of 1159 on the
following manner:
1. Subject Resolution No.60 of 1988 intended disbursements fall between the
principles of “police power and Parens Patrice.
2. The MetroManila Commission under Certification June 1989 had appropriated Php
400,000 to implement the ID resolution and only the COA’s approval at question.

Issue:
Whether or Not Resolution No.60 reenacted under Resolution no.243 of the Municipality
of Makati is a valid exercise of Police Power under the General Welfare Clause.

Ruling:
The petition of the Municipality of Makati is hereby granted and COA’s decision set aside.
Resolution no.60 which was re-enacted by the continuing program of the government to
promote social justice. The Burial Assistance Program is a relief of pauperism though not
complete. The care for the poor is generally recognized as a public duty. The support for
the poor has long been accepted exercise of police power in the promotion of common
good.

Page 108 of 166


CASE NO. 52
QUEZON CITY VS. ERICTA
G.R. No. L-34915
June 24, 1983

Facts:
This is a petition for review which seek the reversal of the decision of the Court of First
Instance of Rizal, Br.18 declaring Section 9 of the Ordinance no.6118, S64 of the Quezon
City Council null and void.” Ordinance requesting establishment, maintenance and
operation of Private Memorial type cemetery or burial grounds within the jurisdiction of
Quezon City and providing penalties for the Violation thereof”

Section 9 states that at least six (6) percent of the total area of the memorial park cemetery
shall be set aside for charity burial of deceased persons who are pauper and hence
resident of Quezon City for at least 5 years prior to their death, to be determined by
competent city authorities. The areas so designated shall be immediately developed and
should be open for operation not later than six months from the date of approval of the
application. For several years this was not enforced but seven years after the enactment
of the ordinance. Queson City Council passed the following resolution:

Resolved by the Council of Quezon City assembled to request the City Engineer to stop
further selling and/or transaction of memorial park lots in Quezon City. Where the owners
thereof have failed to donate the required 6% spaced intended for burial. The Himlayang
Filipino was notified thereof which led to the petition for declaratory relief,prohibition and
mandamus with preliminary injunction seeking to annul the ordinance which Judge Ericta
construed and make it null and void. This led to petition for review by the local
government of Quezon City.

Issue:
Whether or not Section 9 Ordinance No.6118 S, 64 of Quezon City is a valid exercise of
police power wherein the taking away of land is reasonable as it is extended for burial
pauper ground.

Ruling:
The petition for review is hereby dismissed and decision of the respondent court affirmed.
Based on the charter of Quezon City (Rep. Act No.537) there is no provision that will justify
the ordinance except the granting of police power. But such power to regulate does not
include the power to prohibit. Police power is usually exercised in the form of mere
regulation or restriction in the use of liberty or property for the promotion of general
welfare. It does not involve confiscation of property unless for the purpose of protecting
peace and order.

Page 109 of 166


CASE NO. 53
MARIANO VS. COMELEC
G.R. Nos. 118577 & 118627
March 7, 1995

Facts:
Certain provisions of Republic Act No. 7854 or "An Act Converting the Municipality of
Makati Into a Highly Urbanized City to be known as the City of Makati are being assailed
by petitioners as unconstitutional on the following grounds:

1. Section 2 did not properly identify the land area or territorial jurisdiction of
Makati by metes and bounds, with technical descriptions, in violation of Section
10, Article X of the Constitution, in relation to Sections 7 and 450 of the Local
Government Code;

2. Section 51 attempts to alter or restart the "three consecutive term" limit for local
elective officials, in violation of Section 8, Article X and Section 7, Article VI of the
Constitution

3. Section 52 of R.A. No. 7854 is unconstitutional for:

(a) it increased the legislative district of Makati only by special law (the
Charter in violation of the constitutional provision requiring a general
reapportionment law to be passed by Congress within three (3) years
following the return of every census;

(b) the increase in legislative district was not expressed in the title of the bill;
and

(c) the addition of another legislative district in Makati is not in accord with
Section 5 (3), Article VI of the Constitution for as of the latest survey (1990
census), the population of Makati stands at only 450,000.

Issue:
Whether or not R.A. 7854 is unconstitutional

Ruling:
No. Said delineation did not change even by an inch the land area previously covered by
Makati as a municipality. Section 2 did not add, subtract, divide, or multiply the
established land area of Makati. In language that cannot be any clearer, section 2 stated
that, the city's land area "shall comprise the present territory of the municipality."

Page 110 of 166


The Court cannot entertain the challenge to the constitutionality of Section 51. The
requirements before a litigant can challenge the constitutionality of a law are well
delineated. They are: (1) there must be an actual case or controversy; (2) the question of
constitutionality must be raised by the proper party; (3) the constitutional question must
be raised at the earliest possible opportunity; and (4) the decision on the constitutional
question must be necessary to the determination of the case itself. Considering that these
contingencies may or may not happen, petitioners merely pose a hypothetical issue which
has yet to ripen to an actual case or controversy.

In Tobias vs Abalos, Court ruled that reapportionment of legislative districts may be made
through a special law, such as in the charter of a new city.

Doctrine: The requirement on metes and bounds was meant merely as tool in the
establishment of local government units. It is not an end in itself. Ergo, so long as the
territorial jurisdiction of a city may be reasonably ascertained, i.e., by referring to common
boundaries with neighboring municipalities, as in this case, then, it may be concluded that
the legislative intent behind the law has been sufficiently served.

Page 111 of 166


CASE NO. 54
NUEVA ERA VS. MARCOS
G.R. No. 169435
February 27, 2008

Facts:
The Municipality of Nueva Era was created from the settlements of Bugayong,
Cabittaoran, Garnaden, Padpadon, Padsan, Paorpatoc, Tibangran, and Uguis which were
previously organized as rancherias, each of which was under the independent control of
a chief. By virtue of E.O. No. 66 5 these rancherias were united and the township of Nueva
Era was created.

The Municipality of Marcos, on the other hand, was created on June 22, 1963 pursuant to
Republic Act (R.A.) No. 3753 entitled "An Act Creating the Municipality of Marcos in the
Province of Ilocos Norte."

Section 1 of R.A. No. 3753 provides that certain barrios in the Municipality of Dingras,
Province of Ilocos Norte, are separated from the said municipality and constituted into a
new and separate municipality to be known as the Municipality of Marcos, with the
following boundaries:

On the Northwest, by the barrios Biding-Rangay boundary going down to the barrios
Capariaan-Gabon boundary consisting of foot path and feeder road; on the Northeast, by
the Burnay River which is the common boundary of barrios Agunit and Naglayaan; on the
East, by the Ilocos Norte-Mt. Province boundary; on the South, by the Padsan River which
is at the same time the boundary between the municipalities of Banna and Dingras; on
the West and Southwest, by the boundary between the municipalities of Batac and
Dingras.

Marcos did not claim any part of Nueva Era as its own territory until after almost 30 years,
7 or only on March 8, 1993, when its Sangguniang Bayan passed Resolution No. 93-015.
Said resolution was entitled: "Resolution Claiming an Area which is an Original Part of
Nueva Era, But Now Separated Due to the Creation of Marcos Town in the Province of
Ilocos Norte."

Marcos submitted its claim of the middle portion of Nueva Era to the SP of Ilocos Norte
for its consideration and approval wherein it posited that Nueva Era was cut into two
parts. And since the law required that the land area of a municipality must be compact
and contiguous, Nueva Era's northern isolated portion could no longer be considered as
its territory but that of Marcos'. Thus, Marcos claimed that it was entitled not only to the

Page 112 of 166


middle portion of Nueva Era but also to Nueva Era's isolated northern portion. These areas
claimed by Marcos were within Barangay Sto. Niño, Nueva Era.

Nueva Era reacted to the claim of Marcos and alleged that since time immemorial, its
entire land area was an ancestral domain of the "tinguians," an indigenous cultural
community. It argued to the effect that since the land being claimed by Marcos must be
protected for the tinguians, it must be preserved as part of Nueva Era. Nueva Era claimed
R.A. No. 3753 specifically mentioned seven (7) barrios of Dingras to become Marcos, the
area which should comprise Marcos should not go beyond the territory of said
barrios. On March 29, 2000, the SP of Ilocos Norte ruled in favor of Nueva Era. The fallo
of its decision. R.A. No. 3753 expressly named the barangays that would comprise Marcos,
but none of Nueva Era's barangays were mentioned. The SP thus construed, applying the
rule of expressio unius est exclusio alterius, that no part of Nueva Era was included by R.A.
No. 3753 in creating Marcos.

Issue:
Whether or not CA erred in its appreciation of facts, in declaring that MARCOS East is not
coterminous with the Eastern boundary of its mother town-Dingras

Ruling:
Yes. No part of Nueva Era's territory was taken for the creation of Marcos under R.A. No.
3753. Since only the barangays of Dingras are enumerated as Marcos' source of territory,
Nueva Era's territory is, therefore, excluded. Under the maxim expressio unius est exclusio
alterius, the mention of one thing implies the exclusion of another thing not mentioned.
If a statute enumerates the things upon which it is to operate, everything else must
necessarily and by implication be excluded from its operation and effect. This rule, as a
guide to probable legislative intent, is based upon the rules of logic and natural workings
of the human mind. Legislature intended other barangays from Nueva Era to become part
of Marcos, it could have easily done so by clear and concise language. Where the terms
are expressly limited to certain matters, it may not by interpretation or construction be
extended to other matters. The rule proceeds from the premise that the legislature would
not have made specified enumerations in a statute had the intention been not to restrict
its meaning and to confine its terms to those expressly mentioned. Furthermore, this
conclusion on the intention of the legislature is bolstered by the explanatory note of the
bill which paved the way for the creation of Marcos. Said explanatory note mentioned
only Dingras as the mother municipality of Marcos. Where there is ambiguity in a statute,
as in this case, courts may resort to the explanatory note to clarify the ambiguity and
ascertain the purpose and intent of the statute. Despite the omission of Nueva Era as a
mother territory in the law creating Marcos, the latter still contends that said law included
Nueva Era. It alleges that based on the description of its boundaries, a portion of Nueva
Era is within its territory.

Page 113 of 166


CASE NO. 55
TANO VS. SOCRATES
278 SCRA 154
August 21, 1997

Facts:
On December 15, 1992, the Sangguniang Panlungsod of Puerto Princesa City enacted
Ordinance No. 15-92 which took effect on January 1, 1993 entitled “An Ordinance banning
the shipment of all live fish and lobster outside Puerto Princesa City from January 1, 1993
to January 1, 1998 and providing exemptions, penalties and for other purposes thereof.”

Petitioners Airline Shippers Association of Palawan and other marine merchants were
charged criminally for violation of the above enactments. Petitioners invoke the
preferential right of marginal fishermen in Sec. 149 of the LGC, which provides: Fishery
Rentals, Fees and Charges; (b) The sangguniang bayan may:
(1) Grant fishery privileges to erect fish corrals, oyster, mussels or other aquatic beds or
bangus fry areas, within a definite zone of the municipal waters, as determined by it:
Provided, however, That duly registered organizations and cooperatives of marginal
fishermen shall have the preferential right to such fishery privileges..

In their comment, public respondents defended the validity of Oridnance No. 2, Series of
1993, as a valid exercise of the Provincial Fovernment’s power under the general welfare
clause.

Issue:
Whether or not LGUs may enact police power measures pursuant to the general welfare
clause

Ruling:
Yes. The so-called “preferential right” of subsistence or marginal fishermen to the use of
marine resources is not at all absolute.

The LGC provisions invoked by private respondents seek to give flesh and blood to the
right of the people to a balanced and healthful ecology. In fact, the Genral Welfare Clause
expressly mentions this right. Sec. 5(c) of the LGC explicitly mandates that the general
welfare provisions of the LGC “shall be liberally interpreted to give more powers to the
local government units in accelerating economic development and upgrading the quality
of life for the people of the community.”

One of the devolved powers enumerated in the section of the LGC on devolution is the
enforcement of fishery laws in municipal waters including the conservation of mangroves.

Page 114 of 166


This necessarily includes the enactment of ordinances to effectively carry out such fishery
laws within the municipal waters.

In light of the principles of decentralization and devolution enshrined in the LGC, and the
powers granted therein to local government units under the General Welfare Clause,
which unquestionaly involve the exercise of police power, the validity of the questioned
ordinances cannot be doubted.

Page 115 of 166


CASE NO. 56
MUNICIPALITY OF KANANGA VS. ORMOC CITY
G.R. No. 141375
April 30, 2003

Facts:
There was a boundary dispute between the Municipality of Kananga and Ormoc City. Both
parties submitted to have an amicable settlement by a joint session of Sangguniang
Panlungsod (Ormoc City) and Sangguniang Bayan (Municiaplity of Kananga). To no avail,
no settlement was reached.

Both parties then elevated the conflict to the Regional Trial Court of Ormoc City via a
resolution. Petitioner filed a Motion to Dismiss on the main contention that the RTC has
no jurisdiction over the subject matter. The trial court ruled that it has jurisdiction over
the subject matter because the parties substantially complied with Section 118 of the
Local Government Code as they were able to thresh out their differences and came to an
agreement to elevate it to the trial court.

Issue:
Whether or not the Regional Trial Court has an original and exclusive jurisdiction over the
settlement of a boundary dispute between a municipality and an independent component
city.

Ruling:
The Court affirms that the Regional Trial Court has an original and exclusive jurisdiction
over the settlement of a boundary dispute between a municipality and a component city.

Under Section 451 of the LGC, a city may be either component or highly urbanized. Ormoc
is deemed an independent component city, because its charter prohibits its voters from
voting for provincial elective officials. Section 118 of the LGC is only applicable when a
component city or a municipality seeks to settle a boundary dispute with a highly
urbanized city, not with an independent city. Kananga is a municipality and Ormoc is an
independent component city, such procedure provided in the said section is not
applicable.

Page 116 of 166


It is then the general rules governing jurisdiction that should be used (Sec. 19 of BP 129
also known as “Judiciary Reorganization Act of 1980”) and that there is no law providing
for the exclusive jurisdiction of any court with regards to the settlement of boundary
dispute between a municipality and an independent component city. RTC’s have general
jurisdiction to adjudicate all controversies except those expressly withheld from their
plenary powers.

Since there is no law providing for the exclusive jurisdiction of any court or agency over
the settlement of boundary disputes between a municipality and an independent
component city of the same province, the RTC of Ormoc City did not commit a grave
abuse of discretion in denying the Motion to Dismiss. They have the power not only to
take judicial cognizance of a case instituted for judicial action for the first time but also to
do so to the exclusion of all other courts at that stage. The power is not only original but
also exclusive.

Page 117 of 166


CASE NO. 57
BAI SANDRA SEMA VS. COMELEC
G.R. No. 177597
July 16, 2008

Facts:
The Province of Maguindanao was created under RA 6734 as amended by RA 9054. It
forms part of the Autonomous Region in Muslim Mindanao. There were two legislative
districts that consists in the Province of Maguindanao. The first comprises of Cotabato
City and eight municipalities. Cotabato City, on the other hand, voted against its inclusion
in the ARMM in a plebiscite, therefore, it is under Region 12.

The ARMM Regional Assembly (ARMM’s legislature), enacted Muslim Mindanao Act No.
201 (MMA 201), creating the Province of Shariff Kabunsuan that composed of eight
municipalities in the first legislative district. It exercised its power to create provinces
provided in Section 19, Article 6 of RA 9054.

Its voters ratified through a plebiscite, the creation of the Province of Kabunsuan. The
COMELEC issued a resolution that would maintain Cotabato City as part of the Shariff
Kabunsuan in the first legislative district. Only to be renamed later on as Shariff Kabunsuan
Province with Cotabato City (COMELEC Resolution 7902).

Running as a candidate for being the Representative of Shariff Kabunsuan with Cotabato
City, Sema prayed for the nullification of COMELEC Resolution 7902 and the exclusion of
votes for Cotabato City to that office she is running for.

The contentions of the parties are as follows:


1. COMELEC Resolution 7902 (maintaining the status quo of Cotabato City as part of the
Province of Shariff Kabunsuan in the first legislative district)

As for Sema, she contended that only one representative should be entitled in the
Congress and that in issuing COMELEC Resolution 7902, the COMELEC usurped the power
of Congress to create legislative districts.

As their answer, COMELEC and Dilangalen counter that the said resolution is
constitutional because it did not apportion a legislative district for Shariff Kabunsuan but

Page 118 of 166


rather, it merely named Maguindanao’s first district. It added that the COMELEC has no
power to reapportion the first legislative district because the power to reapportion
legislative districts only lies with the Congress.

2. Constitutionality of Section 19, Article 6 of RA 9054 (delegation of the power to create


province, cities, municipalities and barangays to the ARMM Regional Assembly)

As for Sema, she contended that it is constitutional as a valid delegation of the Congress
to the ARMM of the power to create provinces. They are granted to do as such through
their organic acts and legislative powers as may be authorized by law for the promotion
of the general welfare of the people and the region.

As for Dilangalen, he contended that it is unconstitutional because the power to create


provinces was not among those granted to the autonomous regions under Section 20,
Article 10 of the Constitution.

Issues:
A. Whether or not Section 19, Article 6 of RA 9054 is constitutional;
B. Whether or not a province created by the ARMM Regional Assembly under MMA 201
(in accordance to its powers in Section 19, Article 6 of RA 9054), is entitled to one
representative in the Congress without a need of a national law creating a legislative
district for such province;
C. Whether or not COMELEC Resolution 7902 is valid

Ruling:
A. Constitutionality of Section 19, Article 6 of RA 9054. The Court ruled that Section 19,
Article 6 of RA 9054 is unconstitutional with regards to the delegation of the powers
to create provinces and cities to the ARMM regional Assembly.

There are 3 conditions that must be met for the creation of local government units:
First is that, the creation of a local government must follow the criteria fixed in the
Local Government Code. Second, that such creation must not be in conflict with any
provision of the Constitution, and third, there must be a plebiscite in the political units
affected.

Under the Local Government Code, only an act of Congress can create provinces, cities
or municipalities. There is neither an express prohibition nor an express grant of

Page 119 of 166


authority in the Constitution for the Congress to delegate to regional or local
legislative bodies the power to create Local Government Units. However, the Congress
can delegate such power as long as no conflict will arise with any provision of the
Constitution and in reasonable standards.

For Congress to validly delegate the power to create provinces or cities, it must validly
delegate at the same time the power to create a legislative district. The power to create
provinces or cities inherently involves the power to create a legislative district.

B. The province created under MMA 201 is entitled to a one representative in Congress
The Court ruled that MMA 201 is void because under the Constitution, the power to
increase the allowable membership in the House of Representatives and to
reapportion legislative districts is vested exclusively in Congress. The Congress
exercises these powers through a law that the Congress itself enacts, and not through
a law that regional or local legislative bodies enact.

There is nothing in Section 20, Article 10 of the Constitution that specifically authorizes
autonomous regions, expressly or impliedly, to create or reapportion legislative
districts for Congress. Meanwhile, in Section 3, Article 4 of RA 9054 (amending the
ARMM Organic Act), it provides that the Regional Assembly may exercise legislative
power and one of the exceptions is with regards to national elections. The ARMM
Regional Assembly therefore, cannot enact a law creating a national office like the
office as that of a district representative of Congress because its legislative powers
operate only within its territorial jurisdiction as provided for in Section 20, Article 10.

C. COMELEC Resolution 7902. The Court ruled that COMELEC Resolution 7902 is valid
because it merely complies with Section 5 of Article 6 and Section 20 of Article 10 of
the Constitution.

Page 120 of 166


CASE NO. 58
MUNICIPALITY OF MALABANG VS. PANGANDAPUN BENITO
G.R. No. L-28113
March 28, 1969

Facts:
Executive Order 386 was created on March 15, 1960. It included Balabagan as a part of
the municipality of Malabang. Amer Balindong (petitioner) is the mayor of municipality of
Malabang while Pagandapun Bonito (respondent) is the mayor of municipality of
Balabagan. Petitioner instituted the action for prohibition and nullification of Executive
Order 386 and to restrain the respondents from exercising their functions from their
offices.

The contentions of the parties are as follows:


As for petitioner, he based his argument on the ruling in Pelaez v. Auditor General. The
ruling was that the President’s power has no authority to create a new barrio under the
Barrio Charter Act because it is only the provincial board who has vested powers. As for
creating municipalities, the President is limited for supervision of the power of local
governments and that it is an undue delegation power by the legislative.

As for respondent, it contended that the municipality of Balabagan is at least a de facto


corporation of which it was organized under color of a statute before it became
unconstitutional. Therefore, the ruling in the Pelaez case cannot stand.

Issue:
Whether or not the municipality of Balabagan is a de facto corporation.

Ruling:
The Court ruled that the municipality of Balabagan is not a de facto corporation. Executive
Order 386 is declared void.

The principle that color of title under an unconstitutional statute can exist only where
there is some other valid law enacted by the legislature or an unconstitutional law which
has been: (1) upheld for a long time by the courts or (2) not yet been declared void
provided, that a warrant for its creation can be found in some other valid law or in
recognition of the constitution or general laws of the state. Where a de facto corporation
was recognized as such despite the fact that the statute creating it was later invalidated,
the decisions could fairly be made to rest on the consideration that there was some other
valid law giving corporate vitality to the organization.

Page 121 of 166


In the case at bar, the mere fact Balabagan was organized at a time when the statute had
not been invalidated cannot make it as a de facto corporation as independently of the
Administrative Code provision in question, there is no other valid statute to give color of
authority to its creation. The existence of Executive Order 386 is an operative fact which
cannot be justly ignored because it created no office and is as inoperative as though it
had never been passed.

Page 122 of 166


CASE NO. 59
ARSADI M. DISOMANGCOP VS. THE SECRETARY OF DPWH
G.R. No. 149848
November 25, 2004

Facts:
On August 1, 1989, Republic Act No. 6734 (R.A. 6734), entitled “An Act Providing for An
Organic Act for the Autonomous Region in Muslim Mindanao,” was passed. Four
provinces voted for inclusion in ARMM, namely: Lanao del Sur, Maguindanao, Sulu and
Tawi-Tawi. In accordance with R.A. 6734, Pres. Cory Aquino issued Executive Order No.
426 (E.O. 426), entitled “Placing the Control and Supervision of the Offices of the
Department of Public Works and Highways within the Autonomous Region in Muslim
Mindanao under the Autonomous Regional Government, and for other purposes.” on
October 12, 1990. The same devolved to the ARMM the power of the DPWH.

Consequently, DPWH Department Order 119 (D.O. 119) entitled "Creation of Marawi Sub-
District Engineering Office" was issued by then Sec. Gregorio R. Vigilar on May 20, 1999,
which is in accordance with EO 124. It created a DPWH Marawi Sub-District Engineering
Office which shall have jurisdiction over all national infrastructure projects and facilities
under the DPWH within Marawi City and Lanao del Sur.

On January 17, 2001, R.A. 8999 which created a new Engineering District in the first district
of Lanao del Sur was passed under Pres. Joseph Estrada entitled “An Act Establishing an
Engineering District as the First District of Lanao Del Sur and Appropriating Funds
Therefor”. On March 31, 2001, RA 9054 entitled “An Act to Strengthen and Expand the
Organic Act for the Autonomous Region in Muslim Mindanao, Amending for the Purpose
Republic Act No. 6734, entitled An Act Providing for the Autonomous Region in Muslim
Mindanao, as Amended” which amended R.A. 6734 was passed. The province of Basilan
and the City of Marawi voted to join ARMM through said law.

Disomangcop and Dimalotang in their capacity as Officer-in-Charge and Engineer II


respectively of the First Engineering District of DPWH-ARMM in Lanao del Sur filed a
petition questioning the constitutionality and validity of D.O. 119 and R.A. 8999 on the
ground that they contravene the constitution and the organic acts of the ARMM.
Moreover they sought mainly the following relief: to prohibit respondent DPWH Secretary
from implementing D.O. 119 and R.A. 8999 and releasing funds for public work projects
intended for Lanao Del Sur and Marawi City to the Marawi Sub-District Engineering Office
and other administrative regions of DPWH.

Issue:
Whether or not D.O. 119 and R.A. 8999 are both invalid and constitutionally infirm

Page 123 of 166


Ruling:
Yes, R.A. 8999 never became an operative and was superseded or repealed by R.A. 9054.
R.A. 8999 is patently inconsistent with R.A. 9054 which is a later law. R.A. 9054, which is
anchored on the 1987 Constitution advances the constitutional grant of autonomy by
detailing the powers of the ARMM which covers among others Lanao del Sur. However,
R.A. 8999 ventures to re-establish the National Government's jurisdiction over the
infrastructure programs in Lanao del Sur. R.A. 8999 is patently inconsistent with R.A. 9054,
and it destroys the latter law's objective of devolution of the functions of DPWH in line
with the policy of the Constitution to grant LGUs meaningful and authentic regional
autonomy.

D.O. 119 creating the Marawi Sub-District Engineering Office which has jurisdiction over
infrastructure projects within Marawi City and Lanao del Sur violates the provisions of E.O.
426 which implements the transfer of control and supervision of the DPWH to the ARMM
in line with R.A. 6734. The office created under D.O. 119 having essentially the same
powers with the District Engineering Office of Lanao del Sur as created under E.O. 426, is
a duplication. The DO in effect takes back powers which have been previously devolved
under E.O. 426. R.A. 9054 however has repealed D.O. 119.

Furthermore, the ARMM Organic Acts are deemed a part of the regional autonomy
scheme. While they are classified as statutes, the Organic Acts are more than ordinary
statutes because they enjoy affirmation by a plebiscite. Hence, the provisions thereof
cannot be amended by an ordinary statute, such as R.A. 8999 in this case. The amendatory
law has to be submitted to a plebiscite.

Page 124 of 166


CASE NO. 60
THE MUNICIPALITY OF SOGOD vs. HON. AVELINO S. ROSAL
G.R. No. 38204
September 24, 1991

Facts:
On June 15, 1950, Congress passed Republic Act No. 522 (R.A. 522) creating the
municipality of Bontoc, formerly a barrio of the municipality of Sogod in the province of
Leyte. A boundary dispute however, later arose between the municipality of Bontoc and
the municipality of Sogod with the latter claiming that the former exercised jurisdiction
not only over the barrios mentioned in R.A. 522 but also over other ten (10) barrios
allegedly belonging to Sogod.

On June 17, 1952, the Provincial Board of Leyte issued Resolution No. 617 directing the
holding of a plebiscite among the barrios of Pangi, Taa part of Sta. Cruz, Tuburan,
Lawgawan and their corresponding sitios. The purpose of the plebiscite is to determine
whether the people in these barrios would like to remain with the municipality of Sogod
or with Bontoc. The plebiscite was conducted on August 1, 1952, and the results thereof
show that more votes were cast in favor of Sogod than those in favor of Bontoc.

On April 4, 1959, the Provincial Board of Leyte issued Resolution No. 519 recommending
to the President of the Philippines and/or to the Congress of the Philippines that R.A. 522
be amended so as to include in said Act creating the municipality of Bontoc, nine (9)
barrios claimed by Sogod which are in the heart of Bontoc but not included in said law.
The Board also recommended that a law be enacted annexing to the municipality of
Sogod five (5) barrios which are very near Sogod and are claimed by the latter but are
included in the law creating Bontoc. On December 28, 1959, then President Carlos P.
Garcia, promulgated Executive Order No. 368, which approved the recommendation of
the Provincial Board of Leyte. However, on July 14, 1960, the President of the Philippines
sent a telegram to the Provincial Board of Southern Leyte suspending the implementation
of EO 368.

On July 18, 1960, the Provincial Board of Southern Leyte passed Resolution No. 62
suspending the implementation of Executive Order 368. The Board also created a
committee to conduct the holding of a plebiscite in the barrios and sitios affected by
Executive Order 368 and to finally settle the boundary dispute.

On June 24, 1970, the Municipality of Sogod filed a case for certiorari and prohibition with
the Court of First Instance of Southern Leyte, to enjoin the provincial board and provincial
governor from taking cognizance of the long pending boundary dispute between the two

Page 125 of 166


municipalities and to enjoin the municipality of Bontoc from exercising territorial
jurisdiction over its claimed barrios allegedly belonging to the municipality of Sogod.

On December 7, 1970, the municipality of Sogod another case with the Court of First
Instance of Southern Leyte for recovery of taxes with receivership against the municipality
of Bontoc. The complaint alleged that the municipality of Bontoc, without any legal basis,
exercised jurisdiction not only over the barrios emunerated in Republic Act No. 522 but
also over ten (10) barrios belonging to the complainant municipality of Sogod. The trial
court dismissed both cases for lack of jurisdiction.

Issue:
Whether or not the trial court gravely erred in dismissing the two cases for lack of
jurisdiction.

Held:
No. The law vested the right to settle boundary disputes between municipalities on the
provincial board pursuant to Section 2167 of the Revised Administrative Code, which
reads provides that disputes as to jurisdiction of municipal governments over places or
barrios shall be decided by the provincial boards of the provinces in which such
municipalities are situated, after an investigation at which the municipalities concerned
shall be duly heard, From the decision of the provincial board appeal may be taken by the
municipality aggrieved to the Secretary of the Interior, whose decision shall be final.
Where the places or barrios in dispute are claimed by municipalities situated in different
provinces, the provincial boards of the provinces concerned shall come to an agreement
if possible, but, in the event of their failing to agree, an appeal shall be had to the Secretary
of Interior, whose decision shall be final.

It is clear from the aforestated legal provision that the authority to hear and resolve
municipal boundary disputes belongs to the provincial boards and not to the trial courts.
The decisions of the boards are then appealable to the Executive Secretary.

However, this dispute has already been overtaken by events, namely, the enactment of
the 1987 Constitution and the New Local Government Code on February 10, 1983, which
imposed new mandatory requirements and procedures on the fixing of boundaries
between municipalities. The 1987 Constitution now mandates that no province, city,
municipality or barangay may be created, divided, merged, abolished or its boundary
substantially altered except in accordance with the criteria established in the local
government code and subject to approval by a majority of the votes cast in a plebiscite
in the political units directly affected. Hence, any alteration or modification of the
boundaries of the municipalities shall only be by a law to be enacted by Congress subject
to the approval by a majority of the votes cast in a plebiscite in the barrios affected

Page 126 of 166


(Section 134, Local Government Code). Thus, under present laws, the function of the
provincial board to fix the municipal boundaries are now strictly limited to the factual
determination of the boundary lines between municipalities, to be specified by natural
boundaries or by metes and bounds in accordance with the laws creating said
municipalities.

Page 127 of 166


CASE NO. 61
VELASCO VS. VILLEGAS
G.R. No. L-24153
February 14, 1983

Facts:
This is an appeal from an order of the lower court dismissing a suit for declaratory relief
challenging the constitutionality based on Ordinance No. 4964 of the City of Manila, being
that it amounts to a deprivation of property of petitioners-appellants of their means of
livelihood without due process of law.

The assailed ordinance is worded thus: "It shall be prohibited for any operator of any
barber shop to conduct the business of massaging customers or other persons in any
adjacent room or rooms of said barber shop, or in any room or rooms within the same
building where the barber shop is located as long as the operator of the barber shop and
the rooms where massaging is conducted is the same person."

Issue:
Whether the attack against the validity of ordinance can succeed?

Ruling:
No.
1. It is a police power measure. The objectives behind its enactment are: "(1) To be
able to impose payment of the license fee for engaging in the business of massage
clinic under Ordinance No. 3659 as amended by Ordinance 4767, an entirely
different measure than the ordinance regulating the business of barbershops and,
(2) in order to forestall possible immorality which might grow out of the
construction of separate rooms for massage of customers." This Court has been
most liberal in sustaining ordinances based on the general welfare clause.
2. There is no showing, therefore, of the unconstitutionality of such ordinance.

Page 128 of 166


CASE NO. 62
DELA CRUZ VS. PARAS
G.R. No. L-42571-72
July 25, 1983

Facts:
1. Vicente De La Cruz et al were club & cabaret operators. They assail the
constitutionality of Ord. No. 84, Ser. of 1975 or the Prohibition and Closure
Ordinance of Bocaue, Bulacan. De la Cruz averred that the said Ordinance
violates their right to engage in a lawful business for the said ordinance would
close out their business

2. On November 5, 1975, two cases for prohibition with preliminary injunction


were filed with the Court of First Instance of Bulacan. The grounds alleged
follow:

a. Ordinance No. 84 is null and void as a municipality has no authority to


prohibit a lawful business, occupation or calling.

b. Ordinance No. 84 is violative of the petitioners' right to due process and the
equal protection of the law, as the license previously given to petitioners was
in effect withdrawn without judicial hearing.

c. That under Presidential Decree No. 189, as amended, by Presidential Decree


No. 259, the power to license and regulate tourist-oriented businesses
including night clubs, has been transferred to the Department of Tourism.

3. Judge Paras however lifted the TRO he earlier issued against Ord. 84 after due
hearing declaring that Ord 84. is constitutional for it is pursuant to RA 938 which
reads “AN ACT GRANTING MUNICIPAL OR CITY BOARDS AND COUNCILS THE
POWER TO REGULATE THE ESTABLISHMENT, MAINTENANCE AND OPERATION
OF CERTAIN PLACES OF AMUSEMENT WITHIN THEIR RESPECTIVE TERRITORIAL
JURISDICTIONS”. Paras ruled that the prohibition is a valid exercise of police
power to promote general welfare. De la Cruz then appealed citing that they
were deprived of due process.

Issue:
Whether or not a municipal corporation, Bocaue, Bulacan can, prohibit the exercise of a
lawful trade, the operation of night clubs, and the pursuit of a lawful occupation, such
clubs employing hostesses pursuant to Ord 84 which is further in pursuant to RA 938.

Page 129 of 166


Ruling:
No. it cannot. The SC ruled that if night clubs were merely then regulated and not
prohibited, certainly the assailed ordinance would pass the test of validity. SC had stressed
reasonableness, consonant with the general powers and purposes of municipal
corporations, as well as consistency with the laws or policy of the State. It cannot be said
that such a sweeping exercise of a lawmaking power by Bocaue could qualify under the
term reasonable. The objective of fostering public morals, a worthy and desirable end can
be attained by a measure that does not encompass too wide a field. Certainly the
ordinance on its face is characterized by overbreadth. The purpose sought to be achieved
could have been attained by reasonable restrictions rather than by an absolute
prohibition. Pursuant to the title of the Ordinance, Bocaue should and can only regulate
not prohibit the business of cabarets.

Page 130 of 166


CASE NO. 63
PHILIPPINE GAMEFOWL COMMISSION VS. INTERMEDIATE APPELLATE COURT
G.R. Nos. L-72969-70
December 17, 1986
Facts:
1. The issue when Hee Acusar, who was operating the lone cockpit in Bogo, was
ordered to relocate the same pursuant to P.D. No. 449, the Cockfighting Law of
1974, on the ground that it was situated in a tertiary commercial zone, a
prohibited area. Although the period of grace for such relocation was extended
to June 11, 1980 by P.D. 1535, Acusar failed to comply with the requirement, as a
result of which the Philippine Constabulary considered the cockpit phased out.

2. Santiago Sevilla, private respondent herein, was granted a license to operate a


cockpit by Mayor Celestino E. Martinez by authority of the Sangguniang Bayan of
Bogo and with subsequent approval of the PC Regional Command 7 as required
by law. As only one cockpit is allowed by law in cities or municipalities with a
population of not more than one hundred thousand.

3. Acusar went to the Philippine Gamefowl Commission seeking a renewal of his


cockpit license and the cancellation of Sevilla's in what was docketed as PGC Case
No. 10. He succeeded initially with the issuance by the PGC on August 16, 1984,
of an interlocutory order allowing him to temporarily operate his cockpit.

Issue:
Whether the license given to Sevilla by the Municipalities prevail over the PGC?

Ruling: Yes. The conferment of the power to license and regulate municipal cockpits in
the municipal authorities is in line with the policy of local autonomy embodied in Article
II, Section 10, and Article XI of the 1973 Constitution. It is also a recognition, as the Court
of Appeals correctly points out, of the superior competence of the municipal officials in
dealing with this local matter with which they can be expected to be more knowledgeable
than the national officials. Surely, the Philippine Gamefowl Commission cannot claim to
know more than the municipal mayor and the Sangguniang Bayan of Bogo, Cebu, about
the issues being disputed by the applicants to the cockpit license.

At any rate, assuming that the resolution of the Sangguniang Bayan authorizing the
issuance of a cockpit license to Sevilla was subject to reversal by the PGC, such action
could be justified only if based upon a proven violation of law by the municipal officials.
It may not be made only for the purpose of substituting its own discretion for the
discretion exercised by the municipal authorities in determining the applicant to which
the lone cockpit license should be issued.

Page 131 of 166


In the absence of a clear showing of a grave abuse of discretion, the choice of the
municipal authorities should be respected by the PGC and in any event cannot be replaced
by it simply because it believes another person should have been selected. Stated
otherwise, the PGC cannot directly exercise the power to license cockpits and in effect
usurp the authority directly conferred by law on the municipal authorities.

Page 132 of 166


CASE NO. 64
JOSE MONDANO VS. FERNANDO SILVOSA
G.R. No. L-7708
May 30, 1955

Facts:
Mondano is the duly elected and qualified mayor of the municipality of Mainit, province
of Surigao. Consolacion Vda. de Mosende filed a sworn complaint with the Presidential
Complaints and Action Committee accusing him of (1) rape committed on her daughter
Caridad Mosende; and (2) concubinage for cohabiting with her daughter in a place other
than the conjugal dwelling. The Assistant Executive Secretary indorsed the complaint to
the respondent provincial governor for immediate investigation, appropriate action and
report. The provincial governor issued Administrative Order No. 8 suspending the
petitioner from office. Respondent assailed that the department head is clothed with
"direct control, direction, and supervision over all bureaus and offices under his
jurisdiction" and to that end "may order the investigation of any act or conduct of any
person in the service of any bureau or office under his Department and in connection
therewith may appoint a committee or designate an official or person who shall conduct
such investigations.

Issue:
Whether or not the suspension of the petitioner as mayor is unlawful and without
authority of law.

Ruling:
YES. Section 10, paragraph 1, Article VII, of the Constitution provides: "The President shall
have control of all the executive departments, bureaus, or offices, exercise general
supervision over all local governments as may be provided by law, and take care that the
laws be faithfully executed."

Under this constitutional provision the President has been invested with the power of
control of all the executive departments, bureaus, or offices, but not of all local
governments over which he has been granted only the power of general supervision as
may be provided by law. The Department head as agent of the President has direct control
and supervision over all bureaus and offices under his jurisdiction as provided for in
section 79 (c) of the Revised Administrative Code, but he does not have the same control
of local governments as that exercised by him over bureaus and offices under his
jurisdiction. Likewise, his authority to order the investigation of any act or conduct of any
person in the service of any bureau or office under his department is confined to bureaus
or offices under his jurisdiction and does not extend to local governments over which, as
already stated, the President exercises only general supervision as may be provided by

Page 133 of 166


law. If "general supervision over all local governments" is to be construed as the same
power granted to the Department Head in section 79 (c) of the Revised Administrative
Code, then there would no longer be a distinction or difference between the power of
control and that of supervision. In administrative law supervision means overseeing or the
power or authority of an officer to see that subordinate officers perform their duties. If
the latter fail or neglect to fulfill them the former may take such action or step as
prescribed by law to make them perform their duties. Control, on the other hand, means
the power of an officer to alter or modify or nullify or set aside what a subordinate officer
had done in the performance of his duties and to substitute the judgment of the former
for that of the latter. The Congress has expressly and specifically lodged the provincial
supervision over municipal officials in the provincial governor who is authorized to
"receive and investigate complaints made under oath against municipal officers for
neglect of duty, oppression, corruption or other form of maladministration of office, and
conviction by final judgment of any crime involving moral turpitude."

In the indorsement to the provincial governor the Assistant Executive Secretary requested
immediate investigation, appropriate action and report on the complaint indorsed to him,
and called his attention to section 2193 of the Revised Administrative Code which
provides for the institution of judicial proceedings by the provincial fiscal upon direction
of the provincial governor. If the indorsement of the Assistant Executive Secretary be
taken as a designation of the provincial governor to investigate the petitioner, then he
would only be acting as agent of the Executive, but the investigation to be conducted by
him would not be that which is provided for in sections 2188, 2189 and 2190 of the
Revised Administrative Code. The charges preferred against the respondent are not
malfeasances or any of those enumerated or specified in section 2188 of the Revised
Administrative Code, because rape and concubinage have nothing to do with the
performance of his duties as mayor nor do they constitute or involve" neglect of duty,
oppression, corruption or any other form of maladministration of office." True, they may
involve moral turpitude, but before the provincial governor and board may act and
proceed in accordance with the provisions of the Revised Administrative Code referred
to, a conviction by final judgment must precede the filing by the provincial governor of
charges and trial by the provincial board. Even the provincial fiscal cannot file an
information for rape without a sworn complaint of the offended party who is 28 years of
age and the crime of concubinage cannot be prosecuted but upon sworn complaint of
the offended spouse. The charges preferred against the petitioner, municipal mayor, not
being those or any of those specified in section 2188 of the Revised Administrative Code,
the investigation of such charges by the provincial board is unauthorized and illegal. The
suspension of the petitioner as mayor is, consequently, unlawful and without authority of
law.

Page 134 of 166


CASE NO. 65
BASCO VS. PAGCOR
G.R. No. 91649
May 14, 1991

Facts:
Petitioners filed a petition seeking to annul the Philippine Amusement and Gaming
Corporation (PAGCOR) Charter — PD 1869, because it is allegedly contrary to morals,
public policy and order, and because it constitutes a waiver of a right prejudicial to a third
person with a right recognized by law. It waived the Manila City government's right to
impose taxes and license fees, which is recognized by law; and that the law has intruded
into the local government's right to impose local taxes and license fees which is in
contravention of the constitutionally enshrined principle of local autonomy

Issues:
A. Whether or not P.D. 1869 constitutes a waiver of the right of the City of Manila to
impose taxes and legal fees
B. Whether or not the tax exemption clause in P.D. 1869 is violative of the principle of
local autonomy.

Ruling:
A. NO. The City of Manila, being a mere Municipal corporation has no inherent right to
impose taxes Thus, "the Charter or statute must plainly show an intent to confer that
power or the municipality cannot assume it". Its "power to tax" therefore must always
yield to a legislative act which is superior having been passed upon by the state itself
which has the "inherent power to tax"

The Charter of the City of Manila is subject to control by Congress. It should be


stressed that "municipal corporations are mere creatures of Congress" which has the
power to "create and abolish municipal corporations" due to its "general legislative
powers" Congress, therefore, has the power of control over Local governments. And if
Congress can grant the City of Manila the power to tax certain matters, it can also
provide for exemptions or even take back the power.

The City of Manila's power to impose license fees on gambling, has long been revoked.
As early as 1975, the power of local governments to regulate gambling thru the grant
of "franchise, licenses or permits" was withdrawn by P.D. No. 771 and was vested
exclusively on the National Government. Therefore, only the National Government has
the power to issue "licenses or permits" for the operation of gambling. Necessarily,
the power to demand or collect license fees which is a consequence of the issuance of
"licenses or permits" is no longer vested in the City of Manila.

Page 135 of 166


Local governments have no power to tax instrumentalities of the National
Government. PAGCOR is a government owned or controlled corporation with an
original charter, PD 1869. All of its shares of stocks are owned by the National
Government. In addition to its corporate powers

PAGCOR has a dual role, to operate and to regulate gambling casinos. The latter role
is governmental, which places it in the category of an agency or instrumentality of the
Government. Being an instrumentality of the Government, PAGCOR should be and
actually is exempt from local taxes. Otherwise, its operation might be burdened,
impeded or subjected to control by a mere Local government.

The states have no power by taxation or otherwise, to retard, impede, burden or in any
manner control the operation of constitutional laws enacted by Congress to carry into
execution the powers vested in the federal government. This doctrine emanates from
the "supremacy" of the National Government over local governments. “No state or
political subdivision can regulate a federal instrumentality in such a way as to prevent
it from consummating its federal responsibilities, or even to seriously burden it in the
accomplishment of them. Otherwise, mere creatures of the State can defeat National
policies thru extermination of what local authorities may perceive to be undesirable
activities or enterprise using the power to tax as "a tool for regulation"

B. NO. Article X of the 1987 Constitution (on Local Autonomy) provides:


Sec. 5. Each local government unit shall have the power to create its own source of
revenue and to levy taxes, fees, and other charges subject to such guidelines and
limitation as the congress may provide, consistent with the basic policy on local
autonomy. Such taxes, fees and charges shall accrue exclusively to the local
government.

The power of local government to "impose taxes and fees" is always subject to
"limitations" which Congress may provide by law. Since PD 1869 remains an
"operative" law until "amended, repealed or revoked" (Sec. 3, Art. XVIII, 1987
Constitution), its "exemption clause" remains as an exception to the exercise of the
power of local governments to impose taxes and fees. It cannot therefore be violative
but rather is consistent with the principle of local autonomy.

Besides, the principle of local autonomy under the 1987 Constitution simply means
"decentralization" It does not make local governments sovereign within the state or
an "imperium in imperio." Local Government has been described as a political
subdivision of a nation or state which is constituted by law and has substantial control
of local affairs. In a unitary system of government, such as the government under the

Page 136 of 166


Philippine Constitution, local governments can only be an intra sovereign subdivision
of one sovereign nation, it cannot be an imperium in imperio. Local government in
such a system can only mean a measure of decentralization of the function of
government. As to what state powers should be "decentralized" and what may be
delegated to local government units remains a matter of policy, which concerns
wisdom. It is therefore a political question.

What is settled is that the matter of regulating, taxing or otherwise dealing with
gambling is a State concern and hence, it is the sole prerogative of the State to retain
it or delegate it to local governments.

Page 137 of 166


CASE NO. 66
LIMBONA VS. MANGELIN
G.R. No. 80391
February 28, 1989

Facts:
Limbona was appointed as a member of the Sangguniang Pampook, Regional
Autonomous Government, Region XII, representing Lanao del Sur and was elected
Speaker of the Regional Legislative Assembly or Batasang Pampook of Central Mindanao
(Assembly for brevity). Mr. Limbona was expelled from the Assembly since, allegedly he
pays Abdula his salaries and emoluments without authority from the Assembly which
constitute a usurpation of the power of the Assembly and that he "had recently caused
withdrawal of so much amount of cash from the Assembly resulting to the non-payment
of the salaries and emoluments of some Assembly members. The Supreme Court ordered
the reinstatement of Limbona. It ruled that the Assembly had not commence any proper
proceedings as a requirement of due process and while it is within the discretion of the
members of the Sanggunian to punish their erring colleagues, their acts are nonetheless
subject to the moderating band of this Court in the event that such discretion is exercised
with grave abuse.

Issue:
Are the so-called autonomous governments of Mindanao, as they are now constituted,
subject to the jurisdiction of the national courts? In other words, what is the extent of self-
government given to the two autonomous governments of Region IX and XII?

Ruling:
YES. The autonomous governments of Mindanao were organized in Regions IX and XII by
Presidential Decree No. 1618 promulgated on July 25, 1979. Among other things, the
Decree established "internal autonomy" in the two regions "[w]ithin the framework of the
national sovereignty and territorial integrity of the Republic of the Philippines and its
Constitution," with legislative and executive machinery to exercise the powers and
responsibilities specified therein.

It requires the autonomous regional governments to "undertake all internal administrative


matters for the respective regions," except to "act on matters which are within the
jurisdiction and competence of the National Government," "which include, but are not
limited to, the following: (1) National defense and security; (2) Foreign relations; (3)
Foreign trade; (4) Currency, monetary affairs, foreign exchange, banking and quasi-
banking, and external borrowing, (5) Disposition, exploration, development, exploitation
or utilization of all natural resources; (6) Air and sea transport (7) Postal matters and
telecommunications; (8) Customs and quarantine; (9) Immigration and deportation; (10)

Page 138 of 166


Citizenship and naturalization; (11) National economic, social and educational planning;
and (12) General auditing. In relation to the central government, it provides that "[t]he
President shall have the power of general supervision and control over the Autonomous
Regions ..."

Now, autonomy is either decentralization of administration or decentralization of power.


There is decentralization of administration when the central government delegates
administrative powers to political subdivisions in order to broaden the base of
government power and in the process to make local governments "more responsive and
accountable," "and ensure their fullest development as self-reliant communities and make
them more effective partners in the pursuit of national development and social progress."
At the same time, it relieves the central government of the burden of managing local
affairs and enables it to concentrate on national concerns. The President exercises
"general supervision" over them, but only to "ensure that local affairs are administered
according to law." He has no control over their acts in the sense that he can substitute
their judgments with his own.

Decentralization of power, on the other hand, involves an abdication of political power in


the favor of local governments units declare to be autonomous . In that case, the
autonomous government is free to chart its own destiny and shape its future with
minimum intervention from central authorities. According to a constitutional author,
decentralization of power amounts to "self-immolation," since in that event, the
autonomous government becomes accountable not to the central authorities but to its
constituency.

Under the 1987 Constitution, local government units enjoy autonomy in these two senses,
thus:
Section 1. The territorial and political subdivisions of the Republic of the Philippines are
the provinces, cities, municipalities, and barangays. Here shall be autonomous regions in
Muslim Mindanao ,and the Cordilleras as hereinafter provided.
Sec. 2. The territorial and political subdivisions shall enjoy local autonomy.
Sec. 15. Mere shall be created autonomous regions in Muslim Mindanao and in the
Cordilleras consisting of provinces, cities, municipalities, and geographical areas sharing
common and distinctive historical and cultural heritage, economic and social structures,
and other relevant characteristics within the framework of this Constitution and the
national sovereignty as well as territorial integrity of the Republic of the Philippines.

An autonomous government that enjoys autonomy of the latter category [CONST. (1987),
art. X, sec. 15.] is subject alone to the decree of the organic act creating it and accepted
principles on the effects and limits of "autonomy." On the other hand, an autonomous
government of the former class is, as we noted, under the supervision of the national

Page 139 of 166


government acting through the President (and the Department of Local Government). If
the Sangguniang Pampook (of Region XII), then, is autonomous in the latter sense, its acts
are, debatably beyond the domain of this Court in perhaps the same way that
the internal acts, say, of the Congress of the Philippines are beyond our jurisdiction. But if
it is autonomous in the former category only, it comes unarguably under our jurisdiction.
An examination of the very Presidential Decree creating the autonomous governments of
Mindanao persuades us that they were never meant to exercise autonomy in the second
sense, that is, in which the central government commits an act of self-immolation.
Presidential Decree No. 1618, in the first place, mandates that "[t]he President shall have
the power of general supervision and control over Autonomous Regions." In the second
place, the Sangguniang Pampook, their legislative arm, is made to discharge chiefly
administrative services.

Page 140 of 166


CASE NO. 67
GANZON VS. COURT OF APPEALS
G.R. No. 93252
August 5, 1991

Facts:
Mayor Ganzon has 10 administrative complaints on various charges, among them, abuse
of authority, oppression, grave misconduct, disgraceful and immoral conduct,
intimidation, culpable violation of the Constitution, and arbitrary detention. Secretary of
Local Government (now, Interior) issued an Order suspending Mayor Ganzon from Office.
Mayor Ganzon's primary argument is that the Secretary of Local Government is devoid, in
any event, of any authority to suspend and remove local officials.

Issue:
Whether or not the Secretary of Local Government, as the President's alter ego, can
suspend and/or remove local officials.

Ruling:
YES. The charter did not intend to divest the legislature of its right or the President of her
prerogative as conferred by existing legislation to provide administrative sanctions
against local officials. The Constitution signifies nothing more than to underscore local
governments' autonomy from congress and to break Congress' "control" over local
government affairs. The Constitution did not, however, intend, for the sake of local
autonomy, to deprive the legislature of all authority over municipal corporations, in
particular, concerning discipline.

Autonomy, in the constitutional sense, is subject to the guiding star, though not control,
of the legislature, albeit the legislative responsibility under the Constitution and as the
"supervision clause" itself suggest-is to wean local government units from over-
dependence on the central government. It is noteworthy that under the Charter, "local
autonomy" is not instantly self-executing, but subject to, among other things, the passage
of a local government code, a local tax law, income distribution legislation, and a national
representation law, and measures designed to realize autonomy at the local level. It is also
noteworthy that in spite of autonomy, the Constitution places the local government under
the general supervision of the Executive. It is noteworthy finally, that the Charter allows
Congress to include in the local government code provisions for removal of local officials,
which suggest that Congress may exercise removal powers, and as the existing Local
Government Code has done, delegate its exercise to the President.

Ganzon is under the impression that the Constitution has left the President mere
supervisory powers, which supposedly excludes the power of investigation, and denied

Page 141 of 166


her control, which allegedly embraces disciplinary authority. It is a mistaken impression
because legally, "supervision" is not incompatible with disciplinary authority.

In administration law supervision means overseeing or the power or authority of an officer


to see that subordinate officers perform their duties. If the latter fail or neglect to fulfill
them the former may take such action or step as prescribed by law to make them perform
their duties. Control, on the other hand, means the power of an officer to alter or modify
or nullify of set aside what a subordinate officer had done in the performance of his duties
and to substitute the judgment of the former for that of the latter." But from this, it cannot
be reasonably inferred that the power of supervision of the President over local
government officials does not include the power of investigation when in his opinion the
good of the public service so requires.

"Control" has been defined as "the power of an officer to alter or modify or nullify or set
aside what a subordinate officer had done in the performance of his duties and to
substitute the judgment of the former for test of the latter." "Supervision" on the other
hand means "overseeing or the power or authority of an officer to see that subordinate
officers perform their duties. "Investigating" is not inconsistent with "overseeing",
although it is a lesser power than "altering".

The contention that the President has inherent power to remove or suspend municipal
officers is without doubt not well taken. Removal and suspension of public officers are
always controlled by the particular law applicable and its proper construction subject to
constitutional limitations.
As the Constitution itself declares, local autonomy means "a more responsive and
accountable local government structure instituted through a system of
decentralization." The Constitution as we observed, does nothing more than to break up
the monopoly of the national government over the affairs of local governments and as
put by political adherents, to "liberate the local governments from the imperialism of
Manila." Autonomy, however, is not meant to end the relation of partnership and inter-
dependence between the central administration and local government units, or otherwise,
to user in a regime of federalism. The Charter has not taken such a radical step. Local
governments, under the Constitution, are subject to regulation, however limited, and for
no other purpose than precisely, albeit paradoxically, to enhance self- government.

Decentralization means devolution of national administration but not power to the local
levels. Thus, autonomy is either decentralization of administration or decentralization of
power. There is decentralization of administration when the central government delegates
administrative powers to political subdivisions in order to broaden the base of
government power and in the process to make local governments "more responsive and
accountable," and "ensure their fullest development as self-reliant communities and make

Page 142 of 166


them more effective partners in the pursuit of national development and social progress."
At the same time, it relieves the central government of the burden of managing local
affairs and enables it to concentrate on national concerns. The President exercises
"general supervision" over them, but only to "ensure that local affairs are administered
according to law." He has no control over their acts in the sense that he can substitute
their judgments with his own.

Decentralization of power, on the other hand, involves an abdication of political power in


the favor of local governments units declared to be autonomous, In that case, the
autonomous government is free to chart its own destiny and shape its future with
minimum intervention from central authorities. According to a constitutional author,
decentralization of power amounts to "self-immolation," since in that event, the
autonomous government becomes accountable not to the central authorities but to its
constituency.

The Court is laying down the following rules:


1. Local autonomy, under the Constitution, involves a mere decentralization of
administration, not of power, in which local officials remain accountable to the central
government in the manner the law may provide;
2. The new Constitution does not prescribe federalism;
3. The change in constitutional language (with respect to the supervision clause) was
meant but to deny legislative control over local governments; it did not exempt the latter
from legislative regulations provided regulation is consistent with the fundamental
premise of autonomy;
4. Since local governments remain accountable to the national authority, the latter may,
by law, and in the manner set forth therein, impose disciplinary action against local
officials;
5. "Supervision" and "investigation" are not inconsistent terms; "investigation" does not
signify "control" (which the President does not have);

Page 143 of 166


CASE NO. 68
THE PROVINCE OF BATANGAS VS. ROMULO
G.R. No. 152774
May 27, 2004

Facts:
The Province of Batangas, represented by its Governor, Hermilando I. Mandanas comes
to this Court assailing as unconstitutional and void the provisons in the GAAs of 1999,
2000 and 2001, relating to the LGSEF. The Governor submits that the assailed provisos in
the GAAs and the OCD resolutions, insofar as they earmarked the amount of five billion
pesos of the IRA of the LGUs for 1999, 2000 and 2001 for the LGSEF and imposed
conditions for the release thereof, violate the Constitution and the Local Government
Code of 1991. The petitioner posits that to subject the distribution and release of the five-
billion-peso portion of the IRA, classified as the LGSEF, to compliance by the LGUs with
the implementing rules and regulations, including the mechanisms and guidelines
prescribed by the Oversight Committee, contravenes the explicit directive of the
Constitution that the LGUs' share in the national taxes "shall be automatically released to
them."

Issue:
Whether or not the assailed GAA provisions are unconstitutional

Ruling:
YES. Section 6, Article X of the Constitution is invoked as it mandates that the "just share"
of the LGUs shall be automatically released to them. Sections 18 and 286 of the Local
Government Code of 1991, which enjoin that the "just share" of the LGUs shall be
"automatically and directly" released to them "without need of further action".

Article II of the Constitution, the State has expressly adopted as a policy that:

Section 25. The State shall ensure the autonomy of local governments. An entire article
(Article X) of the Constitution has been devoted to guaranteeing and promoting the
autonomy of LGUs. Section 2 thereof reiterates the State policy in this wise:

Section 2. The territorial and political subdivisions shall enjoy local autonomy. Consistent
with the principle of local autonomy, the Constitution confines the President's power over
the LGUs to one of general supervision. (This provision has been interpreted to exclude
the power of control. )

Section 6, Article X of the Constitution provides that government units shall have a just
share, as determined by law, in the national taxes which shall be automatically released to

Page 144 of 166


them. When parsed, it would be readily seen that this provision mandates that (1) the
LGUs shall have a "just share" in the national taxes; (2) the "just share" shall be determined
by law; and (3) the "just share" shall be automatically released to the LGUs.

The Local Government Code of 1991, among its salient provisions, underscores the
automatic release of the LGUs' "just share".

A basic feature of local fiscal autonomy is the automatic release of the shares of LGUs in
the National internal revenue. This is mandated by no less than the Constitution. The Local
Government Code specifies further that the release shall be made directly to the LGU
concerned within five (5) days after every quarter of the year and "shall not be subject to
any lien or holdback that may be imposed by the national government for whatever
purpose." As a rule, the term "SHALL" is a word of command that must be given a
compulsory meaning. The provision is, therefore, IMPERATIVE.

The entire process involving the distribution and release of the LGSEF is constitutionally
impermissible. The LGSEF is part of the IRA or "just share" of the LGUs in the national
taxes. To subject its distribution and release to the vagaries of the implementing rules and
regulations, including the guidelines and mechanisms unilaterally prescribed by the
Oversight Committee from time to time, as sanctioned by the assailed provisos in the
GAAs of 1999, 2000 and 2001 and the OCD resolutions, makes the release not automatic,
a flagrant violation of the constitutional and statutory mandate that the "just share" of
the LGUs "shall be automatically released to them." The LGUs are, thus, placed at the
mercy of the Oversight Committee.

Where the law, the Constitution in this case, is clear and unambiguous, it must be taken
to mean exactly what it says, and courts have no choice but to see to it that the mandate
is obeyed. Moreover, as correctly posited by the petitioner, the use of the word "shall"
connotes a mandatory order. Its use in a statute denotes an imperative obligation and is
inconsistent with the idea of discretion.

Local autonomy includes both administrative and fiscal autonomy. Local fiscal autonomy
includes the power of the LGUs to, inter alia, allocate their resources in accordance with
their own priorities. Under existing law, local government units, in addition to having
administrative autonomy in the exercise of their functions, enjoy fiscal autonomy as well.
Fiscal autonomy means that local governments have the power to create their own
sources of revenue in addition to their equitable share in the national taxes released by
the national government, as well as the power to allocate their resources in accordance
with their own priorities. It extends to the preparation of their budgets, and local officials
in turn have to work within the constraints thereof. They are not formulated at the national
level and imposed on local governments, whether they are relevant to local needs and

Page 145 of 166


resources or not. Further, a basic feature of local fiscal autonomy is the constitutionally
mandated automatic release of the shares of LGUs in the national internal revenue. The
value of local governments as institutions of democracy is measured by the degree of
autonomy that they enjoy.

Page 146 of 166


CASE NO. 69
REPUBLIC VS. DAVAO CITY
G.R. No. 148622
September 12, 2002

Facts:
Respondent filed an application for a Certificate of Non-coverage for its proposed project,
the Davao City Artica Sports Dome, with the Environmental Management Bureau.
However, the petitioner denied such application on the grounds that, as an agent of the
government, the respondent is required to secure an Environmental Compliance
Certificate which they failed to do so. The respondent filed a petition for mandamus with
the RTC of Davao to compel the DENR to issue the Certificate of Non-coverage. The RTC
ruled in favor of the respondent, and ratiocinated its decision on the grounds that the
respondent is not an agent or instrumentality of the National Government hence it is not
required to secure the Environmental Compliance Certificate. Furthermore, it was ruled
that the project is not an environmentally critical one. Hence this petition.

Issue:
Whether the respondent is an agent of the National government

Ruling:
Yes. The Court emphasized that Local Government Units performs dual functions, which
are Governmental and Proprietary. When a local government unit performs its
governmental functions, it acts as an agent of the National Government. On the other
hand, if the local government unit exercise its proprietary functions, it acts as an agent of
its constituents.

In the case at bar, considering that the respondent is an agent or instrumentality of the
National Government, it is required by law that if its project is an environmentally critical
one or within an environmentally critical area, then it is required to secure an
Environmental Compliance Certificate.

However, based on the evidence provided, the project of the respondent is deemed not
to be an environmentally critical one hence it is not necessary for it to secure the
Environmental Compliance Certificate.

Page 147 of 166


CASE NO. 70
Pollution Adjudicatory Board VS CA
G.R. No. 93891
March 11, 1991

Facts:
Petitioner board issued an ex parte cease and desist order against herein respondent from
operating its plant which allegedly discharges untreated waterwaste directly to the canals
and rivers. By virtue of several inspections conducted by the petitioner board, they ruled
that the respondent plant failed to comply with the standards laid down for the proper
disposal of polluted waste. The respondent filed a case against the petitioner to nullify
the said cease and desist order. The trial court ruled in favor of the petitioner, finding the
respondent to have violated the standards set by law for proper waste disposal. The case
was elevated to the CA, which reversed the decision of the trial court, and ruled in favor
of the respondent on the grounds that the latter was denied of its right to due process.

Issue:
Whether or not the petitioner erred in issuing ex parte the cease and desist order

Ruling:
No. The Court ruled that as an agency of the Government, whose sole duty is to monitor
establishments regarding their discharge of effluents or waste water, it has the legal
authority to issue an ex parte cease and desist order against any establishment if there is
a prima facie evidence that such discharge exceeds the standards set by law and it
imposes a grave threat to the environment which the Government is obliged to preserve
and protect.

In the case at bar, it was proved that after several inspections conducted by the petitioner
board, the respondent indeed discharges waste water exceeding the standards prescribed
by the law which is harmful to the environment. Furthermore, due to these inspections
and several prior orders from the board to the respondent regarding the same issue, the
latter cannot insist that it was deprived of its right to due process.

Page 148 of 166


CASE NO. 71
RAMOS VS. CA
G.R. No. L-53766
October 30, 1981
Facts:
The petitioners filed a case against respondent Municipality of Baliuag for the declaration
of nullity of a municipal ordinance which it enacted regarding a contract of lease over a
commercial arcade which the latter entered into. The case was set for trial, and a hearing
was conducted wherein private respondents the provincial fiscal Atty. Regalado and a
private counsel Atty. Romanillos appeared as counsel of the respondent municipality.
During the proceedings, Atty. Romanillos represented the municipality of Baliuag as
collaborating counsel of the provincial fiscal. The petitioner now contests the validity of
the respondent’s representation of the said municipality, and filed a motion for his
disqualification and for the annulment of all proceedings which he participated in as the
representative of the municipality in the said case. The petitioner insists that such
representation is unauthorized by law because the law expressly provides that a
municipality should only be represented by the Provincial fiscal and municipal attorneys.
Pursuant to such motion, Atty. Regalado, the provincial fiscal, took over the case and
replaced Atty. Romanillos as the representative of the municipality of Baliuag.

Issue:
Whether a municipality may be represented by a private counsel

Ruling:
No. The law expressly provides that only the provincial fiscal and municipal attorneys may
represent a municipality or its political subdivisions, and only in special cases may it be
represented by a private counsel such as when a municipality is a party adverse to the
provincial government.

In the case at bar, Atty. Romanillos in his capacity as a private counsel in the said case,
cannot represent the municipality of Baliuag and does not fall under the exceptions
provided for by law. The fact that Atty. Romanillos acted as a collaborating counsel of
Atty. Regalado cannot justify his representation to the municipality hence it is void.

However, notwithstanding such violation, the proceedings where Atty. Romanillo


participated in should not be nullified for it would be contrary to the disposition of justice.
The Court emphasized that the disposition of justice should not be impaired by mere
technicalities. Furthermore, the fact that Atty. Regalado replaced Atty. Romanillo and
acted as the representative of the municipality after the filing of the motion to disqualify
the latter validates such act, hence there is no point in nullifying such proceedings and
conducting the same all over again.

Page 149 of 166


CASE NO. 72
THE PEOPLE OF THE PHILIPPINES VS. REMIGIO B. CHAN
G.R. No. L-45435
June 17, 1938

Facts:
On April 17, 1935 Ordinance No. 2347 was approved. In section 1 it provides that all first
run theatres or cinematographs should register their seating capacity with the City
Treasurer, and in section 2 it prohibits the sale of tickets in said theatres or
cinematographs in excess of their registered seating capacity.

The accused Remigio B. Chan, as manager of the Capitol Theatre, a first class
cinematograph located on the Escolta, Manila, was charged and sentenced in the
municipal court to pay a fine for having sold to the public tickets in excess of seating
capacity of said cinematograph.

Issue:
The legal question involved in this appeal is whether a municipal ordinance which
prohibits the sale by first run cinematographs of tickets in excess of their seating capacity,
is discriminatory and, therefore, unconstitutional.

Ruling:
The prohibition applies with equal force wherever the same reason exists, that is, to first-
and second-class theatres which show films for the first time. The City of Manila
exercises police power by delegation and that in the exercise of that power, it is
authorized to enact ordinances for the regulation of the operation of theatres and
cinematographs. The appealed order is hereby reversed and it is ordered that the case
be remanded to the Court of First Instance of Manila for further proceedings.

Page 150 of 166


CASE NO. 73
RESTITUTO YNOT vs. INTERMEDIATE APPELLATE COURT
G.R. No. 74457
March 20, 1987

Facts:
The Petitioner challenges the constitutionality of Executive Order No. 626-A prohibiting
the interprovincial movement of carabaos and the slaughtering of carabaos. The
petitioner had transported six carabaos in a pump boat from Masbate to Iloilo when they
were confiscated by the police station commander of Barotac Nuevo, Iloilo, for violation
of the above EO.

The petitioner sued for recovery, and the Regional Trial Court of Iloilo City issued a writ
of replevin upon his filing of a supersedeas bond of P12,000.00. After considering the
merits of the case, the court sustained the confiscation of the carabaos and, since they
could no longer be produced, ordered the confiscation of the bond. The court also
declined to rule on the constitutionality of the executive order, as raise by the petitioner,
for lack of authority and also for its presumed validity.

Issue:
Whether executive order is unconstitutional insofar as it authorizes outright confiscation
of the carabao or carabeef being transported across provincial boundaries.

Ruling:
Executive Order No. 626-A is declared unconstitutional. We find that the challenged
measure is an invalid exercise of the police power because the method employed to
conserve the carabaos is not reasonably necessary to the purpose of the law and, worse,
is unduly oppressive. Due process is violated because the owner of the property
confiscated is denied the right to be heard in his defense and is immediately condemned
and punished. The conferment on the administrative authorities of the power to adjudge
the guilt of the supposed offender is a clear encroachment on judicial functions and
militates against the doctrine of separation of powers. There is, finally, also an invalid
delegation of legislative powers to the officers mentioned therein who are granted
unlimited discretion in the distribution of the properties arbitrarily taken. For these
reasons, we hereby declare Executive Order No. 626-A unconstitutional.

Page 151 of 166


CASE NO. 74
RAMON FABIE, ET AL. VS. THE CITY OF MANILA
G.R. No. L-6583
February 16, 1912

Facts:
The City of Manila enacted Ordinance No. 124 as an amendment of section 107 of the
Revised Ordinances of the city of Manila, enacted on June 13, 1908 relating to the issuance
of permits for the erection of buildings.

Plaintiffs-appellees Ramon Fabie, et al. sought to obtain from the city of Manila a building
permit authorizing the construction of a small nipa house upon a large tract of land which
forms a part of the estate known as the Hacienda de Santa Ana de Sapa and which is
inclosed between Calle Herran of the District of Paco and an estero known as Tripa de
Gallina, and lying within the corporate limits of the city of Manila.

The permit was denied by the city authorities on the ground that the site of the proposed
building did not conform to the requirements of section 107 of the Revised Ordinances
of the city of Manila, as amended by Ordinance No. 124, which provides: "That the
building shall abut or face upon a public street or alley or on a private street or alley which
has been officially approved."

Issue:
Whether or not Ordinance No. 124 is valid.

Ruling:
YES. To justify the State in the exercise of it police powers on behalf of the public, it must
appear that the interests of the public generally, as distinguished from those of a
particular class, require such interference; and, second, that the means are reasonably
necessary for the accomplishment of the purpose, and not unduly oppressive upon
individuals.

In the present case, the provision that denies permits for the construction of buildings
within the city limits unless they "abut or face upon a public street or alley or on a private
street or alley which has been officially approved," is reasonably necessary to secure the
end in view.

In the first place it prevents the huddling and crowding of buildings in irregular masses
on single or adjoining tracts of land, and secures an air space on at least one side of each
new residence or other building constructed in the city. The menace to the health and
safety of the residents of Manila resulting from the crowding of nipa shakes, and even

Page 152 of 166


more substantial buildings upon small tracts of land is a matter of common knowledge;
and in a community, exposed as this city is to destructive conflagrations and epidemic
diseases, a legislative measures which tends to prevent the repitition of such unfortunate
conditions should not be judicially declared to be unreasonable, in the absence of the
most compelling reasons.

In the second place, the provisions of the ordinance in question manifestly promote the
safety and security of the citizens of Manila and of their property against fire and disease,
especially epidemic disease, by securing the easy and unimpeded approach to all new
buildings: First, of fire engines, and other apparatus for fighting fire; second, of
ambulances, refuse wagons, and apparatus used by the sanitary department in caring for
the sanitation of the city; third, of fire and health inspectors generally; of employees of
the fire department and others engaged in fighting fire; and of employees of the Bureau
of Health engaged in their duty as guardians of the sanitary conditions and general health
of the city.

Page 153 of 166


CASE NO. 75
FORTICH VS. CORONA
G.R. No. 131457
November 17, 1998

Facts:
This case involves a 144-hectare land located at San Vicente, Sumilao, Bukidnon, owned
by the Norberto Quisumbing, Sr. Management and Development Corporation
(NQSRMDC), one of the petitioners. In 1984, the land was leased as a pineapple plantation
to the Philippine Packing Corporation, now Del Monte Philippines, Inc. (DMPI) for a period
of ten (10) years. The lease expired in April, 1994. During the existence of the lease, the
Department of Agrarian Reform (DAR) placed the entire 144-hectare property under
compulsory acquisition and assessed the land value at P2.38 million. NQSRMDC sought
and was granted by the DAR Adjudication Board (DARAB), through its Provincial Agrarian
Reform Adjudicator (PARAD) a writ of prohibition with preliminary injunction which
ordered the DAR Region X Director, the Provincial Agrarian Reform Officer (PARO) of
Bukidnon, the Municipal Agrarian Reform Office (MARO) of Sumilao, Bukidnon, the Land
Bank of the Philippines (Land Bank), and their authorized representatives "to desist from
pursuing any activity or activities" concerning the subject land "until further orders."

Despite the DARAB order of March 31, 1992, the DAR Regional Director issued a
memorandum directing the Land Bank to open a trust account for P2.38 million in the
name of NQSRMDC and to conduct summary proceedings to determine the just
compensation of the subject property. NQSRMDC objected to these moves and filed an
Omnibus. Motion to enforce the DARAB order of March 31, 1992 and to nullify the
summary proceedings undertaken by the DAR Regional Director and Land Bank on the
valuation of the subject property. DARAB acted favorably on the Omnibus Motion. Land
Bank complied with the DARAB. In the meantime, the Provincial Development Council
(PDC) of Bukidnon, headed by Governor Carlos O. Fortich passed Resolution No. 6, dated
January 7, 1993, designating certain areas along Bukidnon-Sayre Highway as part of the
Bukidnon Agro-Industrial Zones where the subject property is situated.

Pursuant to Section 20 of R.A. No. 7160, otherwise known as the Local Government Code,
the Sangguniang Bayan of Sumilao, Bukidnon, on March 4, 1993, enacted Ordinance No.
24 converting or re-classifying 144 hectares of land in Bgy. San Vicente, said Municipality,
from agricultural to industrial/institutional with a view of providing an opportunity to
attract investors who can inject new economic vitality, provide more jobs and raise the
income of its people. Under said section, 4th to 5th class municipalities may authorize the
classification of five percent (5%) of their agricultural land area and provide for the manner
of their utilization or disposition.

Page 154 of 166


On 11 December 1993, the instant application for conversion was filed by Mr. Gaudencio
Beduya in behalf of NQSRMDC/BAIDA (Bukidnon Agro-Industrial Development
Association). This was favorably recommended by a lot government officials. The people
of the affected barangay even rallied behind their respective officials in endorsing the
project. Notwithstanding the foregoing favorable recommendation, however, on
November 14, 1994, the DAR, thru Secretary Garilao, invoking its powers to approve
conversion of lands under Section 65 of R.A. No. 6657, issued an Order denying the instant
application for the conversion of the subject land from agricultural to agro-industrial and,
instead, placed the same under the compulsory coverage of CARP and directed the
distribution thereof to all qualified beneficiaries. Motion for Reconsideration of the
aforesaid Order was filed by applicant but the same was denied. Thus, the DAR Secretary
ordered the DAR Regional Director "to proceed with the compulsory acquisition and
distribution of the property."

Governor Carlos O. Fortich of Bukidnon appealed" the order of denial to the Office of the
President and prayed for the conversion/reclassification of the subject land as the same
would be more beneficial to the people of Bukidnon. The Office of the President, through
then Executive Secretary Ruben D. Torres, issued a Decision in OP Case No. 96-C-6424,
dated March 29, 1996, reversing the DAR Secretary's decision. It stated that: deciding in
favor of NQSRMDC, the DARAB correctly pointed out that under Section 8 of R.A. No.
6657, the subject property could not validly be the subject of compulsory acquisition until
after the expiration of the lease contract with Del Monte Philippines, a Multi-National
Company, or until April 1994. xxx the language of Section 20 of R.A. No. 7160, supra, is
clear and affords no room for any other interpretation. By unequivocal legal mandate, it
grants local government units autonomy in their local affairs including the power to
convert portions of their agricultural lands and provide for the manner of their utilization
and disposition to enable them to attain their fullest development as self- reliant
communities.

Some alleged farmer-beneficiaries began their hunger strike in front of the DAR
Compound in Quezon City to protest the OP Decision of March 29, 1996. On November
7, 1997, the Office of the President resolved the strikers' protest by issuing the so-called
"Win/Win" Resolution penned by then Deputy Executive Secretary Renato C. Corona
which modified the decision of the Office of the President, through Executive Secretary
Ruben Torres, dated March 29, 1996:

 NQSRMDC's application for conversion is APPROVED only with respect to the


approximately forty-four (44) hectare portion of the land adjacent to the highway
 Remaining approximately one hundred (100) hectares traversed by an irrigation
canal and found to be suitable for agriculture shall be distributed to qualified
farmer-beneficiaries in accordance with RA 6657 or the Comprehensive Agrarian

Page 155 of 166


Reform Law with a right of way to said portion from the highway provided in the
portion fronting the highway.

A copy of the "Win-Win" Resolution was received by Governor Carlos O. Fortich of


Bukidnon, Mayor Rey B. Baula of Sumilao, Bukidnon, and NQSRMDC on November 24,
1997 and, on December 4, 1997, they filed the present petition for certiorari, prohibition
and injunction with urgent prayer for a temporary restraining order and/or writ of
preliminary injunction against then Deputy Executive Secretary Renato C. Corona and DAR
Secretary Ernesto D. Garilao. Respondents, through the Solicitor General, opposed the
petition and prayed that it be dismissed outright

Issue:
Whether or not the power of the local government units to reclassify lands is subject to
the approval of the DAR

Ruling:
NO. The issue of whether or not the power of the local government units to reclassify
lands is subject to the approval of the DAR is no longer novel, this having been decided
by this Court in the case of Province of Camarines Sur, et al. vs. Court of Appeals wherein
we held that local government units need not obtain the approval of the DAR to convert
or reclassify lands from agricultural to non-agricultural use.

Page 156 of 166


CASE NO. 76
STA. ROSA REALTY DEVELOPMENT CORPORATION VS. COURT OF APPEALS
G.R. No. 112526
October 12, 2001

Facts:
The case is a petition regarding Department of Agrarian Reform Adjudication Board’s
(DARAB) order of compulsory acquisition of petitioner’s property under the
Comprehensive Agrarian Reform Program (CARP). Sta. Rosa was the registered owner of
two parcels of land in Cabuyao Laguna. According to them, these lands are watersheds
which provide clean and potable (drinkable) water to the Canlubang community and that
90 light industries are located in that area. They were alleging respondents usurped its
rights over their property thereby destroying the ecosystem. Since the said land provides
water to the residents, respondents sought an easement of a right of a way to and from
Barangay Castile, to which, by counterclaim, Sta. Rosa sought ejectment against
respondents. Respondents went to the DAR and filed a case for compulsory acquisition
of the Sta. Rosa Property under the Comprehensive Agrarian Reform Program.
Compulsory acquisition is the power of the government to acquire private rights in land
without the willing consent of its owner or occupant in order to benefit the society. The
said land was inspected by the Municipal and Agrarian Reform Officer, and upon
consensus of the authorities concerned, they decided that the said land must be placed
under compulsory acquisition. Petitioners filed an objection on the ground that: The area
is not appropriate for agricultural purposes. The area was rugged in terrain with slopes
18% and above. (which falls under the exception in compulsory acquisition of CARP) The
occupants of the land were illegal settlers or (squatters) who by no means are entitled to
the land as beneficiaries. Another issue raised by the petitioners was that the DAR failed
to follow the due process because instead of paying just compensation, a trust account
was made in favour of the petitioners.

Issue:
Whether or not the petition for land conversion of the parcels of land may be granted.

Ruling:
For a valid implementation of the CARP Program, two notices are required: (1) the notice
of coverage and letter of invitation to a preliminary conference sent to the landowner, the
representative of the BARC, LBP, farmer beneficiaries and other interested parties
pursuant to DAR A. O. No. 12, series of 1989; and (2) the notice of acquisition sent to the
landowner under Section 16 of the CARL. In the case at bar, DAR has executed the taking
of the property in question. However, payment of just compensation was not in
accordance with the procedural requirement. The law required payment in cash or LBP
bonds, not by trust account as was done by DAR.

Page 157 of 166


In Association of Small Landowners in the Philippines v. Secretary of Agrarian Reform, we
held that "The CARP Law, for its part, conditions the transfer of possession and ownership
of the land to the government on receipt of the landowner of the corresponding payment
or the deposit by the DAR of the compensation in cash or LBP bonds with an accessible
bank. Until then, title also remains with the landowner. No outright change of ownership
is contemplated either." Consequently, petitioner questioned before the Court of Appeals
DARAB's decision ordering the compulsory acquisition of petitioner's property. Here,
petitioner pressed the question of whether the property was a watershed, not covered by
CARP.

Watersheds may be defined as "an area drained by a river and its tributaries and enclosed
by a boundary or divide which separates it from adjacent watersheds." Watersheds
generally are outside the commerce of man, so why was the Casile property titled in the
name of SRRDC? The answer is simple. At the time of the titling, the Department of
Agriculture and Natural Resources had not declared the property as watershed area. The
parcels of land in Barangay Casile were declared as "PARK" by a Zoning Ordinance
adopted by the municipality of Cabuyao in 1979, as certified by the Housing and Land
Use Regulatory Board. On January 5, 1994, the Sangguniang Bayan of Cabuyao, Laguna
issued a Resolution voiding the zoning classification of the land at Barangay Casile as Park
and declaring that the land is now classified as agricultural land.

The authority of the municipality of Cabuyao, Laguna to issue zoning classification is an


exercise of its police power, not the power of eminent domain. "A zoning ordinance is
defined as a local city or municipal legislation which logically arranges, prescribes, defines
and apportions a given political subdivision into specific land uses as present and future
projection of needs."

The parcels of land in Barangay Casile were declared as “PARK” by a Zoning Ordinance
adopted by the municipality of Cabuyao in 1979, as certified by the Housing and Land
Use Regulatory Board. On January 5, 1994, the Sangguniang Bayan of Cabuyao, Laguna
issued a Resolution voiding the Zoning classification of the lands at Barangay Casile as
Park and declaring that the land was now classified as agricultural land. The authority of
the municipality of Cabuyao, Laguna to issue zoning classification is an exercise of its
police power, not the power of eminent domain. “A zoning ordinance is defined as a local
city or municipal legislation which logically arranges, prescribes, defines and apportions a
given political subdivision into specific land uses as present and future projection of
needs.”

Page 158 of 166


CASE NO. 77
JOSE C. MIRANDA VS. HON. ALEXANDER AGUIRRE
G.R. No. 133064
September 16, 1999

Facts:
This is a petition for a writ of prohibition with prayer for preliminary injunction assailing
the constitutionality of Republic Act No. 8528 converting the city of Santiago, Isabela from
an independent component city to a component city. Petitioners assail the
constitutionality of R.A. No. 8528.2They alleged as ground the lack of provision in R.A. No.
8528 submitting the law for ratification by the people of Santiago City in a proper
plebiscite. Petitioner Miranda was the mayor of Santiago at the time of the filing of the
petition at bar. Petitioner Afiado is the President of the Liga ng mga Barangay ng Santiago
City. Petitioners Dirige, Cabuyadao and Babaran are residents of Santiago City.

Issues:
A. Whether or not the petitioners, mayor of Santiago, Isabela have locus standi
B. Whether R.A. No. 8528 is unconstitutional for its failure to be submitted to its people
in a proper plebiscite.

Ruling:
A. YES. Petitioner Miranda was the mayor of Santiago City when he filed the present
petition in his own right as mayor and not on behalf of the city, hence, he did not need
the consent of the city council of Santiago City. It is also indubitable that the change
of status of the city of Santiago from independent component city to a mere
component city will affect his powers as mayor, as will be shown hereafter. The injury
that he would sustain from the enforcement of R.A. No. 8528 is direct and immediate
and not a mere generalized grievance shared with the people of Santiago City.

B. YES. Section 10, Article X addressed the undesirable practice in the past whereby local
government units were created, abolished, merged or divided on the basis of the
vagaries of politics and not of the welfare of the people. Thus, the consent of the
people of the local government unit directly affected was required to serve as a
checking mechanism to any exercise of legislative power creating, dividing, abolishing,
merging or altering the boundaries of local government units. It is one instance where
the people in their sovereign capacity decide on a matter that affects them—direct
democracy of the people as opposed to democracy thru people’s representatives. This
plebiscite requirement is also in accord with the philosophy of the Constitution
granting more autonomy to local government units.

Page 159 of 166


Relevant Doctrines:
Section 10, Chapter 2 of the Local Government Code (R.A. No. 7160), thus:
“Sec. 10. No province, city, municipality, or barangay may be created, divided, merged,
abolished, or its boundary substantially altered except in accordance with the criteria
established in the local government code and subject to approval by a majority of the
votes cast in a plebiscite in the political units directly affected.”

Plebiscite—(1) no creation, conversion, division, merger, abolition, or substantial


alteration of boundaries of LGUS shall take effect unless approved by a majority of the
votes cast in a plebiscite called for the purpose in the LGU or LGUs affected. The plebiscite
shall be conducted by the Commission on Elections (COMELEC) within one hundred
twenty (120) days from the effectivity of the law or ordinance prescribing such action,
unless said law or ordinance fixes another date.

Page 160 of 166


CASE NO. 78
ORDILLO VS. COMMISSION ON ELECTIONS
G.R. No. 93054
December 4, 1990
Facts:
On January 30, 1990, the people of the provinces of Benguet, Mountain Province, Ifugao,
Abra and Kalinga-Apayao and the city of Baguio cast their votes in a plebiscite held
pursuant to Republic Act No. 6766 entitled "An Act Providing for an Organic Act for the
Cordillera Autonomous Region." The official COMELEC results of the plebiscite showed
that the creation of the Region was approved by a majority of 5,889 votes in only the
Ifugao Province and was overwhelmingly rejected by 148,676 votes in the rest of the
provinces and city above-mentioned. As a result of this, on March 8, 1990, Congress
enacted Republic Act No. 6861 setting the elections in the Cordillera Autonomous Region
of Ifugao. The petitioners maintain that there can be no valid Cordillera Autonomous
Region in only one province as the Constitution and Republic Act No. 6766 require that
the said Region be composed of more than one constituent unit.

Issue:
Whether or not Ifugao can validly constitute the Cordillera Autonomous Region

Ruling:
No. Provinces, cities, municipalities and geographical areas connote that "region" is to be
made up of more than one constituent unit. The term "region" used in its ordinary sense
means two or more provinces. Ifugao is a province by itself. To become part of a region,
it must join other provinces, cities, municipalities, and geographical areas. It joins other
units because of their common and distinctive historical and cultural heritage, economic
and social structures and other relevant characteristics. The Constitutional requirements
are not present in this case.

Relevant Doctrine:
Article X, Section 15 of the 1987 Constitution that:
"Section 15. There shall be created autonomous regions in Muslim Mindanao and in the
Cordillera consisting of provinces, cities, municipalities and geographical areas sharing
common and distinctive historical and cultural heritage, economic and social structures,
and other relevant characteristics within the framework of this Constitution and the
national sovereignty as well as territorial integrity of the Republic of the Philippines."

Page 161 of 166


CASE NO. 79
ARONG VS. RAFFINAN
G.R NO. L-8673
February 18, 1956

Facts:
This is an appeal from a decision rendered in two cases which were jointly tried by the
Court of First Instance of Cebu. In one Pedro P. Arong, as owner of Liberty Theater
operated in the City of Cebu, impleaded the mayor and treasurer of said city to recover
the sum of P11,021.22 paid under protest as license fee collected under Ordinance No.
43, series of 1947, on the ground that said ordinances are illegal and oppressive and were
enacted in violation of the City Charter (Case No. R-728). In the other, John D. Young and
six other owners and operators of theaters in the city likewise impleaded the same city
officials to recover the amounts they had paid under protest which were collected as
license fees under the same ordinances on the same ground that they are illegal and ultra
vires and were enacted in violation of the City Charter (Case No. R-1113).

Defendants alleged as special defense that the amounts collected from the plaintiffs are
not taxes but license fees and so the enactment of the aforesaid ordinances comes within
the power granted to the City of Cebu by its charter and are not ultra vires.

The City of Cebu contends that they merely impose an additional license fee in order to
raise finds to cover the expenses that are entailed by the city in carrying out its duty to
supervise and protect the theaters and different places of amusements that are operated
within its limits against fire hazards and other risk incidental to their business.

The issues raised in the two cases are common both to plaintiffs and defendants, the
parties submitted a stipulation of facts without prejudice of presenting additional
evidence. On March 6, 1953, the court rendered decision declaring the two ordinances
valid and absolving the defendants from the complaint. From this decision the plaintiffs
have appealed.

Issues:
A. Whether or not the City of Cebu has the power to approve the ordinances?
B. Whether or not the ordinances can impose a tax for purposes of revenue or merely fix
a license fee for purposes of regulations?

Ruling:
A. The provisions are embodied in section 17 (1) of Commonwealth Act No. 58:
"Sec. 17. General powers and duties of the Board. Except as otherwise provided by
law, and subject to the conditions and limitation thereof the Municipal Board shall

Page 162 of 166


have the following legislative powers:

To regulate and fix the amount of the license fees for the following: Hawkers,
peddlers, hucksters, not including hucksters or peddlers who sell only native
vegetables, fruits, or foods, personally carried by the hucksters or peddlers etc...

B. The ordinances in question are ultra vires. In the cases of Eastern Theatrical Company,
Inc., v. Victor Alfonso, et al., 46 Off. Gaz., (Supplement) p. 303, and City of Baguio, v.
Jose Y. de la Rosa, et al., G.R. No. L-8288-70, this Court has held that an ordinance
enacted which imposes a fee on every price of admission tickets sold by
cinematographs, theaters, vaudeville companies, theatrical shows and boxing
exhibitions, in addition to other license fees paid by the same enterprises is one which
imposes a tax on business and is not merely regulatory. In the case at bar, the City of
Cebu enacted similar ordinance but as its charter gives only the City of Cebu power to
regulate and fix the amount of theaters, the said ordinances posed on the business of
theaters, the said ordinances are ultra vires and in violations in the City charter.

Page 163 of 166


CASE NO. 80
GRINO VS. COMELEC
G.R. No. 105120
September 4, 1992

Facts:
Local Government code was enacted, and the sub-province of Iloilo will be considered a
new province of Guimaras. Because of the new law, the COMELEC conducted a Plebiscite
in the affected areas. Forms were given in the day of election but the said forms did not
contain any space or provision that will allow the voters to choose on who will be their
Provincial officers such as Governor, vice Governor and Sanggunian. Because of that LPD
party list and Grino a candidate for the seat of Governor filed a case against COMELEC
saying that they did not allow the populant to vote for their local official if the said
formation of a new province will not push through. In their answer, Comelec said that
since there will be a new province of Guimaras, the code said that the incumbent official
will remain in their seat and then the president will appoint the officials

Issue:
Whether or not there is a failure of election in the said area

Ruling:
Yes there is. Upon the examination of the law, the Supreme court said that the law fails
to see the scenario when there is a negative vote ( yung hindi sila pumayag sa creation
ng bagong province) the court said that negative votes comes naturally.

They said that the people should have given a chance to vote that if their vote is no then
it should be counted and if they said yes then it should not be counted. However since
the populates majority vote is yes, new province is made and that officials where already
have been appointed by the president it is no longer material to change what the
Comelec did. The case was dismissed

Page 164 of 166


CASE NO. 81
TAN VS. COMMISSION ON ELECTIONS
G.R. No. L-73155
July 11, 1986

Facts:
BATAS BILANG 885 was enacted that seeks the establishments of a new Province to be
called Negros Del Norte. The said province will include the 3 major cities and 8
municipalities of the Parent Province, Negros occidental. A plebiscite was conducted but
the only people that were allowed to vote was only the people from the 3 major cities
and 8 municipalities that will be inside the new Province of Negros Del Norte with the
exclusion of the other cities and municipality of Negros occidental. Petitioners files a case
to stop the said formation of a new province saying that it is unconstitutional because the
other Cities and Municipality of the Parent Province were not allowed to vote. They said
that according to the constitution the term “unit or units affected” mean that all of the
people from Negros Occidental should be included to the plebescite.

However, the Respondents relying on the of case Paredes vs. the Honorable Executive
Secretary that the court allowed the some cities and municipality of Bulacan and Rizal to
be part of metropolitan manila and that only the people of those cities and municipality
were allowed to cast vote. For the respondents the meaning of “units or units affected”
means only the people who wants to form a new local government unit. That they should
be allowed to do so away from others to promote local autonomy.

Issue:
Whether or not Batas Bilang 885 is Constitutional?

Ruling:
No. The Supreme Court held that the case is not a good president because there are
different scenario. The case that was used by the respondents involves small barangays
to create municipalities that would be joining the metropolitan manila. The court allowed
that to happen because it is for the betterment of the economy of the local government
unit that will join metro manila. However in this case it is the other way around because if
the new province will form, then the Parent Province will be deprived of its long
established cities, and that will affect the economy of the province. Finally, the plebiscite
is void because it exclude the people of the Parent Province from participating from
voting. The term Units or Units affected means all from the Parent Province.

Page 165 of 166


CASE NO. 82
PADILLA VS. COMELEC
G.R. No. 103328
October 19, 1992

Facts:
RA no. 7155 was created to form the municipality of tulay-na-lupa in the Province of
Camarines Norte to be Composed of Barangays Tulay-na-lupa, luigi, San Antonio, mabili
I, Napaod, Benit, Bayang-Bayan, Matanlang, Pag-asa, Maot and CAlabasa, all in the
Municipality of Labo. According to the law, COMELEC issued a resolution for the conduct
of plebiscite. The said resolution provides that the plebiscite shall be held in the areas of
affected units namely the barangay comprising the proposed new municipality and the
remaining areas of the Mother Municipality.

In the plebiscite held throughout the municipality of Labo, majority of the vote were
against the creation of the new municipality. Petitioner as Governor of Camarines Norte,
seeks to set aside the plebiscite conducted throughout the municipality of Labo and prays
that a new plebiscite be under taken. It is the contention of petitioner that the plebiscite
was a complete failure and that the result obtained were invalid and illegal because the
plebiscite should have not include the whole municipality but only those who will create
their own.

Issue:
Whether or not the Plebiscite is null and void?

Ruling:
Yes. When the law states that the plebiscite shall be conducted in the political units
directly affected it means that resident of the political entity who would economically
dislocated by the separation of a portion thereof have a right to vote in said plebiscite.
Evidently what is contemplated by the phase political units directly affected is the plurality
of political units which would participate in the plebiscite. Logically, those to be included
in such political areas are the inhabitants of the 12 barangays of the proposed municipality
of tulay-na-lupa as well as those living in the parent municipality of Labo, Camarines Norte
thus, it was concluded that the Act of Comelec is proper.

Page 166 of 166

Вам также может понравиться